Cell Biology Final

Ace your homework & exams now with Quizwiz!

How many steps in glycolysis involve the action of a kinase? - 4. - 6. - 10. - 2. - 0. - 1.

4.

Export of RNA from the nucleus requires the RNA to have which characteristic(s)? - 5' cap. - Poly-A tail. - 5' cap and poly-A tail. - Introns.

5' cap and poly-A tail.

The nucleotide sequence of one DNA strand in a DNA double helix is 5'-CATTGCCAGAAAAAT-3'. What is the sequence of the complementary strand produced during replication? - 5'-TAAAAAGACCGTTAC-3' - 5'-GATTCGGAGTTTTTA-3' - 5'-CATTGCCAGAAAAAT-3' - 5'-ATTTTTCTGGCAATG-3' - 5'-GTAACGGTCTTTTTA-3'

5'-ATTTTTCTGGCAATG-3'

The sequence of the template strand of a DNA molecule is 5'-ACTGGCAATG-3'. What is the sequence of the RNA transcribed from this DNA? - 5'-ACUGGCUUAC-3' - 5'-GUAACGGUCA-3' - 5'-TGACCGTTAC-3' - 5'-CAUUGCCAGU-3' - 5'-UGACCGUUAC-3'

5'-CAUUGCCAGU-3'

How many kinetochores are present in a human cell during mitosis? - 2. - 23. - 46. - Thousands. - 92.

92.

What structure in an interphase eukaryotic cell is the site of ribosomal RNA transcription? - Ribosome. - Nucleolus. - Nucleosome. - Nuclear lamina.

- Nucleolus.

Once heterochromatin has been established, it will often spread until it encounters which of the following? - A gene. - Histone H3 methylated on lysine 9. - Euchromatin. - A barrier DNA sequence. - A nucleosome

A barrier DNA sequence.

At what site do all charged tRNAs (with the exception of the initiator tRNA) first bind on the ribosome? - P site. - In any of the three sites (A, P, or E). - E site. - A site.

A site.

Proteins have to be unfolded to cross the membranes of which of these organelles? (choose one or more) - Chloroplasts. - Nucleus. - Mitochondria. - Endoplasmic reticulum.

Chloroplasts. Mitochondria. Endoplasmic reticulum.

The citric acid cycle converts the carbon atoms in acetyl CoA to which of the following? - Oxaloacetate. - Glucose. - Citrate. - Pyruvate. - CO2.

CO2

Each molecule of acetyl-CoA entering the citric acid cycle produces two ___________ and four ___________. - Activated carriers; H2O. - ATP; GTP. - CO2; activated carriers. - NADH; ATP.

CO2; activated carriers.

More than 500 human genetic diseases are caused by haploinsufficiency, a heterozygotic condition in which the wild-type allele cannot produce enough normal protein to make up for the loss of functional protein expression from the mutated allele to get a normal phenotype. What might be an approach for treating haploinsufficiency disorders in non-human disease models? - Cre recombinase conditional knockout of the mutated gene. - CRISPR targeting wild-type gene with inactive Cas9 fused to a transcription activator. - Polymerase chain reaction of the wild-type allele DNA sequence to amplify the gene. - siRNA targeted to the mutated gene sequence to reduce mutated mRNA levels.

CRISPR targeting wild-type gene with inactive Cas9 fused to a transcription activator.

When an action potential reaches a nerve terminal, what type of voltage-gated channels are opened and result in the fusion of synaptic vesicles with the cell membrane? - Cl- - Ca2+ - K+ - Na+

Ca2+

A sodium-potassium antiport maintains the extracellular concentration of sodium at levels that are about 20-30 times higher than inside the cells. What directly supplies the energy for maintaining this gradient? - A proton gradient in the mitochondria drives the antiport. - Sodium supplies the energy, as it is moving along its concentration gradient. - ATP hydrolysis drives the function of the pump. - Potassium supplies the energy, as it is moving along its concentration gradient.

ATP hydrolysis drives the function of the pump.

Which of the following ions has a low cytosolic concentration so that a flood of this ion into the cell can be used as a signal for cell processes like fertilization? - Phosphate. - H+ - K+ - Ca2+

Ca2+

Rigor mortis occurs in muscle cells after death and is characterized by muscles being locked in a contracted state where myosin is bound to actin. What could be added to the muscle cell to release the rigor state? - Pi. - ATP. - ADP + Pi. - ADP.

ATP.

The translation of an mRNA begins at the start codon. What is the sequence of this codon? - AUG. - AGU. - UAG. - UGG.

AUG.

Fatty acids can be used to produce energy by conversion to ___________ in the ___________ of the cell. - Lipids; plasma membrane. - NADH; cytosol. - Acetyl CoA; mitochondria. - Pyruvate; endoplasmic reticulum.

Acetyl CoA; mitochondria.

When does homologous recombination most likely occur in order to flawlessly repair double-stranded DNA breaks? - Before DNA is replicated, to avoid propagating the error. - After the cell's DNA has been replicated. - After the broken ends have been "cleaned" by a nuclease. - When the damage is overlooked by the fast-acting mismatch repair system. - Whenever a double-strand break is detected.

After the cell's DNA has been replicated.

Match the following structures used in receptor-mediated endocytosis with their functions.

Cargo: Molecules packaged into vesicle for transport. Receptor: Captures the correct cargo molecules. Adaptin: Mediates contact between the receptor and another component. Clathrin: Shapes the forming vesicle.

Based on your understanding of DNA replication, which statement best describes the daughter helices in a eukaryotic cell after one round of replication has taken place in the absence of telomerase? - Both daughter helices will have gaps at both ends. - One daughter helix will have a gap at one end, while the other daughter helix will be normal. - A repair polymerase will replace all missing DNA, so both daughter helices will be the same size as the parent helix. - One daughter helix will have gaps at both ends, while the other daughter helix will be normal. - Both daughter helices will have a gap at only one end.

Both daughter helices will have a gap at only one end.

Taxol and colchicine are used to treat cancer because they arrest dividing cells in mitosis. What is the mechanism of action of these drugs? - Both drugs stabilize microtubules. - Both drugs destabilize microtubules. - Both drugs constrain the dynamic instability of microtubules. - Taxol prevents polymerization of microtubules, colchicine prevents disassembly of microtubules.

Both drugs constrain the dynamic instability of microtubules.

In a DNA double helix, which strand serves as the template during replication? - The strand that runs in the 3'-to-5' direction. - The strand that encodes a protein. - The strand that runs in the 5'-to-3' direction. - Both strands serve as templates during replication. - The strand that is richer in A and T nucleotides.

Both strands serve as templates during replication.

At which step of gene expression can cells amplify the number of copies of a protein made from a single gene? - Transcription. - Both transcription and translation. - Translation. - Neither transcription nor translation.

Both transcription and translation.

In a globular protein, where would the amino acid tryptophan most likely be found? - Exposed at the protein's surface. - Located at the protein's C-terminus. - Hydrogen-bonded to cysteine. - Located at the protein's N-terminus. - Buried in the protein's interior.

Buried in the protein's interior.

Which is true about electrons as they move through the electron-transport chain? - Electrons start out at very high energy and lose energy at each transfer step along the electron-transport chain. - Electrons can gain or lose energy, depending on where they enter along the electron-transport chain. - Electrons start out at very low energy and gain energy at each transfer step. - Electrons are pumped across the mitochondrial membrane as they move along the electron-transport chain. - Electrons neither gain nor lose energy as they move along the electron-transport chain.

Electrons start out at very high energy and lose energy at each transfer step along the electron-transport chain.

Which of the following animal cell types, under appropriate conditions, can be kept proliferating indefinitely in culture and yet retain unrestricted developmental potential? - Proliferating precursor cells. - Somatic cells. - Germ cells. - Embryonic stem cells.

Embryonic stem cells.

What is the name of the process by which eukaryotic cells engulf material captured from an external medium? - Exocytosis. - Cytokinesis. - Endocytosis. - Endomitosis. - Endosymbiosis.

Endocytosis.

Why do cells use enzymes to harvest energy from food molecules rather than by direct oxidation? - Enzymes release all the energy from food molecules in a single efficient step. - Enzymes transfer energy from food to carrier molecules in small steps. - Direct oxidation is energetically unfavorable. - Direct oxidation cannot release all the energy from food molecules.

Enzymes transfer energy from food to carrier molecules in small steps.

Treatment with methotrexate blocks the folate pathway and causes defects during embryogenesis in vertebrates. In humans, a lack of folate during pregnancy can lead to development of neural tube defects. In zebrafish development, methotrexate blocks embryo development at the stage of early somite formation. Which of the following events would also be blocked by methotrexate treatment of zebrafish embryos if development stops early in the stage of somite formation? (choose one or more). - Eye development. - Epiboly. - Movement of cells of external layer into the interior to eventually form the gut cavity. - Internalization of yolk into the developing gut cavity.

Eye development. Internalization of yolk into the developing gut cavity.

During oxidative phosphorylation, why does a single molecule of NADH result in the production of more ATP molecules than a single molecule of FADH2? - NADH donates more electrons to the electron-transport chain than does FADH2. - FADH2 promotes the pumping of more protons than does NADH. - FADH2 and NADH feed their electrons to different carriers in the electron-transport chain. - FADH2 is less likely than NADH to participate in the electron-transport chain. - FADH2 has a lower electron affinity than does NADH.

FADH2 and NADH feed their electrons to different carriers in the electron-transport chain.

Determine whether the following statement is true or false: A tissue is made of a single type of cell bound together by extracellular matrix. This statement is:

False.

Determine whether the following statement is true or false: If the Rb protein binds to transcription regulators, the cell will move from G1phase to S phase. This statement is:

False.

Determine whether the following statement is true or false: In the laboratory, transcription regulators can be used to convert one differentiated cell type into another cell type, but not to convert differentiated cells into a less differentiated, pluripotent stem cells. This statement is:

False.

Determine whether the following statement is true or false: Most ion channels undergo conformational changes with each ion that passes through. This statement is:

False.

Phosphorylation is the only form of covalent modification that can affect a protein's activity or location. - True. - False. - Impossible to determine.

False.

After an overnight fast, most of the acetyl CoA entering the citric acid cycle is derived from what type of molecule? - Fatty acids. - Glycogen. - Amino acids. - Glucose. - Pyruvate.

Fatty acids.

Under anaerobic conditions, which metabolic pathway regenerates the supply of NAD+ needed for glycolysis? - Citric acid cycle. - Formation of acetyl CoA. - Fermentation. - Breakdown of fats. - Breakdown of amino acids.

Fermentation.

Which of the following epithelial cell junctions is not involved in providing mechanical strength to an epithelial sheet by connecting the cytoskeletons of adjacent cells? - Adherens junctions. - Desmosomes. - Hemidesmosomes. - Gap junctions.

Gap junctions.

Useful energy is obtained by cells when sugars derived from food are broken down by which processes? - Glycolysis, the citric acid cycle, and gluconeogenesis. - Glycolysis, the citric acid cycle, and oxidative phosphorylation. - Glycolysis, the Calvin cycle, and oxidative phosphorylation. - Gluconeogenesis, fermentation, and oxidative phosphorylation. - Gluconeogenesis, the citric acid cycle, and oxidative phosphorylation.

Glycolysis, the citric acid cycle, and oxidative phosphorylation.

Which of the following is considered a reporter protein? - Green fluorescent protein. - DNA ligase. - Histones. - Ribosomal proteins. - RNA polymerase.

Green fluorescent protein.

Weight loss can occur when glucose is oxidized to CO2 rather than being stored as glycogen. The first step in glucose oxidation is glycolysis. A 1930s diet drug, DNP, made the inner mitochondrial membrane permeable to protons, increasing the rate of glycolysis. What is the explanation for the DNP-induced increase in glycolysis? - High ATP activates phosphofructokinase. - DNP also makes the inner mitochondrial membrane leaky to glucose, increasing the ability of the mitochondria to perform glycolysis. - High AMP inhibits phosphofructokinase. - High ADP activates phosphofructokinase.

High ADP activates phosphofructokinase.

What condition must exist for glucose to be transported into a cell using the glucose-Na+ symport? - High Na+ concentration inside the cell. - High glucose concentration outside the cell. - High Na+ concentration outside the cell. - High ATP concentration inside the cell for phosphorylation of the glucose-Na+ symport.

High Na+ concentration outside the cell.

Which of the following statements is true? - Each extracellular signal molecule interacts with a single class of cell-surface receptor. - G-protein-coupled receptors are GTP-binding proteins. - Ion-channel-coupled receptors can rapidly alter the membrane potential in response to signal binding. - All enzyme-coupled receptors have an intrinsic catalytic domain on the cytosolic side of the plasma membrane. - Many ion-channel-coupled receptors have an intrinsic catalytic domain on the cytosolic side of the plasma membrane.

Ion-channel-coupled receptors can rapidly alter the membrane potential in response to signal binding.

What type of enzyme catalyzes the rearrangement of chemical bonds within a single molecule? - Isomerase. - Scramblase. - Dehydrogenase. - Kinase.

Isomerase.

What does the phosphorylation of Cdc25 by M-Cdk do? - It activates Cdc25, allowing the cell to exit mitosis. - It activates Cdc25, which inactivates M-Cdk. - It inactivates Cdc25, which promotes activation of more M-Cdk. - It inactivates Cdc25, preventing further activation of M-Cdk. - It activates Cdc25, which in turn activates more M-Cdk.

It activates Cdc25, which in turn activates more M-Cdk.

Which statements are true of receptor-mediated endocytosis? (choose one or more). - It allows hemoglobin to be taken up by immature red blood cells. - Internalized endocytic vesicles fuse with lysosomes, which can return empty receptors to the plasma membrane. - It allows cholesterol-carrying low-density lipoproteins (LDLs) to be taken up by cells. - It allows the internalization of extracellular substances in clathrin-coated vesicles. - The process can be hijacked by viruses to gain entry into cells.

It allows cholesterol-carrying low-density lipoproteins (LDLs) to be taken up by cells. It allows the internalization of extracellular substances in clathrin-coated vesicles. The process can be hijacked by viruses to gain entry into cells.

For voltage-gated channels, a change in the membrane potential has what effect on the channel? - It makes the channel more sensitive to the binding of neurotransmitters. - It alters the probability that the channel will be found in its open conformation. - It changes the width of the channel opening. - It changes which ions can pass through the channel. - It either opens the channel or closes it, depending on the voltage.

It alters the probability that the channel will be found in its open conformation.

When Na+ channels are opened in an animal cell, what happens to the membrane potential? - It stays the same. - It becomes more negative inside the cell. - It becomes less negative inside the cell. - It rapidly reaches the resting membrane potential. - It disappears, and membrane potential stabilizes at 0 mV.

It becomes less negative inside the cell.

Compared to plant cell walls, what is true of the extracellular matrix of most animal tissues? - It contains more nitrogen. - It is generally thicker and stronger. - It contains more carbon. - It contains cellulose that is produced inside the cell in a precursor form, rather than synthesized extracellularly.

It contains more nitrogen.

What is true of the process of combining CO2 and H2O to make a carbohydrate? - It is energetically favorable. - It takes place in all eukaryotic cells. - It is energetically unfavorable. - It occurs only during daylight hours. - It produces ATP.

It is energetically unfavorable.

What determines the nucleotide sequence of a newly transcribed RNA molecule? - Complementary base-pairing with a DNA during transcription. - The amino acid sequence in a protein. - The ribonucleotides available. - The RNA polymerase enzymes.

Complementary base-pairing with a DNA during transcription.

What determines the direction that glucose is transported across the membrane, through a glucose transporter? - Membrane potential. - Concentration gradient. - A molecule's size. - A molecule's charge.

Concentration gradient.

The shortening and thickening of chromosomes in M phase depends on which of the following? - Microtubules. - Cohesins. - Actin and myosin. - Condensins.

Condensins.

ATP synthase is a large molecular machine that converts the energy in an electrochemical gradient into the bond energy stored in ATP. Which of the following events are required for the synthesis of ATP? (choose one or more). - Conformational changes of the F1 ATPase. - Movement of protons down their gradient through ATP synthase. - Rotation of the rotor in the membrane. - Binding of ATP to an empty F1 ATPase subunit.

Conformational changes of the F1 ATPase. Movement of protons down their gradient through ATP synthase. Rotation of the rotor in the membrane.

Fibroblasts are the major cellular component in which tissues? - Connective. - Epithelial. - Muscle. - Nervous.

Connective.

Which of the following animal tissues is characterized by a plentiful extracellular matrix? - Nervous. - Muscular. - Connective. - Epithelial.

Connective.

Which does not affect gene expression at the post-transcriptional level? - microRNAs. - DNA methylation. - Small interfering RNAs. - Nucleotide sequences in the untranslated regions of an mRNA.

DNA methylation.

Protein concentration can be regulated by all of the steps listed EXCEPT: - DNA replication. - mRNA stability. - RNA processing. - Nuclear export.

DNA replication.

Which of the following processes occurs only in S phase of the cell cycle? - Organelle replication. - Cleavage of the cytoplasm. - DNA replication. - Cell growth. - Chromosome segregation.

DNA replication.

Alterations in signaling in the pituitary gland can lead to human disease. The GH-releasing hormone (GHRH) stimulates release of growth hormone (GH) from the pituitary gland by binding to GHRH receptors, which are G-protein-coupled receptors. Excessive activity of the GHRH signaling pathway leads to excessive release of growth hormone, which can lead to acromegaly, a form of gigantism. Some patients can reach more than 8 feet tall as they continue to grow even in adulthood. Consider steps that could be taken to reduce GH release. Sort each of the following interventions into the proper category.

Decreases GH Release: Inhibit interaction of Ga with receptor. Block ligand binding to receptor. Activate phosphorylation of the receptor by GRK kinase. Increases GH Release: Block binding of arrestin to the receptor. Block hydrolysis of GTP.

The hydrolysis of bound ATP to ADP in an actin filament _______ the strength of binding between monomers in the filament.

Decreases.

At the end of M phase, cells shut down Cdk activity through which of the following mechanisms? - Deployment of Cdk inhibitors. - Mitogen-activated signaling. - Shutting down all gene expression. - Degrading the Rb protein.

Deployment of Cdk inhibitors.

Motor neuron degeneration occurs in several diseases and leads to loss of muscle control. One form of motor neuron degeneration was found to have defects in retrograde transport (backward transport to cell body, in blue below) that were caused by mutations in a gene that codes for a particular protein. What protein, when mutated, would inhibit backward, but not outward, transport along axon microtubules? - Tubulin. - Kinesin. - Dynein. - Myosin.

Dynein.

Match each of the following functions with the protein that completes that function.

Proteasome: Degrades misfolded proteins. Ribosome: Synthesizes proteins. Chaperone: Helps proteins properly fold. elF2: Controls rate of translation initiation.

The maltose operon contains genes that code for proteins that catabolize the disaccharide maltose. Similar to the Lac operon, which is only efficiently transcribed in the presence of lactose, the maltose operon is only efficiently transcribed in the presence of maltose. How might induction of the maltose operon in response to maltose be achieved? (choose one or more). Maltose causes an activator to bind an operon with an inefficient promoter. - Maltose causes an activator to bind an operon with an efficient promoter. - Maltose removes an activator from an operon with an inefficient promoter. - Maltose removes a repressor from an operon with an efficient promoter. - Maltose causes a repressor to bind an operon with an efficient promoter.

Maltose causes an activator to bind an operon with an inefficient promoter. Maltose removes a repressor from an operon with an efficient promoter.

Which of the following statements is true? - Many cells die by apoptosis in many normal, adult human tissues. - Many cells die by necrosis in many normal, adult human tissues. - Many cells die by necrosis during normal animal development.

Many cells die by apoptosis in many normal, adult human tissues.

Which of the following is a cell type in humans that uses voltage-gated ion channels? - Skin cells. - Liver cells. - Gut epithelial cells. - Muscle cells.

Muscle cells.

What type of enzyme catalyzes the shifting of a chemical group from one position in a molecule to another? - Mucinase. - Dehydrogenase. - Kinase. - Flippase. - Mutase.

Mutase.

Although the citric acid cycle itself does not use O2, it requires a functioning electron transport chain (which uses O2) in order to regenerate which molecule for further use in the citric acid cycle? - FADH2. - NAD+. - ATP. - ADP. - NADH.

NAD+

Which of the following types of intermediate filaments are found in all animal cells? - Vimentin and vimentin-related filaments. - Nuclear lamins. - Neurofilaments. - Keratin filaments.

Nuclear lamins.

Which type of receptors do steroid hormones generally employ? - Ion-channel-coupled receptors. - Enzyme-coupled receptors. - Cell-surface receptors. - Nuclear receptors. - G-protein-coupled receptors.

Nuclear receptors.

In the living cell, histone proteins pack DNA into a repeating array of DNA-protein particles called what? - Heterochromatin. - Nucleosomes. - Octamers. - Euchromatin. - Beads on a string.

Nucleosomes.

When a transcription regulator binds to DNA, its most important interactions occur with which of the following? - Phosphate groups in the DNA backbone. - Nucleotide pairs in the major groove of the double helix. - The 5' and 3' ends of the same DNA strand. - Sugar groups in the DNA backbone.

Nucleotide pairs in the major groove of the double helix.

In eukaryotic cells, what is the final electron acceptor in the electron transport chain? - CO2. - NAD+. - O2. - FADH2. - ATP.

O2

Which molecules are required for the citric acid cycle to fully oxidize the carbons donated by acetyl CoA? (choose one or more). - O2. - GTP. - ATP. - NAD+. - GDP. - Oxaloacetate.

O2. NAD+. GDP. Oxaloacetate.

Fibroblasts play a role in doing which of the following? - Destructing collagen. - Synthesizing bone. - Triggering apoptosis. - Orienting cellulose microfibrils. - Orienting collagen.

Orienting collagen.

What is true of the inside of a cell? - It is slightly more positive than the outside of a cell. - It is slightly more negative than the outside of a cell. - It has the same charge as the outside of the cell.

It is slightly more negative than the outside of a cell.

Which statement describes the extracellular matrix of plant tissues? - It contains collagen. - It is produced in precursor form in the plant cell cytosol. - It has virtually no tensile strength. - It is filled with compression-resistant proteoglycans. - It is the cell wall.

It is the cell wall.

To pass through the pore of an ion channel, what must be true of an ion? - It must avoid contact with the channel wall. - It must be positively charged. - It must interact with polar groups in the narrowest part of the channel. - It must surround itself with water molecules. - It must interact with non-polar groups in the selectivity filter.

It must interact with polar groups in the narrowest part of the channel.

What is true of the phosphorylation of glucose in step 1 of glycolysis? - It is energetically unfavorable. - It has a ΔG° of 8 kcal/mole. - It traps the sugar inside the cell. - It produces NADH. - It is carried out by an enzyme called phosphoglucokinase.

It traps the sugar inside the cell.

Which statement is true of the ER signal sequence on soluble proteins? - It will be cleaved from the polypeptide but remain embedded in the ER membrane. - It will function to open the protein translocator. - When it is located in the middle of the polypeptide, it will remain part of the mature protein. - It will be cleaved from the polypeptide before translocation begins. - When it is found at the C-terminus of the protein, it will remain in the cytosol.

It will function to open the protein translocator.

In most animal cells, which ion can move through "leak" channels? - Cl- - K+ - Na+ - H+ - Ca2+

K+

Which ion is generally maintained at a high concentration inside the cell and a low concentration outside the cell? - H+ - Cl− - K+ - Na+

K+

What kind of enzyme adds a phosphate group to another protein? - ATPase. - Kinase. - Phosphatase. - GTPase. - Phosphorylase.

Kinase.

As a cell grows, which microtubule-associated protein pulls the ER membrane outward, stretching it like a net? - Kinesin. - Dynein. - Cohesin. - Actin.

Kinesin.

Which of the following terms describes a mouse in which a particular gene has been genetically eliminated? - Knockout mouse. - Hybrid mouse. - Transfected mouse. - Nonsense mouse.

Knockout mouse.

Ethylene is a hormone that promotes the ripening of fruit. What happens in the absence of ethylene? - The ethylene receptor is inactive and the transcription regulator needed to turn on the ethylene-responsive genes is degraded. - The activated ethylene receptor promotes the degradation of a transcription regulator and the ethylene-responsive genes remain turned off. - The ethylene receptor is inactive and a transcription regulator shuts off the ethylene-responsive genes. - The ethylene receptor is internalized and degraded, shutting down the expression of the ethylene-responsive genes. - The activated ethylene receptor switches on a transcription regulator that inhibits the expression of ethylene-responsive genes.

The activated ethylene receptor promotes the degradation of a transcription regulator and the ethylene-responsive genes remain turned off.

Which of the following statements is false? - The activation of S-Cdk initiates DNA replication. - The activation of S-Cdk helps prevent the onset of M phase. - The activation of S-Cdk helps prevent DNA replication occurring more than once each cycle. - DNA replication occurs only in S phase.

The activation of S-Cdk helps prevent the onset of M phase.

What can happen if heterochromatin spreads inappropriately into an area with active genes? - The genes will resist and remain active. - The active genes can become silenced. - The region will convert into a telomere. - Heterochromatin cannot spread into areas with active genes.

The active genes can become silenced.

What recognizes the stop codons in an mRNA? - Nothing. - Release factor. - The 3' poly-A tail. - A specialized tRNA linked to methionine.

Release factor.

Why does cytochrome c oxidase bind to oxygen very tightly? - Release of partially reduced oxygen radicals can damage the cell. - The complex contains multiple iron-sulfur clusters. - Oxygen has a very low affinity for electrons. - Oxygen is too scarce. - Cyanide prevents oxygen from dissociating.

Release of partially reduced oxygen radicals can damage the cell.

The binding of ATP causes a conformational change in myosin that: - Releases the myosin head from the actin filament. - Attaches the myosin head to the actin filament. - Moves the myosin head one step forward on the actin filament. - Move the actin filament in a force-generating "power stroke."

Releases the myosin head from the actin filament.

An actin filament undergoing treadmilling at the leading edge of a lamellipodium can do what? - Add actin monomers to its minus end while losing them from its plus end. - Collapse and instantly disappear. - Remain the same size. - Experience exponential growth.

Remain the same size.

Put the following steps used to transport proteins into mitochondria into the proper order.

The mitochondria protein is synthesized in the cytosol. The receptor on the mitochondrial membrane binds the signal sequence on the protein. The protein is delivered to the translocation apparatus on the mitochondria. The protein is pass through the translocation apparatus. The signal sequence is removed by signal peptidase and the protein folds into its final conformation.

Which of the following is a key group of proteins that regulate the actin cytoskeleton? - Rho protein family. - Ran protein family. - Rab protein family. - Rheb protein family.

Rho protein family.

It has been proposed that the first cells used RNA for both information storage and catalysis and that DNA and proteins evolved later. Which modern macromolecules may be relics of the hypothesized RNA world? (choose one or more). - Ribosome. - Spliceosome. - RNA polymerase. - Sigma factor.

Ribosome. Spliceosome.

As the human population grows, it becomes increasingly important to maximize crop yields. As such, scientists search for more efficient ways for plants to convert CO2 into biomass. One approach is to genetically modify plant enzymes involved in photosynthesis to increase their efficiency. Which plant enzyme, directly responsible for carbon fixation, is a focus of research? - Chlorophyll - Cytochrome c oxidase. - Rubisco. - ATP synthase.

Rubisco.

In which phase of the cell cycle does duplication of the centrosome begin? - M phase. - S phase. - G2 phase. - Prophase. - G1 phase.

S phase.

The FRAP technique occurs in a series of steps. Select every statement that correctly describes a step in the FRAP procedure. (choose one or more). - The speed of repair of the fluorescent marker is measured. - The molecule of interest is fluorescently labeled. - All fluorescent molecules in the cell are irreversibly bleached. - The relative mobility of the fluorescently labeled molecule is measured.

The molecule of interest is fluorescently labeled. The relative mobility of the fluorescently labeled molecule is measured.

What type of protein moves randomly selected phospholipids from one monolayer of a lipid bilayer to the other? - Flippase. - None; such movement occurs spontaneously and relatively quickly. - Scramblase. - None; phospholipids cannot move from one monolayer to another. - Phospholipase.

Scramblase.

Which of the following statements is not true regarding active transport by transmembrane pumps? - Some solutes are moved against their concentration gradients, from one side of a membrane to the other, using energy from ATP hydrolysis. - Some solutes are transported across the membrane in tandem with other molecules, both moving from lower concentration to higher concentration. - Some solutes are moved across a membrane against their concentration gradient using energy from sunlight.

Some solutes are transported across the membrane in tandem with other molecules, both moving from lower concentration to higher concentration.

In the human genome, mutations that change a single nucleotide in a gene _______ have deleterious effects on the protein encoded by that gene.

Sometimes

Suppose the 3' splice site is mutated from AGG to ACG. Predict the consequence of this mutation. - Splicing would not begin if the 3' splice site is altered. - Splicing would occur as normal but there would be a point mutation in the mRNA and final protein sequences. - Splicing would occur as normal because this splice site is less important than the 5' splice site. - Splicing would begin but not be properly completed if the 3' splice site is altered.

Splicing would begin but not be properly completed if the 3' splice site is altered.

When nutrients are plentiful, plants can store glucose as what? - Glycogen and starch. - Starch. - Glucose 6-phosphate. - Fats. - Glycogen.

Starch.

Taxol, a plant alkaloid originally found in yew trees, is an anticancer drug. How does Taxol prevent cell division? - Taxol stabilizes intermediate filaments of the nuclear lamina, preventing nucleus breakdown during mitosis. - Taxol prevents actin filament depolymerization, halting mitosis. - Taxol prevents actin filament polymerization, halting mitosis. - Taxol prevents microtubule depolymerization, halting mitosis. - Taxol prevents microtubule polymerization, halting DNA replication.

Taxol prevents microtubule depolymerization, halting mitosis.

At which site on the DNA of a gene does RNA polymerase release its newly made RNA? - Stop codon. - Poly-A tail. - Terminator. - Promoter. - TATA box.

Terminator.

Which of the following occurs when a cell repairs a double-strand DNA break by the process of nonhomologous end joining? - The DNA sequence at the site of repair is altered by a short addition. - The DNA sequence at the site of repair is altered by a short deletion. - The original DNA sequence at the site of repair is reconstituted with 100% accuracy. - The DNA sequence at the site of repair matches that of a homologous chromosome. - The DNA sequence at the site of repair contains a short segment of telomere DNA.

The DNA sequence at the site of repair is altered by a short deletion.

The assembly of general transcription factors at a eukaryotic promoter typically begins at what site? - The TFIID sequence. - The TATA box. - The sigma site. - The start codon.

The TATA box.

Which feature of glycosaminoglycans (GAGs) allows the extracellular matrix in cartilage to resist compression? - GAGs are typically attached to proteins to form proteoglycans, and both the protein and GAG chains resist compression. - The negative charges on GAGs attract sodium ions, which draw water into the extracellular matrix to form a water-filled gel. - The negative charges on GAGs attract calcium phosphate, which forms a hard, resilient mesh that resists compression. - GAGs form stiff rods in the extracellular matrix and thereby resist compression.

The negative charges on GAGs attract sodium ions, which draw water into the extracellular matrix to form a water-filled gel.

In bacteria, how does the cell recognize which strand is the newly synthesized strand and thus contains the mismatch? - The sequence on the newly synthesized strand is made of RNA. - The newly synthesized strand is methylated. - The sequences on both strands are cut. - The newly synthesized strand is unmethylated.

The newly synthesized strand is unmethylated.

During an action potential, which of the following actions does not help return the membrane to its resting potential? - The inactivation of voltage-gated Na+ channels. - The opening of voltage-gated Na+ channels. - The flow of K+ through K+ leak channels. - The opening of voltage-gated K+ channels.

The opening of voltage-gated Na+ channels

In the α helices of transmembrane proteins, the hydrophobic side chains face which direction? - The external or lumenal side of the membrane. - The cytosolic side of the membrane. - The outside of the membrane-spanning helix. - The inside of the membrane-spanning helix.

The outside of the membrane-spanning helix.

In the absence of oxygen, in cells that cannot carry out fermentation, glycolysis would halt at which step? - The oxidation of glyceraldehyde 3-phosphate to 1,3-bisphosphoglycerate. - The transfer of a phosphate group from 1,3-bisphosphoglycerate to ADP to form ATP. - The phosphorylation of fructose 6-phosphate to fructose 1,6-bisphosphate. - The reversible rearrangement of glucose 6-phosphate to fructose 6-phosphate.

The oxidation of glyceraldehyde 3-phosphate to 1,3-bisphosphoglycerate.

How do enzymes maximize the energy harvested from the oxidation of food molecules? - They allow what would otherwise be an energetically unfavorable oxidation reaction to occur. - They guarantee that each reaction involved in the oxidation of food molecules proceeds in just one direction. - They allow a larger amount of energy to be released from food molecules such as glucose. - They allow the stepwise oxidation of food molecules, which releases energy in small amounts. - They allow oxidation reactions to take place without an input of activation energy.

They allow the stepwise oxidation of food molecules, which releases energy in small amounts.

What is true of the GTP-binding proteins that act as molecular switches inside cells? - They are active only in their trimeric forms. - They are active when GDP is bound. - They turn themselves on by phosphorylating GDP to form GTP. - They are active when GTP is bound. - They turn themselves on by hydrolyzing GTP to form GDP.

They are active when GTP is bound.

What is typically true of ion channels? - They are nonselective. - They are gated. - They are open all the time. - They operate by active transport. - They hydrolyze ATP.

They are gated.

What is true of bacterial mRNAs? - They are transcribed and translated simultaneously. - They are processed in the same way as eukaryotic mRNAs. - They are translated after they are exported from the nucleus. - They must be folded into the correct three-dimensional shape before they can be translated. - They contain extensive noncoding sequences.

They are transcribed and translated simultaneously.

What happens to the final products of the digestion of macromolecules? - They are ultimately destroyed by lysosomal enzymes. - They are expelled from the lysosome by H+ pumps in the lysosomal membrane. - They are secreted from the cell when lysosomes fuse with the plasma membrane. - They are transferred to the cytosol through transporters in the lysosomal membrane. - They are removed from the lysosome by transport vesicles that carry them to where they are needed.

They are transferred to the cytosol through transporters in the lysosomal membrane.

What is true of eukaryotic mRNAs? - They are transcribed and translated simultaneously. - They are translated by enzymes that ride on the phosphorylated tail of RNA polymerase II. - They are translated after they are exported from the nucleus. - They must always be folded into a complex three-dimensional shape before they can be translated. - They are processed only after being released by the polymerase.

They are translated after they are exported from the nucleus.

In a cytosolic folded protein, what orientation and/or interaction do the hydrophobic amino acids tend to have? (choose one or more). - They are clustered near the N-terminus of the protein. - They are tucked away inside the protein. - They are exposed on the outside of the protein. - They are interacting with other non-polar amino acids.

They are tucked away inside the protein. They are interacting with other non-polar amino acids.

What generally is the fate of mutations to the genome that have harmful consequences to an organism? - They are usually eliminated from the population by natural selection. - They tend to persist and spread. - They do not occur. - They give rise to new species. - They are always fixed by DNA repair.

They are usually eliminated from the population by natural selection.

Which is true of induced pluripotent stem (iPS) cells? - They behave like embryonic stem cells. - They can be produced artificially using a single transcription regulator. - They can be used to produce whole organs for transplantation. - They are produced by nuclear transplantation. - They can be produced by incubating adult fibroblasts with Ca2+.

They behave like embryonic stem cells.

What is the function of single-strand binding proteins in DNA replication? - They bind to single-stranded DNA and scan for replication errors. - They release completed Okazaki fragments. - They break a DNA double helix to form two separate single strands. - They strip away used RNA primers. - They bind to regions of DNA, preventing them from forming base pairs together.

They bind to regions of DNA, preventing them from forming base pairs together.

Proteins entering the cis Golgi network can do which of the following? - They can be held in the cis Golgi while they are properly folded and assembled. - They can either move backward to the trans Golgi network or be sent forward to the plasma membrane. - They are either sent to the nucleus or to the plasma membrane for secretion. - They are sorted according to whether they are destined for lysosomes or for the cell surface. - They can either move onward through the Golgi stack or be returned to the ER.

They can either move onward through the Golgi stack or be returned to the ER.

Which is true of prion proteins? (choose one or more). - They can spread from one organism to another. - They cannot spread from cell to cell. - They are mis-folded forms of normal proteins. - They form unstable protein aggregates.

They can spread from one organism to another. They are mis-folded forms of normal proteins.

What happens to the miRNAs that are bound in a RISC? - They remain bound to the RISC, where they can target the elimination of multiple mRNAs. - Their translation is blocked. - If they include a sufficiently extensive region of complementarity with an mRNA, they are diverted to a region of the cytosol where they are eventually degraded. - They are destroyed as soon as they bind to an mRNA.

They remain bound to the RISC, where they can target the elimination of multiple mRNAs.

What do the phosphorylated tyrosines on activated RTKs do? - They activate the enzymatic activity of the RTKs. - They serve as binding sites for a variety of intracellular signaling proteins. - They serve as binding sites for G proteins. - They promote receptor internalization. - They help the receptor dimerize.

They serve as binding sites for a variety of intracellular signaling proteins.

Several organisms have a homologous protein (inherited from a common ancestor) that is highly similar at the amino acid level. You are comparing the genes that code for these proteins in the different organisms when you note that one of the codon nucleotide positions shows more nucleotide variation than the other nucleotide positions. In which codon nucleotide position do you expect to see the most variability among species? - Fourth nucleotide position. - First nucleotide position. - Second nucleotide position. - Third nucleotide position.

Third nucleotide position.

Which structure or process mediates the exchange of materials between the endoplasmic reticulum, Golgi apparatus, the lysosomes, and the outside of the cell? - Transport vesicles. - Simple diffusion. - Cytosol. - Peroxisomes. - Extracellular matrix.

Transport vesicles.

Plasma membrane proteins that move ions in and out of cells using active transport are called: - Receptors. - Anchors. - Channels. - Transporters.

Transporters.

When a muscle is stimulated to contract, what does Ca2+ bind to, and what effect does that have? - Actin, allowing it to associate with myosin. - Tropomyosin, which moves the troponin that otherwise blocks the interaction of actin and myosin. - Myosin, allowing it to associate with actin. - Troponin, which moves the tropomyosin that otherwise blocks the interaction of actin and myosin.

Troponin, which moves the tropomyosin that otherwise blocks the interaction of actin and myosin.

Determine whether the following statement is true or false: A cell can change the expression of its genes in response to external signals. This statement is:

True.

Determine whether the following statement is true or false: A single transcription regulator, if expressed in the appropriate precursor cell, can trigger the formation of a specialized cell type or even an entire organ. This statement is:

True.

Determine whether the following statement is true or false: Apoptosis depends on a tightly regulated proteolytic cascade. This statement is:

True.

Determine whether the following statement is true or false: Cancer cells fail to obey normal "social" controls on cell behavior and therefore outgrow, out-divide, and out-survive their normal neighbors. This statement is:

True.

Determine whether the following statement is true or false: Cells joined together in epithelial sheets line all external and internal surfaces of the animal body. This statement is:

True.

Determine whether the following statement is true or false: Communication between neurons involves an interconversion of electrical and chemical signals. This statement is:

True.

Determine whether the following statement is true or false: Patch-clamp recording can be used to observe the transport of ions through individual ion channels. This statement is:

True.

Determine whether the following statement is true or false: Some genes do not encode proteins but instead encode functional RNA. This statement is:

True.

Determine whether the following statement is true or false: The glucose-Na+ symporter in epithelial cells uses the electrochemical gradient of Na+ to draw glucose into the cell. This statement is:

True.

Determine whether the following statement is true or false: The nuclear lamina disassembles and re-forms at each cell division. This statement is:

True.

Determine whether the following statement is true or false: The rate of cell turnover varies in different tissue types. This statement is:

True.

Determine whether the following statement is true or false: The subunits of microtubules and actin filaments are globular proteins, whereas those of intermediate filaments are fibrous. This statement is:

True.

The binding of a regulatory ligand can change the equilibrium between two protein conformations. - True. - False - Impossible to determine.

True.

The mismatch repair system recognizes mismatched base pairs, removes a portion of the DNA strand containing the error, and then resynthesizes the missing DNA using the correct sequence as a template. But what if the mismatch repair system instead removed a piece of the DNA strand that contained the correct sequence? What would replication of this improperly repaired sequence produce? - Two DNA molecules that are missing one nucleotide pair. - Two DNA molecules with different mutations. - Two DNA molecules with a gap where the correct sequence was excised. - One DNA molecule with a mutation and one DNA molecule with a mismatch. - Two DNA molecules bearing the same mutation.

Two DNA molecules bearing the same mutation.

What is the "central dogma"? - Much like all viruses, all cells use DNA to encode genetic information. - The earliest cells on Earth most likely used RNA to store and copy genetic information. - Within the cell, genetic information flows from DNA to RNA to protein. - For a given individual, the genetic information contained in the DNA of every cell is identical. - The ability of a cell to survive depends on the accurate duplication of genetic information carried in its DNA.

Within the cell, genetic information flows from DNA to RNA to protein.

In a lipid bilayer, where do lipids rapidly diffuse? - In and out of the bilayer. - Within the plane of their own monolayer. - Within the plane of one monolayer and back and forth between the monolayers. - Back and forth from one monolayer to the other in the bilayer. - Not at all, because they remain in place within the bilayer.

Within the plane of their own monolayer.

Which of the following techniques allows a researcher to identify the location of particular RNAs within a living cell? - Electron microscopy. - Southern blotting. - in situ hybridization. - RNA-Seq. - Reporter gene expression.

in situ hybridization.

In response to DNA damage, the ___________ protein is phosphorylated and activates the transcription of a Cdk inhibitor to halt cell cycle progression. - p53. - p21. - p27. - Rb.

p53.

In a centrosome, which structures serve as nucleation sites for the formation of microtubules? - Tubulin protofilaments. - α- and β-tubulin monomers. - αβ-tubulin dimers. - γ-tubulin rings.

γ-tubulin rings.

Approximately how many molecules of ATP can be produced in mitochondria from the complete oxidation of a single glucose molecule? - 4. - 2. - 1000 to 2000. - It depends on the number of mitochondria present. - 30.

30.

Lysosomal enzymes are directed to lysosomes by which of the following? - A C-terminal hydrophobic membrane-spanning amino acid sequence. - Their ability to fold properly and function at low pHan N-terminal signal sequence. - Their ability to fold properly and function at high pH. - A phosphorylated sugar group.

A phosphorylated sugar group.

Which base pairs are found in DNA? - A-U and C-G. - A-G and C-T. - T-T and C-C. - A-T and G-C. - A-C and T-G.

A-T and G-C.

Which of the following are required for glycolysis to take place? (choose one or more). - ADP. - NAD+. - ATP. - Pi. - NADH. - O2.

ADP. NAD+. ATP. Pi.

Margarine is produced from vegetable oil by a process that does which of the following? - Freezes the phospholipids, which removes their double bonds. - Adds carbons, which increases the length of their phospholipid tails. - Removes hydrogen, which increases the number of double bonds in their phospholipid tails. - Removes carbons, which decreases the length of their phospholipid tails. - Adds hydrogen, which removes the double bonds from their phospholipid tails.

Adds hydrogen, which removes the double bonds from their phospholipid tails.

Which term correctly describes the entire phospholipid molecule? - Amphipathic. - Apathetic. - Hydrophilic. - Hydrophobic. - Hydropathic.

Amphipathic.

Inhibitory neurotransmitters such as glycine and GABA make a postsynaptic cell harder to depolarize by allowing what? - The escape of Na+. - An influx of Cl-. - An influx of Na+. - An influx of K+.

An influx of Cl-.

The consistent diameter of the DNA double helix arises because of which property? - Base pairing of pyrimidines with purines. - The use of deoxyribose sugar with a $$3′-OH. - Phosphodiester linkages having a consistent bond length. - The antiparallel nature of the two strands of DNA.

Base pairing of pyrimidines with purines.

How do protein, nucleic acid, and polysaccharide molecules polymerize (grow in length)? - By hydrolysis reactions. - By oxidation reactions. - By condensation reactions. - None of these.

By condensation reactions.

As a cell grows, which microtubule-associated protein pulls the Golgi apparatus inward toward the nucleus? - Dynein. - Actin. - Kinesin. - Cohesin.

Dynein.

Which of the following describes a breakdown process in which enzymes degrade complex molecules into simpler ones? - Anabolism. - Metabolism. - Catabolism. - Enzymes are not needed to break down complex molecules into simpler ones.

Catabolism.

Ras is activated by a Ras-activating protein that does what? - Causes Ras to hydrolyze its bound GTP. - Phosphorylates Ras. - Causes Ras to exchange GDP for GTP. - Dephosphorylates Ras. - Causes Ras to interact with a phosphorylated RTK.

Causes Ras to exchange GDP for GTP.

If DNA is damaged or incompletely replicated in S phase, the inhibition of which of these molecules prevents the cell from entering M phase? - Cdc6. - S-Cdk. - Cdc25. - ORC. - p53.

Cdc25.

Antibiotics tend to target features that are unique to bacterial cells and absent from eukaryotic cells such as our own. Which of the following would present a safe target for a new antibiotic? - Intermediate Filaments. - Endoplasmic Reticulum. - Plasma Membrane. - Nuclear Envelope. - Cell Wall.

Cell Wall.

Supportive tissues, such as bone or wood, derive their strength from which of the following? - Mitochondria. - Turgor pressure. - Cytoskeletal filaments. - Cell walls. - Extracellular matrix.

Extracellular matrix.

What gives the plant cell wall its tensile strength? - Starch. - Actin filaments. - Cellulose. - Myosin.

Cellulose.

Which of the following form tiny hydrophilic pores in the membrane through which solutes can pass by diffusion? - Liposomes. - Channels. - Transporters. - Anions. - Pumps.

Channels.

Which of the following can be a component of cell membranes? (choose one or more). - DNA. - Cholesterol. - Protein. - Lipid. - Sugar.

Cholesterol. Protein. Lipid. Sugar.

A stretch of amino acids in a polypeptide chain that is capable of independently folding into a defined structure is called a: - Backbone. - Domain. - Ligand. - Subunit.

Domain.

When are chromosomes in their most compacted form? - During replication. - During mitosis. - During interphase. - They are always equally compacted.

During mitosis.

Which of the following processes generates the largest number of ATP molecules? - Glycolysis. - Gluconeogenesis. - Electron transport chain. - Citric acid cycle. - Fermentation.

Electron transport chain.

Researchers often want to isolate a certain type of RNA. For some RNA species, this can be accomplished via affinity chromatography, using beads coated with chains of poly-deoxythymidine (poly-dT). The desired RNA will stick to the beads while unwanted RNAs will flow through the column. The retained RNA can then be eluted. What RNA species can be purified using this method? - Eukaryotic mRNA. - Bacterial rRNA. - Bacterial mRNA. - Eukaryotic rRNA.

Eukaryotic mRNA.

In desmosomes, cadherin molecules are anchored inside an epithelial cell to which type of cytoskeletal filament? - Microtubules. - Intermediate filaments. - Fibronectins. - Actin filaments.

Intermediate filaments.

The entire period between one M phase and the subsequent M phase is called what? - Anaphase. - Metaphase. - G1 phase. - Prophase. - Interphase.

Interphase.

Which of these human cells undergoes the most rapid turnover? - Skin epidermal cells. -Bone-forming cells. - Intestinal epithelial cells. - Nerve cells. - Red blood cells

Intestinal epithelial cells.

Which of the following statements are true of electron transfer in the electron-transport chain? (choose one or more). - NADH is a strong electron donor. - NADH has a relatively high electron affinity and initiates the electron-transport process. - When an electron carrier accepts an electron, it becomes oxidized. - Each electron transfer is an oxidation-reduction reaction. - Electrons move toward molecules with a high redox potential.

NADH is a strong electron donor. Each electron transfer is an oxidation-reduction reaction. Electrons move toward molecules with a high redox potential.

Most of the energy for the synthesis of ATP comes from which molecule? - GTP produced by the citric acid cycle. - NADH produced by the citric acid cycle. - FADH2 produced by the citric acid cycle. - NADH produced by glycolysis. - NADH produced by the conversion of pyruvate to acetyl CoA.

NADH produced by the citric acid cycle.

The electron-transport chain in mitochondria accepts high-energy electrons directly from which molecule? - NADH. - ATP. - Acetyl CoA. - Pyruvate. - H2O.

NADH.

What are the end products of glycolysis? (choose one or more). - NADH. - ADP. - H2O + CO2. - NADPH. - Pyruvate. - ATP. - Acetyl CoA.

NADH. Pyruvate. ATP.

Order these components involved in electron transport from those with the lowest redox potential to the highest.

NADH/NAD+ NADH dehydrogenase complex. Cytochrome c reductase. Cytochrome c oxidase. H2O/O2.

What would be the consequences to NADPH production if the redox potential of pC (plastocyanin) were altered to be more negative than the redox potential of cytochrome b6-f? - NADPH production would not be altered since NADPH is produced from photosystem I. - NADPH production would halt since pC would no longer accept electrons from the cytochrome b6-fcomplex, blocking the electron transport chain. - NADPH production would be higher since pC would be a better electron donor. - NADPH would be broken down since the electron transport chain would work backward.

NADPH production would halt since pC would no longer accept electrons from the cytochrome b6-fcomplex, blocking the electron transport chain.

Lipid bilayers are highly impermeable to which molecule(s)? - Carbon dioxide. - Water. - Na+ and Cl- - Oxygen. - Steroid hormones.

Na+ and Cl-

When a cell is acutely injured, it usually dies by which process? - Apoptosis. - Phagocytosis. - Endocytosis. - Necrosis.

Necrosis.

Which of the following statements is true? - New microtubules grow out in random directions from the centrosomes. - The dynamic instability of microtubules decreases at the start of mitosis. - The assembly of the spindle is driven, in part, by myosins associated with the interpolar microtubules, which help to cross-link the two sets of microtubules. - The mitotic spindle is made primarily of microtubules and actin filaments.

New microtubules grow out in random directions from the centrosomes.

Which type of bond makes it possible for a macromolecule to interact with great specificity with just one out of the many thousands of different molecules present inside a cell? - Covalent bonds. - Non-covalent bonds.

Non-covalent bonds.

Which double-strand break repair mechanism is a simple ligation mechanism? - Non-homologous end joining. - DNA ligase. - DNA mismatch repair. - Homologous recombination.

Non-homologous end joining.

Sodium ions, oxygen (O2), and glucose pass directly through lipid bilayers at dramatically different rates. Which of the following choices presents the correct order, from fastest to slowest? - Glucose, oxygen, sodium ions. - Oxygen, glucose, sodium ions. - Oxygen, sodium ions, glucose. - Sodium ions, oxygen, glucose. - Glucose, sodium ions, oxygen.

Oxygen, glucose, sodium ions.

At what site does the charged initiator tRNA first bind on the ribosome? - A site. - E site. - P site. - In any of the three sites (A, P, or E).

P site.

Which of these is not part of the cytoskeleton? - Actin filaments. - Intermediate filaments. - Plasma membrane. - Microtubules.

Plasma membrane.

Which of the following are the plant equivalent of a gap junction? - Desmosomes. - Plasmodesmata. - Tight junctions. - Adherens junctions.

Plasmodesmata.

Protein molecules that have a quaternary structure must have two or more of which of the following? - Alpha-helices and Beta-sheets. - Binding sites. - Disulfide bonds. - Proteins domains. - Polypeptide chains.

Polypeptide chains.

What is one of the main differences in the behavior of the proteins in a vesicle destined for constitutive secretion, and the proteins in the vesicle destined for regulated secretion? - Proteins in the regulated secretion vesicle tend to aggregate and become highly concentrated in the ionic conditions in the vesicle. - Proteins in the constitutive secretion vesicle are stored for release in response to a signal. - The regulated secretion vesicle proteins are bound to lipids for supplying the plasma with new lipids. - Proteins in the constitutive secretion vesicle unfold to keep the concentration in the vesicle low, and then refold at the cell surface.

Proteins in the regulated secretion vesicle tend to aggregate and become highly concentrated in the ionic conditions in the vesicle.

In the extracellular matrix of animal tissues, which of the following molecules allows the matrix to resist compression? - Fibroblasts. - Collagen. - Integrin. - Fibronectin. - Proteoglycans.

Proteoglycans.

Which of the following statements is false? - Tyrosine kinases phosphorylate intracellular proteins on tyrosines. - Serine/threonine kinases are opposed by the action of tyrosine kinases. - Many protein kinases are themselves activated by phosphorylation. - Serine/threonine kinases phosphorylate intracellular proteins on serines or threonines. - Intracellular signaling pathways can include both tyrosine kinases and serine/threonine kinases.

Serine/threonine kinases are opposed by the action of tyrosine kinases.

Cdk inhibitors inhibit the function of cyclin-dependent protein kinases. Different inhibitors display varying mechanisms of Cdk inhibition. Which of the following might a Cdk inhibitor block in order to block Cdk function? (choose one or more). - Substrate binding. - ATP binding. - Cyclin binding. - Movement of the T loop into the active site.

Substrate binding. ATP binding. Cyclin binding.

The synthesis of ATP in glycolysis occurs by which process? - A transfer of phosphate from AMP. - Oxidative phosphorylation. - Substrate-level phosphorylation. - A transfer of electrons from NADH. - Substrate-level dephosphorylation.

Substrate-level phosphorylation.

Which term describes a coupled transporter that moves both solutes in the same direction across a membrane? - Symport. - Antiport. - Uniport.

Symport.

What is the first step that must occur to repair damage on one strand of the double helix? - The DNA backbone on both strands must be cut to make a double-strand break. - The DNA backbone must be sealed. - The gap caused by the damage must be filled. - The damaged region must be removed.

The damaged region must be removed.

The NADH generated during glycolysis and the citric acid cycle feeds its high-energy electrons to which of the following? - ADP. - FAD. - The electron transport chain. - H2O. - The citric acid cycle.

The electron transport chain.

What is a major problem with using foreign cells grown in culture for transplantation in humans? - Such cells would die immediately upon their removal from culture. - The host immune system would destroy such cells. - Such cells would not successfully integrate into host tissue. - Such cells would tend to form cancers.

The host immune system would destroy such cells.

What is the source of energy that drives transcription elongation forward? - The energy of ATP formation. - The hydrolysis of high-energy bonds of ribonucleoside triphosphates. - The phosphorylation of the tail of RNA polymerase. - The breaking of hydrogen bonds as the DNA double helix is pried apart. - The hydrolysis of ATP by the general transcription factors associated with RNA polymerase.

The hydrolysis of high-energy bonds of ribonucleoside triphosphates.

Reverse genetics is defined as: - The inactivation of a gene of interest to determine its function. - The isolation of mutants that display a specific phenotype. - The reinvigoration of old cells into young cells. - The study of inheritance in organisms with RNA as genetic material.

The inactivation of a gene of interest to determine its function.

Signaling via a GPCR ceases when which condition occurs? - The G protein dissociates from the activated GPCR. - The α subunit exchanges GDP for GTP. - The α subunit hydrolyzes its bound GTP. - The α subunit dissociates from the βγ complex. - The G protein associates with an activated GPCR.

The α subunit hydrolyzes its bound GTP.

For each acetyl group that enters the citric acid cycle, what activated carriers are produced? - Three NADH, one FADH2, one GTP. - Three NADH, one FADH2, one ATP. - Three NADH, two FADH2, one GTP. - Three NADH, one FAD, one GTP. - Three NADPH, one FADH2, one GTP.

Three NADH, one FADH2, one GTP.

In principle, how many reading frames in an RNA molecule can potentially be translated into protein? - One. - Two. - Three. - Twenty. - Unlimited.

Three.

Solve the riddle by determining which base could be speaking. "Although my nucleotide partner in double-stranded DNA is nearly twice my size, we are remarkably well matched. Of course, when the heat is on, we tend to go our separate ways easily because we are only connected by two hydrogen bonds. Who am I?" - Thymine. - Uracil. - Guanine. - Adenine. - Cytosine.

Thymine.

Which epithelial cell junctions serve to seal neighboring cells together so that water-soluble molecules cannot easily leak between them? - Adherens junctions. - Gap junctions. - Desmosomes. - Tight junctions. - Plasmodesmata.

Tight junctions.

Which contains the largest number of carbon-carbon double bonds? - Vegetable oil. - Margarine. - Butter. - Saturated fats. - Lard.

Vegetable oil.

Generally, bacterial promoters that are regulated by transcriptional activators bind ___________ to RNA polymerase on their own, but promoters that are regulated with transcriptional repressors bind ___________ to RNA polymerase. - Weakly; strongly. - Strongly; strongly. - Weakly; weakly. - Strongly; weakly.

Weakly; strongly.

Which statement regarding G proteins is true? - The GTPase activity resides in the βγ complex. - When a G protein is inactive, GDP is bound to its α subunit. - The β subunit is tethered to the membrane by a short lipid tail. - The GTPase activity resides in the β subunit. - Only the α subunit interacts with target proteins.

When a G protein is inactive, GDP is bound to its α subunit.

Which of the following organisms have mitochondria in their cells? (choose one or more). - Yeasts. - Animals. - Plants. - Bacteria. - Protozoa.

Yeasts. Animals. Plants. Protozoa.

Which of the following events occur when TATA-binding protein binds to the DNA? (choose one or more). - An eight-stranded β-sheet domain of the TATA-binding protein lies on the DNA helix. - The DNA backbone is kinked nearly 90 degrees. - Four α helices separate the two strands of DNA. - Binding leads to assembly of the rest of the transcription complex at the initiation site.

An eight-stranded β-sheet domain of the TATA-binding protein lies on the DNA helix. The DNA backbone is kinked nearly 90 degrees. Binding leads to assembly of the rest of the transcription complex at the initiation site.

The proton gradient that drives ATP synthesis during photosynthesis is generated by which of the following? - An electron carrier that receives electrons from photosystem I. - An electron carrier that removes electrons from water. - An electron carrier that pumps protons out of the stroma into the thylakoid space. - The operation of two photosystems that work in series. - An electron carrier that pumps protons out of the thylakoid space into the stroma.

An electron carrier that pumps protons out of the stroma into the thylakoid space.

How do fermentation and anaerobic respiration differ? - Fermentation requires NADH, while anaerobic respiration does not. - Anaerobic respiration generates ATP, while fermentation does not. - Anaerobic respiration involves an electron transport chain, while fermentation does not. - Fermentation regenerates NAD+, while anaerobic respiration does not. - Fermentation requires molecular oxygen, while anaerobic respiration does not.

Anaerobic respiration involves an electron transport chain, while fermentation does not.

Match each of the following events with the correct binding to or release of nucleotide from myosin.

Binding to ATP: Myosin detaches from actin. ATP Hydrolysis: Recocking of the lever arm of myosin in preparation for next power stroke, weak binding of myosin to actin. Pi Release: Strengthens myocin binding to actin and triggers power stroke, moving actin. ADP Release: Allows ATP to bind to nucleotide binding site on myosin.

What is required for PKC activation? - Continuing presence of Ca2+. - Binding to Gq and DAG. - Binding to DAG and continuing presence of Ca2+. - Binding to Gq. - Binding to DAG.

Binding to DAG and continuing presence of Ca2+.

Which of the following would increase the level of muscle contraction? (choose one or more). - Mutation in troponin such that it no longer binds tropomyosin. - Blockage of the Ca2+ pump. - Addition of a leaky Ca2+ channel to the sarcoplasmic reticulum. - Addition of a molecule to bind free Ca2+.

Blockage of the Ca2+ pump. Addition of a leaky Ca2+ channel to the sarcoplasmic reticulum.

Which of the following would increase phosphorylation of CREB by PKA? - Blocking nuclear entry of PKA - Blocking binding of the regulatory subunits to the catalytic subunits of PKA. - Blocking cAMP binding to the regulatory subunits of PKA. - Blocking ATP binding to the active site of PKA.

Blocking binding of the regulatory subunits to the catalytic subunits of PKA.

How is muscle contraction reversed? - Calcium is neutralized by an influx of negatively charged ions, which block calcium from binding to myosin. - Calcium is pumped out of the cell to reverse the voltage and cause myosin to degrade. - Calcium binds to calmodulin and is quickly degraded to relieve calcium's inhibition of tropomyosin. - Calcium ion pumps in the sarcoplasmic reticulum reduce the cytosolic calcium levels and tropomyosin blocks the myosin binding site on actin.

Calcium ion pumps in the sarcoplasmic reticulum reduce the cytosolic calcium levels and tropomyosin blocks the myosin binding site on actin.

When fatty acids are oxidized to produce acetyl CoA, each cycle of the reaction removes how many carbon atoms from the fatty acid molecule? - It depends on whether the initial fatty acid has an even or an odd number of carbons. - 3. - 2. - Fatty acids are not oxidized to produce acetyl CoA. - 1.

2.

Which of the following describes the chromosomal makeup of a somatic cell of a human biological male? - 23 pairs of autosomes + 1 X chromosome + 1 Y chromosome. - 22 pairs of autosomes + 1 X chromosome + 1 Y chromosome. - 23 pairs of autosomes + 1 Y chromosome. - 23 pairs of autosomes.

22 pairs of autosomes + 1 X chromosome + 1 Y chromosome.

A primary transcript (immature, non-processed) single-stranded RNA molecule has the following nucleotide composition: 30% A, 20% G, 24% C, and 26% U. What is the nucleotide composition of the double-stranded DNA molecule from which it was transcribed? - 30% A, 20% G, 24% C, and 26% T. - 26% A, 24% G, 20% C, and 30% U. - 20% A, 30% G, 26% C, and 24% T. - 28% A, 22% G, 22% C, and 28% T. - 26% A, 24% G, 20% C, and 30% T

28% A, 22% G, 22% C, and 28% T

In 1952, Alfred Hershey and Martha Chase, working with a virus called T2, conducted what is now considered a landmark experiment to determine whether genes are made of DNA or protein. When this virus, which is made entirely of DNA and protein, infects E. coli, it injects its genetic material into the bacterial cell, leaving the empty virus head stuck to the cell surface. To determine whether it was DNA or protein that enters the infected bacterial cell, the researchers radioactively labeled one batch of T2 with the isotope 35S, which resulted in radiolabeled viral proteins, and a second batch of T2 with 32P, which resulted in radiolabeled DNA. They then incubated the radioactive viruses with E. coli. After allowing a few minutes for the viruses to transfer their genetic material to the bacterial cells, the researchers used a blender to shear the empty virus heads from the bacterial cell surface. They then used a centrifuge to separate the infected bacteria from the empty virus heads: spinning the sample at high speed caused the heavier, infected bacteria to pellet at the bottom of the centrifuge tube, while the lighter, empty virus heads remained in solution. Using this protocol, what should the researchers have seen in terms of the distribution of radioactivity in the centrifuged sample? - 32P in the pellet, 35S in the solution. - No radioactivity in the solution. - No radioactivity in the pellet. - An equal amount of 35S and 32P in the pellet and the solution. - 35S in the pellet, 32P in the solution.

32P in the pellet, 35S in the solution

If a stretch of DNA on the parental strand of a replicating chromosome has the sequence 5ʹ-AGCTCGATCGGCTA-3ʹ, what will the sequence of the newly synthesized strand made from this stretch of template be? - 5ʹ-TCGAGCTAGCCGAT-3ʹ - 3ʹ-TCGAGCTAGCCGAT-5ʹ - 3ʹ-AGCTCGATCGGCTA-5ʹ - 5ʹ-AGCTCGATCGGCTA-3ʹ

3ʹ-TCGAGCTAGCCGAT-5ʹ

An RNA chain elongates in what direction? - 5'-to-3' only. - It depends on which RNA polymerase does the transcribing. - 3'-to-5' only. - It depends on which DNA strand acts as the template.

5'-to-3' only.

How thick is the plasma membrane? - 50mm. - 50nm. - 50 atoms. - 50 meters. - 50 μm.

50 atoms.

In a typical animal cell, proteins constitute what percentage of the mass of the plasma membrane? - 98% - 2% - 25% - 50% - 75%

50%

How does an action potential spread along the cell membrane? - The ions entering the cell upon triggering an action potential travel laterally along the membrane to carry the charge. - A change in membrane potential triggers the opening of nearby voltage-gated sodium channels in a one-way direction. - Voltage-gated Ca2+ channels are activated by the action potential and the calcium diffuses along the membrane. - Potassium leak channels quickly reverse the action potential to move the membrane depolarization away from the original site.

A change in membrane potential triggers the opening of nearby voltage-gated sodium channels in a one-way direction.

A single-pass transmembrane protein destined for one of the organelles in the endomembrane system would be marked by what type of signal sequence? - A cleaved N-terminal ER signal sequence - An internal ER signal sequence and an internal stop-transfer sequence. - A cleaved N-terminal ER signal sequence and an internal stop-transfer sequence. - An internal ER signal sequence.

A cleaved N-terminal ER signal sequence and an internal stop-transfer sequence.

What is a polyribosome? - A mutant ribosome that contains additional ribosomal subunits. - A ribosome that is in the process of translating an mRNA into a polypeptide. - A ribosome translating an mRNA molecule that encodes a membrane-embedded protein. - A cluster of ribosomes simultaneously translating the same mRNA, but positioned at different sites along the mRNA. - A ribosome translating a polycistronic mRNA molecule.

A cluster of ribosomes simultaneously translating the same mRNA, but positioned at different sites along the mRNA.

Which of the following describes the resting membrane potential of a neuron? - A voltage difference across the plasma membrane when the neuron has been stimulated. - A voltage difference that is chiefly a reflection of the electrochemical Na+ gradient across the plasma membrane. - A voltage difference across the plasma membrane, with more positive membrane potential inside. - A state in which the flow of positive and negative ions across the plasma membrane is precisely balanced.

A state in which the flow of positive and negative ions across the plasma membrane is precisely balanced.

Inside a muscle fiber, what triggers sarcomeres to contract? - A sudden rise in Ca2+ inside an organelle lumen. - A sudden rise in cytosolic Ca2+. - A sudden rise in cytosolic Na+. - Polarization of the muscle fiber plasma membrane. - An increased availability of ATP.

A sudden rise in cytosolic Ca2+.

The surface of the villi in the small intestine is covered by a single layer of epithelial cells called enterocytes. Enterocytes absorb nutrients from the intestinal lumen as they move up the side of the villus. The enterocytes are shed from the end of the villus through apoptosis after 3 to 4 days. Consequently, new cells must be continuously supplied from the crypt and move up the villus. When a stem cell in the crypt divides, what are its immediate daughter cells? - A transit amplifying cell and a goblet cell. - Another stem cell and a transit amplifying cell. - Another stem cell and an enterocyte. - A transit amplifying cell and an enterocyte. - Another stem cell and a goblet cell.

Another stem cell and a transit amplifying cell.

Cardiac muscle cells contain a Na+/Ca2+ transporter responsible for maintaining a low cytosolic Ca2+ concentration, which helps regulate cardiac muscle contraction. Ca2+ is transported out of the cell as Na+ is brought into the cell. What type of transporter is this protein? - Antiport. - Channel. - Uniport. - Symport.

Antiport.

Why would a cell express the aquaporin protein if water can cross the membrane in the absence of aquaporin? - Water molecules cannot cross the membrane in the absence of a pore like aquaporin. - Aquaporin facilitates the faster movement of water molecules across the membrane. - Aquaporin moves a positively charged ion along with water across the membrane. - Aquaporin limits the movement of water molecules so they do not move too quickly across the membrane.

Aquaporin facilitates the faster movement of water molecules across the membrane.

Which of the following are prokaryotes? - Bacteria and fungi. - Viruses and microbes. - Archaea and protozoans. - Archaea and bacteria. - Monotremes and prototheria.

Archaea and bacteria.

Which of these three classifications is included under one of the others: prokaryotes, eukaryotes, archaea? - Prokaryotes. - Eukaryotes. - Archaea. - None of these, as these are all distinct cell types from one another.

Archaea.

The cell-cycle control system uses Cdk inhibitory proteins to do what? - Degrade cyclins. - Trigger different phases of the cell cycle. - Arrest the cell cycle at specific transition points. - Degrade Cdks.

Arrest the cell cycle at specific transition points.

Where or when does RNA capping at the 5' end of the transcript take place? - When an RNA exits the nucleus. - After RNA splicing. - Once an RNA is released by RNA polymerase - Within the nuclear pore complex. - As an RNA is being transcribed.

As an RNA is being transcribed.

Actin filaments, microtubules, and the spherical shells of certain virus particles are all structures built from which of the following? - More than one type of lipid. - More than one type of protein. - Only one type of polysaccharide. - Assemblages of identical proteins. - A combination of proteins and polysaccharides.

Assemblages of identical proteins.

In a globular protein, where will the amino acid that occurs at the N-terminus generally be found? - In the protein's interior. - In the middle of an alpha-helix. - All of these locations. - At the protein's surface. - In the middle of a beta-sheet.

At the protein's surface.

The sequence at which DNA replication begins tends to have which characteristic? - G-rich. - AT-rich. - A-rich. - GC-rich.

AT-rich.

Which of the following would most likely interact by forming an ionic bond? - Urea and water. - ATP and magnesium cations. - Fatty acid chain and inorganic phosphate. - Two strands of DNA. - The enzyme hexokinase and its substrate glucose.

ATP and magnesium cations.

Most of the energy released by oxidizing glucose is saved in the high-energy bonds of what molecules? - ATP and other activated carriers. - H2O and CO2. - GDP and other activated carriers. - O2. - ADP and other activated carriers.

ATP and other activated carriers.

The drug 2,4-dinitrophenol (DNP) makes the mitochondrial inner membrane permeable to H+. The resulting disruption of the proton gradient inhibits the mitochondrial production of ATP. What additional effect would DNP have on the transport of ATP out of the mitochondrial matrix? - None, because the inner membrane is permeable to ATP. - ATP export will decrease because its carrier exploits the difference in voltage across the inner membrane. - None, because ATP export is not coupled to the movement of protons across the inner membrane. - ATP transport will increase because ATP synthase will be forced to operate in the "reverse" direction. - ATP transport will decrease because less ATP will be available to diffuse across the inner membrane.

ATP export will decrease because its carrier exploits the difference in voltage across the inner membrane.

What happens when ATP synthase operates "in reverse" and pumps H+across a membrane against its electrochemical proton gradient? - ATP is hydrolyzed to form ADP and Pi. - Nutrients are transported in the opposite direction across the membrane. - ATP is synthesized from ADP and Pi. - ATP is co-transported across the membrane. - Na+ is transported in the opposite direction across the membrane.

ATP is hydrolyzed to form ADP and Pi.

Suppose the shaft of ATP synthase that is attached to the rotor were truncated (shortened) such that it no longer extended into the F1 ATPase head. What would be the consequence of this mutation? - ATP synthase would work in reverse, breaking down ATP and pumping protons against their gradient. - ATP would not be produced because the conformation of the F1 ATPase head would not be changed. - The rotor in the membrane would no longer turn. - Protons would not cross the membrane using the rotor.

ATP would not be produced because the conformation of the F1 ATPase head would not be changed.

What does a target cell require to respond to an extracellular signal molecule? (choose one or more). - Access to the signal molecule. - Effector molecules that alter cell behavior in response to the signal molecule. - The appropriate machinery to produce and secrete the signal molecule. - A receptor that recognizes the signal molecule. - Appropriate intracellular signaling pathways.

Access to the signal molecule. Effector molecules that alter cell behavior in response to the signal molecule. A receptor that recognizes the signal molecule. Appropriate intracellular signaling pathways.

In cells, pyruvate can be converted to which of the following? (choose one or more). - Alanine. - Oxaloacetate. - Lactate. - Glucose. - Acetyl CoA.

Alanine. Oxaloacetate. Lactate. Glucose. Acetyl CoA.

Where does the splicing of pre-mRNAs take place? - In the nucleus as the RNA is still being transcribed. - In the cytosol when associating with a ribosome. - In the cytosol when not associating with a ribosome.

In the nucleus as the RNA is still being transcribed.

What is the conformation of the voltage-gated Na+ channel that keeps the action potential from traveling backward along the axonal membrane? - Closed. - Inactivated. - Open. - Triggered.

Inactivated.

When a ligand binds to an allosteric enzyme's regulatory site, it changes the activity of that enzyme by: - Denaturing the enzyme. - Inducing a conformational change. - Directly blocking the active site. - Acting as a chaperone.

Inducing a conformational change.

Which of the following alterations to the signaling pathway would lead to increased transcription by the CREB protein? (choose one or more). - Inhibition of GTP hydrolysis. - Inhibition of entry of PKA into the nucleus. - Inhibition of cAMP breakdown. - Inhibition of adenylyl cyclase.

Inhibition of GTP hydrolysis. Inhibition of cAMP breakdown.

When activated, a G-protein has a structure that: - Is bound to GTP. - Forms a complete complex with three subunits. - Is able to bind to extracellular signals. - Forms a transmembrane domain.

Is bound to GTP.

In bacterial cells, the tryptophan operon encodes the genes needed to synthesize tryptophan. What happens when the concentration of tryptophan inside a cell is high? - It activates the tryptophan repressor, which breaks down excess tryptophan. - It inactivates the tryptophan repressor, which shuts down the tryptophan operon. - It inactivates the tryptophan repressor, allowing transcription of the tryptophan operon. - It activates the tryptophan repressor, allowing transcription of the tryptophan operon. - It activates the tryptophan repressor, which shuts down expression of the tryptophan operon.

It activates the tryptophan repressor, which shuts down expression of the tryptophan operon.

Which statements are true of the cytochrome c oxidase complex? (choose one or more). - It acts as a proton pump. - It produces nearly all of the oxygen we breathe. - Its redox potential is higher than that of oxygen. - It reduces molecular oxygen.It oxidizes molecular oxygen. - It oxidizes cytochrome c. - It reduces cytochrome c.

It acts as a proton pump. It reduces molecular oxygen. It oxidizes cytochrome c.

How does an allosteric inhibitor work? - It binds to a site other than the active site, causing a conformational change in the enzyme that makes the active site less accommodating to the substrate. - It outcompetes the substrate molecule and binds to the active site, preventing substrate molecules from binding there. - It binds to a site other than the active site, causing a conformational change in the enzyme that forces the product to leave the active site. - It interacts covalently with the substrate, preventing it from fitting into the enzyme's active site.

It binds to a site other than the active site, causing a conformational change in the enzyme that makes the active site less accommodating to the substrate.

What is true of the inhibition of APC activation? - It blocks entry to S phase. - It blocks exit from mitosis. - It blocks entry to mitosis. - It triggers anaphase. - It triggers entry into mitosis.

It blocks exit from mitosis.

How does methylation of histone tails affect the accessibility of DNA? - It relaxes the chromatin to make DNA more accessible. - Histone methylation has no effect on chromatin. - It can have different effects depending on the location. - It compacts the chromatin to make DNA less accessible.

It can have different effects depending on the location.

Which of the following correctly describes phosphorylation of a protein? - It can increase or decrease the protein's activity. - It is an irreversible protein modification. - It always increases the protein's activity. - It is catalyzed by a protein phosphatase. - It always decreases the protein's activity.

It can increase or decrease the protein's activity.

What is true of the antenna complex of a photosystem? - It passes electrons to a photosynthetic electron transport chain. - It extracts electrons from water. - It converts light energy into chemical energy. - It includes a special pair of chlorophylls. - It captures light energy.

It captures light energy.

Why does nitric oxide (NO) cause blood vessels to dilate? - It causes smooth muscle cells in the walls of blood vessels to contract. - It inhibits cyclic GMP production by inhibiting guanylyl cyclase. - It causes smooth muscle cells in the walls of blood vessels to relax. - It causes endothelial cells lining blood vessels to relax. - It causes endothelial cells lining blood vessels to contract.

It causes smooth muscle cells in the walls of blood vessels to relax.

Which is true of gluconeogenesis? - It consumes four molecules of ATP and two molecules of GTP to produce one molecule of glucose. - It is the opposite of glycolysis in that it requires an initial investment of four molecules of ATP and generates two molecules of ATP for each molecule of glucose produced. - It requires an initial investment of two molecules of ATP, but it produces four molecules of ATP for each molecule of pyruvate consumed. - It requires an initial investment of four molecules of ATP, but it produces two molecules of GTP plus two molecules of glucose. - It is the opposite of glycolysis in that it requires an initial investment of four molecules of ATP and generates two molecules of ATP for each molecule of glucose consumed.

It consumes four molecules of ATP and two molecules of GTP to produce one molecule of glucose.

What does the pyruvate dehydrogenase complex do? - It converts pyruvate into acetyl CoA in the mitochondrial matrix. - It regenerates NAD+ in the mitochondrial matrix. - It converts pyruvate into acetyl CoA in the cytosol. - It produces pyruvate in the cytosol. - It completes the oxidation of pyruvate to CO2 in the mitochondrial matrix.

It converts pyruvate into acetyl CoA in the mitochondrial matrix.

Which of the following statements about the cell-cycle control system is not true? - It ensures that the environment is favorable for the cell to initiate DNA replication. - It ensures that all DNA is replicated before M phase begins. - It ensures that all chromosomes are attached to the mitotic spindle before the chromosomes are segregated to the two daughter cells. - It ensures that M phase is completed before cytokinesis begins.

It ensures that M phase is completed before cytokinesis begins.

As a polypeptide is being translocated across the membrane of the endoplasmic reticulum, a stop-transfer sequence can halt the process. What eventually becomes of this stop-transfer sequence? - It is cleaved from the protein.It forms an α-helical membrane-spanning segment of the protein. - It remains in the cytosol. - It stops protein synthesis and causes the ribosome to be released back to the cytosol. - It is translocated into the lumen of the endoplasmic reticulum.

It forms an α-helical membrane-spanning segment of the protein.

During photosynthesis, what happens in the water-splitting step? - It takes place during stage 2 of photosynthesis. - It consumes protons and thereby depletes the proton gradient across the thylakoid membrane. - It is catalyzed by an enzyme containing an iron-sulfur cluster. - It occurs on the stromal side of the thylakoid membrane. - It generates essentially all of the O2 in the Earth's atmosphere.

It generates essentially all of the O2 in the Earth's atmosphere.

The ER signal sequence on a growing polypeptide chain is recognized by a signal-recognition particle (SRP) in the cytosol. What does this interaction accomplish? - It returns the ribosome to the pool of free ribosomes in the cytosol. - It cleaves the ER signal sequence from the polypeptide chain. - It releases the polypeptide chain from the ribosome. - It speeds the synthesis of the polypeptide chain. - It guides the ribosome and its polypeptide to the ER.

It guides the ribosome and its polypeptide to the ER.

How does the GTP-bound form of a GTP-binding protein switch to a GDP-bound form? - It releases GTP and takes up GDP from the cytosol. - In the presence of high concentrations of GDP, it trades GTP for GDP. - It hydrolyzes GTP, releasing a phosphate. - It releases GTP and takes up GDP from the mitochondrion. - It interacts with a phosphatase, which removes a phosphate from GTP.

It hydrolyzes GTP, releasing a phosphate.

Which statement is true of glycogen phosphorylase? - It is inhibited by glucose 6-phosphate, but activated by ATP. - It is inhibited by glucose 6-phosphate and by ATP. - It is activated by glucose 6-phosphate, but inhibited by ATP. - It is activated by glucose 6-phosphate and by ATP. - It stimulates gluconeogenesis.

It is inhibited by glucose 6-phosphate and by ATP.

How does the error rate of RNA polymerases compare to the error rate of DNA polymerase (one error in 107 nucleotides)? - It is much less. - It is about the same. - It is much greater.

It is much greater.

During the citric acid cycle, what happens to the acetyl group in acetyl CoA? - It is reduced to produce CO2. - It is oxidized to produce CO2. - It is eliminated to produce CO2. - It is vaporized to produce CO2. - It is hydrolyzed to produce CO2.

It is oxidized to produce CO2.

What is the role of the sliding clamp during replication? - It hydrolyzes ATP to push the DNA polymerase along the DNA template. - It keeps the RNA primer attached to the lagging strand. - It keeps DNA polymerase attached to the template while the polymerase synthesizes a new strand of DNA. - It loads DNA helicase onto the replication fork. - It unwinds the double helix at the replication fork to allow DNA polymerase to progress along the DNA.

It keeps DNA polymerase attached to the template while the polymerase synthesizes a new strand of DNA.

Which of the following accurately describes the role of the Na+-K+pump? - It equilibrates the concentrations of Na+ and K+across the plasma membrane. - It maintains a higher K+ concentration outside the cell. - It maintains a higher Na+ concentration outside the cell. - It maintains a lower Na+ concentration outside the cell.

It maintains a higher Na+ concentration outside the cell.

How does IP3 function in the inositol phospholipid pathway? - It produces diacylglycerol. - It directly activates protein kinase C (PKC). - It opens Ca2+ channels that are embedded in the ER membrane, allowing Ca2+ to enter the cytosol. - It directly activates phospholipase C (PLC). - It generates a hole in the endoplasmic reticulum (ER), through which Ca2+ can enter the cytosol.

It opens Ca2+ channels that are embedded in the ER membrane, allowing Ca2+ to enter the cytosol.

A reporter gene is an experimentally engineered regulatory DNA sequence from a gene of interest that has been fused to a gene that encodes a protein that is easily observed experimentally. Why is this approach useful? - It can provide information as to when a gene is expressed. - It provides information on the binding interactions of the gene product. - It can provide information as to where a gene is expressed. - It provides information into where and when a gene is expressed.

It provides information into where and when a gene is expressed.

If a signal sequence is removed from an ER protein, what happens to the altered protein? - It can enter any organelle other than the ER. - It is unable to exit the ER. - It is immediately degraded. - It is escorted to the ER by chaperone proteins. - It remains in the cytosol.

It remains in the cytosol.

Which is true of the carbon-fixation (Calvin) cycle? (choose one or more). - It produces NADPH and ATP. - It requires NADPH and ATP.It produces CO2. - It requires CO2.It produces O2. - It requires energy-rich ribulose 1,5-bisphosphate. - It regenerates energy-rich ribulose 1,5-bisphosphate. - It requires O2.

It requires NADPH and ATP. It requires CO2. It requires energy-rich ribulose 1,5-bisphosphate. It regenerates energy-rich ribulose 1,5-bisphosphate.

What is true about the phosphorylation of condensins by M-Cdk? - It triggers their assembly onto DNA, allowing chromosomes to decondense. - It marks them for degradation, allowing sister chromatids to separate. - It triggers their assembly onto DNA, keeping sister chromatids held together. - It triggers their assembly onto DNA, compressing chromosomes into a more compact form. - It marks them for degradation, allowing chromosomes to decondense.

It triggers their assembly onto DNA, compressing chromosomes into a more compact form.

In passive transport, the net movement of a charged solute across the membrane is determined by which of the following? - Its electrochemical gradient. - The membrane potential - Its osmotic gradient alone. - Its concentration gradient.

Its electrochemical gradient.

Which of the following is NOT used as a source of energy by a transmembrane pump to actively transport a solute? - K+ - ATP - H+ - Na+

K+

Which membrane transport protein randomly flickers between open and closed states regardless of the conditions inside or outside of the cell? - Na+ pump. - Aquaporins. - Ca2+ pump. - H+ pump. - K+ leak channels.

K+ leak channels.

With the identification of skin stem cells, it has become possible to add the normal copy of a mutated gene into skin stem cells in vitro, grow the skin, and then transplant onto the patient. This approach was used to successfully treat a patient with a different form of epidermolysis bullosa known as junctional epidermolysis bullosa. What gene would need to be introduced into the skin stem cells of a patient with epidermolysis bullosa simplex in order to regain proper epithelial function? - Nuclear lamins. - Myosin. - Keratin. - Tubulin.

Keratin.

The catalytic sites for peptide bond formation during translation is found in which part of the ribosome? - Large subunit RNAs. - Small subunit proteins. - Large subunit proteins. - Small subunit RNAs.

Large subunit RNAs.

The energy in ATP is used to fuel the movement of kinesin motor proteins along microtubules. Sort each of the following events into the proper category, indicating whether they occur on the leading or trailing head of kinesin.

Leading Head: ADP release. ATP binding. Neck linker zippers to catalytic core of head group. Trailing Head: ATP hydrolysis. Pi release.

Which of the following statements concerning leucine zipper protein dimerization and DNA binding is correct? - Leucine zipper proteins contain many leucine amino acids in the DNA-binding region that facilitate sequence-specific DNA binding. - Leucine zipper protein dimerization is facilitated by polar amino acids in the dimerization domains. - Leucine zipper proteins function as a dimer with both subunits making contact with the sequence-specific DNA site. - Leucine zipper proteins use ionic bonds to bind with the sequence-specific DNA site.

Leucine zipper proteins function as a dimer with both subunits making contact with the sequence-specific DNA site.

Which statements are true regarding protein transport into peroxisomes? (choose one or more). - Like proteins destined for the ER, nucleus, or mitochondria, peroxisomal proteins bear a distinct signal sequence. - Like mitochondria, peroxisomes contain translocators through which proteins enter the organelle. - Like proteins transported into mitochondria, proteins can enter the peroxisome in a folded conformation. - Like mitochondrial proteins, some peroxisomal proteins arrive via vesicles that bud from the ER. - Like prospective nuclear proteins, some peroxisomal proteins are recognized by receptor proteins in the cytosol. - Like proteins transported into the nucleus, proteins must unfold before entering the peroxisome.

Like proteins destined for the ER, nucleus, or mitochondria, peroxisomal proteins bear a distinct signal sequence. Like mitochondria, peroxisomes contain translocators through which proteins enter the organelle. Like prospective nuclear proteins, some peroxisomal proteins are recognized by receptor proteins in the cytosol.

Covalent bonds in macromolecules are primarily important for: - Enzyme-substrate binding. - Linking together monomers. - Interactions with other macromolecules.- Forming the three-dimensional folded conformation.

Linking together monomers.

________ are the most abundant molecules in the animal cell membrane, whereas ________ make up 50% of the membrane by mass. - Lipids, carbohydrates. - Carbohydrates, lipids. - Lipids, proteins. - Proteins, lipids.

Lipids, proteins.

In humans and other mammals, gluconeogenesis is most likely to occur in which type of cells? - Muscle cells. - Red blood cells. - Liver cells. - Pancreatic cells. - Nerve cells.

Liver cells.

The reading frame to use for translating an mRNA into functional protein is determined by the: - Location of an AUG. - 5' cap. - Replication origin. - Promoter sequence.

Location of an AUG.

A cell membrane made up primarily of lipids with which characteristics would be the least fluid (i.e., most stiff)? - Long, saturated fatty acid tails. - Short, unsaturated fatty acid tails. - Long, unsaturated fatty acid tails. - Short, saturated fatty acid tails.

Long, saturated fatty acid tails.

Abnormally stretchy skin is part of a genetic syndrome that could result from which of the following? - Loss of proteinase that cleaves procollagen. - Synthesis of excess cellulose. - Increased production of collagen. - Overactivity of proteinase that cleaves procollagen. - Hypersecretion of procollagen.

Loss of proteinase that cleaves procollagen.

Which of the following is a small, irregularly shaped organelle in which intracellular digestion occurs? - Lysosome. - Endoplasmic reticulum. - Peroxisome. - Ribosome. - Gogli apparatus.

Lysosome.

Strong and specific associations between macromolecules or between an enzyme and its substrate are due to: - Many strong non-covalent bonds. - Many strong covalent bonds. - Many weak non-covalent bonds. - Few strong covalent bonds.

Many weak non-covalent bonds.

In addition to its role in DNA repair, homologous recombination is also responsible for generating genetic diversity during what process? - Mitosis. - Meiosis. - DNA replication. - Fertilization. - Independent assortment of chromosomes.

Meiosis.

What is the voltage difference across a membrane of a cell called? - Potential balance. - Membrane potential. - Gradient establishment. - Electrical current.

Membrane potential.

Imagine you collected bacteria from the sediment in a frozen lake in Minnesota in January and compared the membranes to membranes from bacteria collected from a lake in Texas in June. Consider how the membranes would likely differ. The membranes in bacteria from the Minnesota lake would most likely have which of the following? - More unsaturated lipid tails than membranes in Texas bacteria. - Fewer lipid tails with cis double bonds than membranes in Texas bacteria. - Phospholipids with more negatively charged phosphate groups than membranes in Texas bacteria. - More saturated lipid tails than membranes in Texas bacteria.

More unsaturated lipid tails than membranes in Texas bacteria.

To identify genes coding for essential proteins, researchers can create temperature-sensitive mutations. These mutations allow proper protein folding and cell proliferation at the permissive temperature of 22ºC, but they cause protein mis-folding and reduced cell proliferation at a higher restrictive temperature, such as 37ºC. Which of the following mutations might increase protein flexibility and lead to a temperature-sensitive phenotype? (choose one or more). - Mutation of a lysine (that was involved in an ionic bond with a glutamic acid) to a glycine. - Mutation of an alanine to a cysteine, leading to the formation of a new disulfide bond. - A premature stop codon that truncates a protein 10 amino acids from the amino terminus. - Mutation of a bulky isoleucine that was buried in the protein interior to a glycine (side chain = H).

Mutation of a lysine (that was involved in an ionic bond with a glutamic acid) to a glycine. Mutation of a bulky isoleucine that was buried in the protein interior to a glycine (side chain = H).

Which of the following statements is true? - Na+ is the most plentiful positively charged ion outside the cell, while K+ is the most plentiful inside. - K+ and Na+ are both excluded from cells. - K+ and Na+ are present in the same concentration on both sides of the plasma membrane. - K+ is the most plentiful positively charged ion outside the cell, while Na+ is the most plentiful inside. - K+ and Na+ are both maintained at high concentrations inside the cell compared to out.

Na+ is the most plentiful positively charged ion outside the cell, while K+ is the most plentiful inside.

Approximately how often does DNA polymerase make an error during DNA replication? - Once per 1 million nucleotides added to the growing DNA chain. - Never, because DNA polymerase is always self-correcting. - Once per 10 million nucleotides added to the growing DNA chain. - Once per 100 nucleotides added to the growing DNA chain. - Once per 1000 nucleotides added to the growing DNA chain.

Once per 10 million nucleotides added to the growing DNA chain.

In the absence of repair, what would the replication of a double helix containing a mismatch yield? - Two DNA molecules containing the mismatch. - Two DNA molecules containing different mismatches at the site of the original error. - Two DNA molecules with a mutated sequence. - One DNA molecule with the normal sequence and one DNA molecule with a mismatch. - One DNA molecule with the normal sequence and one DNA molecule with a mutated sequence.

One DNA molecule with the normal sequence and one DNA molecule with a mutated sequence.

Why are individual DNA strands considered polar? - One end is positively charged, while the other is negative. - One end terminates in a hydroxyl group, while the other terminates in a phosphate. - One end contains the telomere, while the other contains the centromere. - The strands twist preferentially into a right-handed helix, not a left-handed helix. - Each end contains a different, complementary nucleotide sequence.

One end terminates in a hydroxyl group, while the other terminates in a phosphate.

Which of the following correct describes the structure of the long fibers of a skeletal muscle? - One large cell with many nuclei. - One large cell with a single nucleus. - Many cells organized end to end.

One large cell with many nuclei.

What type of cell response would take the longest amount of time (on the scale of minutes to hours) to execute? - One that uses a phosphorylation event to activate an enzyme. - One that involves the firing of an action potential along a neuron. - One that involves a change in gene expression. - One that involves the release of secretory vesicles.

One that involves a change in gene expression.

One of the first steps in obtaining a karyotype is treating cells with a drug that stalls cells in mitosis. Why must cells arrest in mitosis for karyotype analysis? - Only in mitosis are homologous chromosomes paired up. - Only mitotic chromosome DNA is separated into single strands, allowing for staining by dyes. - Only mitotic cells contain homologous chromosomes, because mitosis happens after DNA replication. - Only mitotic chromosomes are highly condensed and visible with a light microscope.

Only mitotic chromosomes are highly condensed and visible with a light microscope.

Identify which statement(s) is/are true regarding telomere characteristics. - Telomeres allow duplicated chromosomes to become separated into daughter cells during M phase. - Telomeres are found in all living cells. - Telomeres cap the ends of linear chromosomes and prevent them from being recognized by the cell as broken DNA in need of repair. - Telomeres contain repeated nucleotide sequences that are required to replicate the ends of linear chromosomes. - All of the above are true regarding telomere characteristics. - Only statements B and D are true regarding telomere characteristics. - Only statements C and D are true regarding telomere characteristics.

Only statements C and D are true regarding telomere characteristics.

Which is true of the GTP-binding proteins that participate in intracellular signaling? - G-protein-coupled receptors interact with all types of GTP-binding proteins. - Only trimeric GTP-binding proteins interact with guanine nucleotide exchange factors (GEFs). - Only trimeric GTP-binding proteins participate in intracellular cell signaling. - Only monomeric GTP-binding proteins relay messages from G-protein-coupled receptors. - Only trimeric GTP-binding proteins relay messages from G-protein-coupled receptors.

Only trimeric GTP-binding proteins relay messages from G-protein-coupled receptors.

Why does optogenetics hold the potential to help scientists better understand neurobiology? - Optogenetics can be used to analyze neural circuits and complex behavior. - Optogenetics can help us understand how light stimulates neurons in the brain. - Optogenetics can be used to visualize how neurons contact other cells. - Optogenetics can be used to study how Na+channels transport Na+.

Optogenetics can be used to analyze neural circuits and complex behavior.

Which of the following determines the direction that cellulose microfibrils are laid down in the extracellular space of a plant cell? - Orientation of cellulite on the cytoplasmic side of the plasma membrane. - Orientation of microtubules on the cytoplasmic side of the plasma membrane. - Availability of sugar monomers for cellulose microfibril elongation. - Orientation of microtubules in the cell wall. - Amount of turgor pressure within a cell.

Orientation of microtubules on the cytoplasmic side of the plasma membrane.

Which statements below accurately describe an aspect of the citric acid cycle? (choose one or more). - Oxaloacetate is regenerated by the end of the citric acid cycle. - The citric acid cycle produces two kinds of high-energy molecules, GTP and NADH. - The two carbons that enter as acetyl CoA are released in the same cycle as CO2. - NADH is generated in steps 3, 4, and 8.

Oxaloacetate is regenerated by the end of the citric acid cycle. NADH is generated in steps 3, 4, and 8.

The ethanol in wine and beer is produced from metabolic reactions carried out by the yeast Saccharomyces cerevisiae. Since it is of great commercial value, researchers have studied factors that influence ethanol production. To maximize ethanol yield, which environmental factor should be limiting? - Oxygen. - Sunlight. - Carbon dioxide. - Glucose.

Oxygen.

Many of the extracellular signal molecules that regulate inflammation are released locally at the site of infection. What form of cell-cell signaling is being used? - Contact-dependent. - Paracrine. - Endocrine. - Neuronal.

Paracrine.

The movement of an ion down its concentration gradient is called what? - Pumping. - Passive transport. - Active transport. - Osmosis.

Passive transport.

Which biochemical reaction is catalyzed by a ribozyme? - DNA polymerization during DNA replication. - RNA polymerization during transcription in prokaryotes. - RNA polymerization during transcription in eukaryotes. - Peptide bond formation in protein synthesis. - Peptide bond hydrolysis by proteases.

Peptide bond formation in protein synthesis.

In which direction does a plant cell elongate? - Perpendicular to the orientation of the cellulose microfibrils. - Parallel to the orientation of the cellulose microfibrils. - Toward the sun and away from the Earth

Perpendicular to the orientation of the cellulose microfibrils.

Stem cells in the gut epithelium form precursor cells that differentiate as they move ___________ to the epithelial sheet, whereas precursor cells formed from the skin epidermis stem cells move ___________ to the epithelial sheet. - Parallel; parallel. - Perpendicular; parallel. - Parallel; perpendicular. - Perpendicular; perpendicular.

Perpendicular; parallel.

Which of the following refers to the appearance or behavior of an individual? - Haplotype. - Prototype. - Phenotype. - Genotype. - Karyotype.

Phenotype.

What kind of enzyme removes a phosphate group from a protein? - Phosphatase. - Kinase. - ATPase. - Phosphorylase. - GTPase.

Phosphatase.

The type of bond that holds together neighboring subunits in a single strand of DNA is a: - Phosphodiester bond. - Peptide bond. - Hydrogen bond. - Phosphoanhydride bond.

Phosphodiester bond.

Nucleotides in each strand of DNA are held together by what type of bonds? - Hydrogen. - Peptide. - Phosphodiester. - Ionic.

Phosphodiester.

RNA polymerases join nucleotides through what kind of bond? - Phosphodiester. - Hydrogen. - Glycosidic. - Peptide. - Ionic.

Phosphodiester.

Which of the following correctly matches a G-protein-activated enzyme with the second messenger molecule it produces? - Guanylyl cyclase → GTP. - Adenylyl cyclase → inositol triphosphate. - Phospholipase C → diacylglycerol. - Protein kinase A → Ca2+.

Phospholipase C → diacylglycerol.

Which of the following would produce the most fluid lipid bilayer? - Phospholipids with tails of 20 carbon atoms and two double bonds. - Phospholipids with fully saturated tails of 20 carbon atoms. - Large amounts of cholesterol. - Phospholipids with tails of 18 carbon atoms and two double bonds. - Phospholipids with fully saturated tails of 18 carbon atoms.

Phospholipids with tails of 18 carbon atoms and two double bonds.

A group of researchers wanted to sort different white blood cell types (monocytes, lymphocytes, and granulocytes) apart from each other based on size differences and to remove unwanted contaminating red blood cells. After a particular manipulation, the red blood cells lysed. The remaining white blood cells increased in size and, more importantly, the size differences among cells increased, allowing for size-based sorting (which requires minimum size differences among cells). What manipulation did the researchers use to increase cell size? - Placing cells in an environment with a lower solute concentration than that in the cells. - Placing cells in an environment with lower temperatures than the cells were previously exposed to. - Patch-clamp recording to monitor ion channel activity. - Placing cells in an environment with a higher solute concentration than that in the cells.

Placing cells in an environment with a lower solute concentration than that in the cells.

The bacteria Listeria monocytogenes move in and between host cells using a comet-like tail. Which of the following accurately describes Listeria motility? - Depolymerization of host microtubule filaments creates passageways for the Listeria to swim through using their flagella. - Depolymerization of host actin filaments creates passageways for the Listeria to swim through using their flagella. - Polymerization of host microtubule subunits creates a comet-like tail of filaments that pushes the cell forward. - Polymerization of host actin subunits creates a comet-like tail of filaments that pushes the cell forward.

Polymerization of host actin subunits creates a comet-like tail of filaments that pushes the cell forward.

Which of the following is true about "junk DNA"? - It usually codes for proteins but the proteins are nonfunctional in the cell. - It serves no biological function and is all just a remnant from evolution. - It is found at the ends of chromosomes from over replication during each cell cycle. - Portions of junk sequence are conserved between species and thus may be functional.

Portions of junk sequence are conserved between species and thus may be functional.

The control of a gene product's levels or activity after transcription has taken place is called: - Cell memory. - Post-transcriptional control. - Combinatorial control. - Epigenetic inheritance.

Post-transcriptional control.

Match each of the following events during Drosophila embryo development to the correct location in the embryo.

Posterior End: Pole cells begin migration from this location. Anterior End: Crest develops to separate a region that will become head, mouth parts, and foregut. Dorsal Side: Pole cells move inward as hindgut invaginates. Ventral Side: Mesodermal cells migrate inward during gatrulation.

Signaling at a synapse occurs in presynaptic and postsynaptic cells. Sort each of the following events into the proper location. - Neruotransmitter is released into the synaptic cleft. - Voltage-gated Ca2+ channels open. - Re-uptake of the neruotransmitter occurs. - Neurotransmitter binding begins a new action potential. - Ligand-gated ion channels open in response to neurotransmitter.

Presynaptic Cell: Neruotransmitter is released into the synaptic cleft. Voltage-gated Ca2+ channels open. Re-uptake of the neruotransmitter occurs. Postsynaptic Cell: Neurotransmitter binding begins a new action potential. Ligand-gated ion channels open in response to neurotransmitter.

Eukaryotic repressor proteins can decrease transcription using which of the following mechanisms? - Attracting a chromatin remodeling complex to open chromatin at the site. - Binding an operon and preventing polymerase binding. - Preventing the assembly of the transcription initiation complex. - Recruiting a histone acetyltransferase complex to modify histones.

Preventing the assembly of the transcription initiation complex.

Mutations that prevent Bcl2 family proteins Bax and Bak from interacting with the outer mitochondrial membrane would have which effect? - Preventing the release of cytochrome c and promoting the assembly of the apoptosome. - Triggering the release of cytochrome c and promoting apoptosis. - Preventing the release of cytochrome c and inhibiting apoptosis. - Triggering the release of cytochrome c and promoting the assembly of the apoptosome. - Preventing the release of cytochrome c and triggering the activation of procaspase-9

Preventing the release of cytochrome c and inhibiting apoptosis.

Cells export collagen to the extracellular matrix as which of the following? - Collagen fibrils, made of collagen molecules. - Collagen fibers, made of collagen fibrils. - A triple-stranded collagen molecule. - Procollagen, a precursor with additional peptide extensions at each end. - A single-stranded collagen polypeptide chain.

Procollagen, a precursor with additional peptide extensions at each end.

In the intestine, what do Wnt proteins promote? - Secretion of mucus at the base of each intestinal crypt. - Proliferation of the stem cells and precursor cells at the base of each intestinal crypt. - Apoptosis of the stem cells at the base of each intestinal crypt. - Apoptosis of precursor cells in each intestinal crypt.

Proliferation of the stem cells and precursor cells at the base of each intestinal crypt.

Which form of post-transcriptional control is being widely exploited by scientists to "knock down" genes of interest? - Histone deacetylation. - RNA interference. - DNA methylation. - Promoter blocking.

RNA interference.

Which of these methods can be used to shut down the activity of genes that encode components of a signaling pathway? - Introduction of constitutively active Ras. - Co-immunoprecipitation. - RNA interference. - Genetic screen. - Column chromatography.

RNA interference.

The splicing of introns out of an mRNA molecule is catalyzed by: - RNA molecules that act as a template for new RNA synthesis using exons only. - RNA molecules that base pair with the splice sites to promote intron removal. - Proteins that act as nucleases to chew away and remove the introns. - Proteins that contain metal ions to pull electrons from the phosphate bonds.

RNA molecules that base pair with the splice sites to promote intron removal.

Ras can exist in two different conformations or states, inactive and active. Which of the following correctly describe(s) the stably active state of Ras? (choose one or more). - Ras is bound to GDP. - Ras is bound to GTP. - Switch 1 and switch 2 regions are in an active conformation. - Ras is bound to Ras-GAP.

Ras is bound to GTP. Switch 1 and switch 2 regions are in an active conformation.

Which of the following mechanisms describes how eukaryotic activator proteins can regulate chromatin packaging to enhance transcription? - Activating polymerase binding through contacts made in the activation region. - Recruiting histone deacetylase complexes to remove acetyl groups from histones. - Binding to enhancer regions to promote formation of the transcription initiation complex. - Recruiting chromatin-remodeling complexes to eject or slide nearby nucleosomes.

Recruiting chromatin-remodeling complexes to eject or slide nearby nucleosomes.

Replication origins typically consist of a small stretch of duplex DNA that is relatively easy to pry apart. Which statement is true regarding replication origins? - Replication origins are rich in A and T nucleotides. - Replication origins are rich in G and T nucleotides. - Replication origins are rich in G and C nucleotides. - Replication origins contain multiple repeats of the nucleotide sequence TTAGGG. - Replication origins have equal numbers of A, C, G, and T nucleotides.

Replication origins are rich in A and T nucleotides.

The CRISPR-Cas9 system was discovered in bacteria, which use it as a defense mechanism against viral infections. Scientists have experimentally modified this system to work in other organisms. Which of the following statements correctly describe an application of the CRISPR system? (choose one or more). - Repression of transcription of a specific gene. - Activation of transcription of a specific gene. - Gene editing of a specific gene. - Editing of the mRNA of a specific gene.

Repression of transcription of a specific gene. Activation of transcription of a specific gene. Gene editing of a specific gene.

Which macromolecule(s) is/are critical in the active site of the ribosome for catalysis of peptide bond formation? - Ribosomal RNA and protein. - Ribosomal protein. - Ribosomal RNA, protein, and carbohydrate. - Ribosomal RNA. - Ribosomal carbohydrate.

Ribosomal RNA.

In a typical human secretory cell, which of the following membranes has the largest surface area? - Rough ER. - Plasma membrane. - Smooth ER. - Lysosome. - Nuclear inner membrane.

Rough ER.

Chemical modifications like phosphorylation and acetylation of proteins occur on ___________ of amino acids and can affect interaction of proteins with other cell components or structures. - Peptide groups. - Amino groups. - Carboxyl groups. - Side chains.

Side chains.

When cells respond to an extracellular signal, they most often convert the information carried by this molecule from one form to another. What is this process called? - Signal integration. - Signal amplification. - Signal transduction. - Signal detection. - Signal production.

Signal transduction.

Endocrine cells that synthesize lipid hormones, such as steroids, contain extensive amounts of which of the following? - Golgi apparatus. - Rough ER. - Nucleus. - Cytosol. - Smooth ER.

Smooth ER.

In muscle cells, which organelle sequesters Ca2+ from the cytosol? - Rough ER. - Smooth ER. - Mitochondrion. - Lysosome. - Endosome.

Smooth ER.

Which of the following is NOT a common source of DNA damage for cells in our bodies? - UV light. - Replication fork problems. - Spontaneous loss of amino groups on cytosine. - Soap.

Soap.

What is false regarding codons in mRNA molecules? - All codons contain three nucleotides. - Some codons code for more than one amino acid. - Codons in mRNAs bind to complementary anticodons in tRNAs. - In some cases, several different codons code for the same amino acid. - Some codons do not code for amino acids.

Some codons code for more than one amino acid.

In the technique called optogenetics, light-gated Na+ channels are introduced into the brains of living animals. Activation of these channels by light can depolarize the membranes of neurons that contain them, selectively activating these target cells. Since its inception, optogenetics has been expanded to include other types of light-gated channels, such as a channel that is selective for Cl-instead of Na+. If this light-gated Cl- channel were introduced into neurons in a region of the brain that stimulates feeding, what might you expect to see? - The animals would avoid eating, but only during the day. - The animals would avoid eating, even when they are hungry—but only when the channels are activated by light. - The animals would avoid eating, even when they are hungry. - The channels would have no effect on behavior because the animal's normal Na+ channels would allow normal depolarization of neurons that regulate feeding. - In response to light activation, the animals would overeat, even when they are full.

The animals would avoid eating, even when they are hungry—but only when the channels are activated by light.

Which part of a protein is synthesized by a ribosome first? - It depends on the protein. - It depends on where the ribosome binds to the mRNA. - The N-terminus. - The C-terminus. - It depends on whether the cell is eukaryotic or prokaryotic.

The N-terminus.

During the activation of a neuron, the action potential propagates in only one direction. How is this achieved in the neuron? - The Na+ channel remains open during the action potential and then rapidly returns to the closed state after the action potential passes. - The Na+ channel closes during the action potential and then rapidly returns to the open state after the action potential passes. - The Na+ channel becomes inactivated and refractory to reopening for a short time after the action potential passes. - The Na+ channel becomes permanently inactivated after the action potential passes.

The Na+ channel becomes inactivated and refractory to reopening for a short time after the action potential passes.

What makes it possible for a combination of signal molecules to evoke a response that differs from the sum of the effects that each signal could trigger on its own? - The ability of different receptors to activate different intracellular relay systems. - The ability of different signal molecules to bind to the same receptor. - The ability of different intracellular relay systems to interact. - The tendency of different cells to display different receptors. - The ability of the same signal molecule to bind to different receptors.

The ability of different intracellular relay systems to interact.

For which reason are α helices and β sheets common folding patterns in polypeptides? - Any amino acid side chain can contribute to their formation. - They allow proteins to interact favorably with many biological molecules. - The amino acid side chains are not directly involved in their formation. - Molecular chaperones tend to fold polypeptides into these common folding patterns.

The amino acid side chains are not directly involved in their formation.

Which of the following is not true of the basal lamina? - The basal lamina contains a specialized type of collagen type IV. - The basal lamina supports epithelial cells by interacting with integrins in the basal plasma membrane. - The basal lamina lines the cytosolic side of the epithelial cell's basal plasma membrane. - The basal lamina is a thin, tough sheet of extracellular matrix.

The basal lamina lines the cytosolic side of the epithelial cell's basal plasma membrane.

The energy for the polymerization reaction in DNA synthesis is powered by: - The breaking of the hydrogen bonds between complementary base pairs. - The formation of the phosphodiester bonds between nucleotides. - The association/disassociation cycles of the DNA polymerase enzyme. - The breaking of high-energy phosphate bonds in the deoxynucleotides.

The breaking of high-energy phosphate bonds in the deoxynucleotides.

When a vesicle fuses with the plasma membrane, which way will the monolayer that was exposed to the interior of the vesicle face? - The endomembrane system. - The direction the monolayer will face will be established randomly. - It depends on where, along the plasma membrane, the vesicle fuses. - The cell cytoplasm. - The cell exterior.

The cell exterior.

Which of the following structural changes is not typically seen in a cell that is undergoing apoptosis? - The cytoskeleton collapses. - The nuclear envelope disassembles. - The cell develops irregular bulges. - The cell swells.

The cell swells.

Which of the following characteristics of aquaporins ensure that the channel selectively transports only water molecules and not other solutes? (choose one or more). - The channel has a narrow pore that is only wide enough for a single water molecule to pass through. - The channel undergoes conformational changes to push water through the channel. - Two asparagines in the center of the pore prevent protons from passing through the channel. - A glutamate at the entrance to the channel prevents positive ions from entering the channel.

The channel has a narrow pore that is only wide enough for a single water molecule to pass through. Two asparagines in the center of the pore prevent protons from passing through the channel.

When transmitter-gated ion channels in the membrane of a postsynaptic cell open in response to neurotransmitter binding, what happens? - The channels convert an electrical signal to a chemical signal. - The channels remain open until an inhibitory neurotransmitter triggers their closure. - The channels always trigger an immediate and sustained action potential. - The channels alter the ion permeability of the postsynaptic membrane, which in turn may depolarize the postsynaptic membrane.

The channels alter the ion permeability of the postsynaptic membrane, which in turn may depolarize the postsynaptic membrane.

If a chromosome is attached to two microtubules from opposite poles and one of the microtubules is experimentally severed, what occurs? - The chromosome remains aligned along the equator of the metaphase spindle. - The severed microtubule immediately regenerates and reattaches. - The chromosome migrates quickly to the pole to which it is still attached. - The remaining microtubule releases the chromosome contacts.

The chromosome migrates quickly to the pole to which it is still attached.

When glucose moves across a phospholipid bilayer by passive transport, which factor determines the direction of its transport? - Whether the cell is metabolically active or not. - The charge difference across the membrane. - The concentrations of glucose on either side of the membrane. - The amount of energy available to fuel the transport process.

The concentrations of glucose on either side of the membrane.

What does the term "genetic code" refer to? - The set of genes encoded in an organism's DNA. - The fact that, for all living things on Earth, genetic information is encoded in DNA. - The correspondence between the four-letter nucleotide alphabet of DNA and the 20-letter amino acid alphabet of proteins. - The principle that genetic information flows from DNA to RNA to protein. - The linear sequence of nucleotides in a gene.

The correspondence between the four-letter nucleotide alphabet of DNA and the 20-letter amino acid alphabet of proteins.

Consider the process that a cell uses to replicate its double-stranded DNA before undergoing cell division. Which statement describes the DNA in the resulting daughter cells? - The double helix in one daughter cell consists of two strands that were originally in the parent cell, while the double helix in the other daughter cell consists of two newly made strands. - The double helix in each daughter cell consists of one parental strand and one newly synthesized strand. - Each daughter cell receives a random mix of parental chromosomes and new chromosomes. - The daughter cells receive only newly synthesized DNA; the parent cell keeps the original DNA. - Each strand of DNA in the daughter cells contains a mix of both newly replicated and parental DNA.

The double helix in each daughter cell consists of one parental strand and one newly synthesized strand.

What determines the position of the cleavage furrow of the dividing cell? - The cleavage furrow position is determined randomly. - Astral microtubules contact the membrane and activate proteins to form a central furrow. - The two spindle poles send signals to the plasma membrane so that the cleavage furrow forms in the same plane as the two poles. - The interpolar microtubules send signals to form a cleavage furrow between the poles.

The interpolar microtubules send signals to form a cleavage furrow between the poles.

An experiment was performed to determine the role that ATP plays in kinesin movement along microtubules. Kinesin and microtubules were incubated together in a test tube, but instead of ATP, a non-hydrolyzable analog of ATP was added to the tube. What impact on kinesin function do you expect to observe in the presence of this ATP analog? - The kinesin would bind tightly to microtubules and not release. - The kinesin would be unable to bind to microtubules since the kinesin would remain inactive. - The leading head would bind tightly, but the trailing head would remain free. - The kinesin would walk faster along the microtubule since the kinesin would remain active while bound to ATP.

The kinesin would bind tightly to microtubules and not release.

What problem with replication of linear chromosomes does telomerase address? - The lagging strand stops short of the 3' end during replication, so chromosomes would shorten in each replication cycle without telomerase. - The leading strand stops short of the 5' end during replication, so chromosomes would shorten in each replication cycle without telomerase. - The lagging strand continues farther during replication, so chromosomes would lengthen in each replication cycle without telomerase. - The leading strand continues farther during replication, so chromosomes would lengthen in each replication cycle without telomerase.

The lagging strand stops short of the 3' end during replication, so chromosomes would shorten in each replication cycle without telomerase.

Which does not contain both actin and myosin? - The contractile ring that carries out cytokinesis. - A muscle cell sarcomere. - A contractile bundle in a nonmuscle cell. - The lamellipodium at the leading edge of a crawling cell.

The lamellipodium at the leading edge of a crawling cell.

Ca2+ channel blockers bind to Ca2+ channels and prevent the movement of Ca2+ through the channel. What would happen if a Ca2+ channel blocker were added to the leaf and hinge cells of the mimosa plant? - The leaflets would stay closed for a much longer period of time. - The leaflets would not close when touched. - Only one leaf would move when touched. - The leaflets would close without being touched.

The leaflets would not close when touched.

What does the primary structure of a protein refer to? - The locations of the protein's alpha-helices and beta-sheets. - The locations of the peptide bonds that form the protein's backbone. - The overall three-dimensional shape of the protein. - The structure that forms first as the protein folds into its most stable form. - The linear amino acid sequence of the protein.

The linear amino acid sequence of the protein.

When Griffith injected heat-killed infectious bacteria mixed with live harmless bacteria, he found that the mice died because: - The heat-killed infectious bacteria were able to kill the mice on their own. - The heat-killed infectious bacteria were not actually dead. - The live harmless bacteria were transformed into infectious bacteria. - The live harmless bacteria were able to kill the mice on their own.

The live harmless bacteria were transformed into infectious bacteria.

What does depurination refer to? - The loss of G or C bases from DNA. - The loss of A or G bases from DNA. - The breaking of the DNA backbone. - The accumulation of mutations and subsequent loss of purity of a nucleotide sequence. - Ehe loss of thymine due to damage from UV radiation.

The loss of A or G bases from DNA.

Which would be more deleterious: the loss of a single nucleotide from the protein-coding region of a gene or the loss of three nucleotides in that same region? - The loss of three nucleotides. - The loss of a single nucleotide. - As both deletions are relatively small, neither would likely have any seriously deleterious effects on the activity of the encoded protein.

The loss of a single nucleotide.

What is the ultimate fate of an mRNA that is targeted by a microRNA (miRNA)? - The mRNA will be destroyed by nucleases. - The mRNA will be destroyed by the proteasome. - The mRNA will be transported to the nucleus. - The mRNA will be secreted from the cell. - The mRNA will be translated more efficiently by ribosomes.

The mRNA will be destroyed by nucleases.

Tetrodotoxin is a potent toxin found in a variety of organisms including the pufferfish. The toxin binds to the extracellular side of the Na+ channel and prevents channel opening. This leads to paralysis of muscles, including the diaphragm. Death from respiratory failure can occur after ingestion of as little as 1 mg of the toxin. Why does this toxin cause paralysis? - The Na+ channel does not open wide enough to allow enough Na+ through the channel. - The axon membranes become over-depolarized. - The Na+ channels remain in the inactive, refractory state. - The membrane depolarization is not amplified along the axon.

The membrane depolarization is not amplified along the axon.

If GTP hydrolysis occurs on a tubulin molecule at the plus end of a microtubule protofilament before another tubulin molecule is added, what typically happens? - The GDP is rapidly exchanged for a fresh molecule of GTP. - The microtubule remains the same size. - The microtubule polymerizes. - The microtubule depolymerizes.

The microtubule depolymerizes.

The concentration of actin monomers is high in the cytosol. What keeps these monomers from polymerizing totally into filaments? - The concentration is high, but not high enough for polymerization to occur. - Not enough of the monomers are bound to ATP. - The monomers are bound by proteins that prevent their polymerization.

The monomers are bound by proteins that prevent their polymerization.

In the eukaryotic flagellum, what drives the bending of microtubules? - The fluid that surrounds the flagellum. - Ciliary actin and myosin. - The basal body. - The motor protein ciliary dynein.

The motor protein ciliary dynein.

Which of the following correctly describes osmosis? - The movement of water from an area of low water concentration to an area of high water concentration. - The movement of water from an area of low solvent concentration to an area of high solvent concentration. - The movement of water from an area of low solute concentration to an area of high solute concentration. - The movement of water from an area of high solute concentration to an area of low solute concentration.

The movement of water from an area of low solute concentration to an area of high solute concentration.

Which membrane-enclosed organelles most likely evolved in a similar manner? - Mitochondria and the nucleus. - Chloroplasts and peroxisomes. - Mitochondria and the Golgi apparatus. - Mitochondria and the ER. - The nucleus and the ER.

The nucleus and the ER.

You are a biologist interested in studying how eukaryotic cells obtained their unique features during their evolution from a prokaryotic ancestor. Which cellular features, unique to eukaryotes, might you focus on? (choose one or more). - Ribosomes. - The nucleus. - The cell membrane. - Mitochondria.

The nucleus. Mitochondria.

The energy to generate ATP in step 7 of glycolysis comes from which? - The consumption of NADH. - The donation of a phosphate from glyceraldehyde 3-phosphate. - The oxidation of glyceraldehyde 3-phosphate. - The donation of a phosphate from GTP. - The donation of a phosphate from 3-phosphoglycerate.

The oxidation of glyceraldehyde 3-phosphate.

Protons are pumped across the mitochondrial inner membrane as electrons are transferred through the mitochondrial electron transport chain. Which of the following statements about proton pumping are correct? (choose one or more). - Protons are pumped into the matrix of the mitochondria. - The pH inside the mitochondrial matrix is higher than in the intermembrane space. - The mitochondria use the proton gradient to synthesize ATP. - The NADH dehydrogenase, cytochrome b-c1, and cytochrome oxidase complexes all pump protons across the membrane.

The pH inside the mitochondrial matrix is higher than in the intermembrane space. The mitochondria use the proton gradient to synthesize ATP. The NADH dehydrogenase, cytochrome b-c1, and cytochrome oxidase complexes all pump protons across the membrane.

Which of the following statements is false? - The pairing of homologous chromosomes during prometaphase is called bi-orientation. - Each sister chromatid in a pair must attach to the opposite pole of the spindle for the sister chromatids to be segregated properly. - The cell-cycle control system monitors the tension on the kinetochores to ensure that anaphase does not occur until the sister chromatids are correctly attached to the spindle. - Attachment of sister chromatid pairs to opposite poles of the spindle creates tension in the kinetochores, which signals that the sister chromatids are ready to be separated.

The pairing of homologous chromosomes during prometaphase is called bi-orientation.

How does phosphorylation control protein activity? - The phosphate group induces a change in the protein's conformation. - The phosphate group, with its negative charges, prevents other negatively charged molecules from interacting with the protein. - The phosphate group serves as an added source of energy for a protein. - The phosphate group alters the primary structure of the protein. - The phosphate group, with its positive charges, temporarily relieves feedback inhibition.

The phosphate group induces a change in the protein's conformation.

ATP is important for chaperone protein function. Why would protein import into mitochondria be disrupted if ATP were depleted from inside mitochondria? - The protein could slip back out of the mitochondria during transport. - The signal sequence would not be recognized on the mitochondrial protein. - The protein would be blocked from entering the translocation apparatus. - The translocation apparatus would be unable to function without ATP hydrolysis.

The protein could slip back out of the mitochondria during transport.

Ricin is one of the most powerful toxins known. The protein consists of two subunits: the A chain is an enzyme that inhibits translation and the B chain is a lectin that binds to carbohydrates on the cell surface. What is the most likely mechanism by which ricin enters the cell? - The protein enters through pore complexes in the plasma membrane. - The A chain binds to clathrin.The A chain stimulates autophagy. - The B chain interacts with SNAREs. - The protein is internalized by endocytosis.

The protein is internalized by endocytosis.

Investigators have engineered a gene that encodes a protein bearing an ER signal sequence followed by a nuclear localization signal. What would be the likely fate of that protein? - The protein will be recognized by an SRP and enter the ER. - Because of its conflicting signals, the protein will remain in the cytosol. - The protein will be recognized by a nuclear import receptor and escorted into the nucleus. - Because of its conflicting signals, the protein will be sent to a lysosome for destruction. - Because of its conflicting signals, the protein will be degraded in the cytosol.

The protein will be recognized by an SRP and enter the ER.

How do chromatin-remodeling complexes work? - They use the energy from GTP hydrolysis to alter the arrangement of nucleosomes, rendering certain regions of the DNA more accessible to other proteins. - They remove acetyl groups from the tails of histones, rendering the DNA more accessible to other proteins. - They add methyl groups to the tails of histones in order to attract other proteins. - They use the energy from ATP hydrolysis to alter the arrangement of nucleosomes, rendering certain regions of the DNA more accessible to other proteins. - They bind to nucleosomes in the 30-nm fiber and induce another level of packing, obscuring DNA from binding by other proteins.

They use the energy from ATP hydrolysis to alter the arrangement of nucleosomes, rendering certain regions of the DNA more accessible to other proteins.

What is the molecular target of the antidepressant Prozac? - The Na+ channels that activate the neuron. - The serotonin receptor, a transmitter-gated ion channel. - The K+ channels that deactivate action potentials. - The symport that drives reuptake of serotonin.

The symport that drives reuptake of serotonin.

What does the term "gluconeogenesis" refer to? - The transport of glucose across a cell membrane. - The breakdown of glucose during glycolysis. - The release of glucose from molecules such as glycogen. - The breakdown of glucose during fermentation. - The synthesis of glucose from small organic molecules such as pyruvate.

The synthesis of glucose from small organic molecules such as pyruvate.

Which is true of the Notch signaling pathway? - Notch is an intracellular receptor that travels to the nucleus where it regulates gene transcription. - The Notch receptor activates gene expression by triggering the production of cyclic AMP. - The Notch receptor is located in the nuclear membrane. - The Notch receptor is activated by cyclic AMP. - The tail of the cell-surface receptor travels to the nucleus where it regulates gene transcription.

The tail of the cell-surface receptor travels to the nucleus where it regulates gene transcription.

Diseases that disrupt the function of mitochondria are particularly harmful to muscle and nerve cells for what reason? - These cells lack mitochondria. - These cells need large amounts of ATP to function normally. - These cells cannot produce energy by fermentation. - These cells can produce energy by fermentation. - These cells undergo rapid cycles of cell division to function.

These cells need large amounts of ATP to function normally.

Which of the following is inconsistent with the function of gap junctions? - They allow the cytoplasm of two adjacent cells to be continuous with each other. - They allow an electrical and a metabolic coupling between attached cells. - They can open or close as needed. - They allow inorganic ions to move directly between attached cells. - They allow ATP-driven pumps to move substances between attached cells.

They allow ATP-driven pumps to move substances between attached cells.

Epithelial cells line body cavities and, in organs like the intestine, separate the extracellular environment (intestinal lumen) from the body. The barrier function of epithelial tissues is important for maintaining homeostasis and preventing infections from gut-associated microbiota and viruses. Some bacteria and viruses compromise this barrier function, aiding their pathogenesis. Which of the following structures, most critical to the barrier function of epithelial tissue, is targeted by these infectious agents? - Desmosomes. - Hemidesmosomes. - Tight junctions. - Gap junctions.

Tight junctions.

What is a function of the nucleolus? - To enclose sex chromosomes into a membrane-bound organelle. - To form the nuclear lamina. - To assemble ribosomal RNA and proteins into ribosomes. - To assist in the segregation of chromosomes during mitosis.

To assemble ribosomal RNA and proteins into ribosomes.

What is the main function of the TATA-binding protein? - To help initiate DNA replication. - To inhibit DNA replication until S phase - To promote initiation of transcription. - To inhibit transcription until transcriptional activators bind.

To promote initiation of transcription.

Although all of the steps involved in expressing a gene can in principle be regulated, what is the most important stage of control for most genes? - RNA processing. - RNA transport and localization. - Transcription initiation. - mRNA translation. - mRNA degradation.

Transcription initiation.

All of the RNA in a cell is made by what process? - Transcription. - Replication. - Recombination. - Translation.

Transcription.

The process of gene expression always involves which process(es) described in the central dogma? - Replication. - Transcription. - Transcription and translation. - Translation.

Transcription.

What would be the effect on cell function of inhibiting phosphorylation of eIF2? - Translation would not be inhibited in response to cell stress. - The chaperones would be activated in response to cell stress. - More proteins would be synthesized in order to respond to cell stress. - The ribosome would be less active.

Translation would not be inhibited in response to cell stress.

The anaphase-promoting complex (APC) triggers the onset of anaphase by doing which of the following? - Phosphorylating the motor proteins that pull the spindle poles apart. - Cleaving spindle microtubules. - Triggering the destruction of the cohesins that hold the sister chromatids together. - Triggering the destruction of the condensins that hold the duplicated chromosomes in a condensed state.

Triggering the destruction of the cohesins that hold the sister chromatids together.

A single DNA strand can serve as a template for both leading and lagging DNA strands during the synthesis of double-stranded DNA. Is this statement true or false? - True, but only for prokaryotic DNA replication. - Completely false regardless of DNA type. - True for all types of DNA replication. - True, but only for eukaryotic DNA replication

True for all types of DNA replication.

At a molecular level, the members of the two domains of prokaryotes—the archaea and bacteria—differ as much from each other as either does from the eukaryotes. - True. - False. - Impossible to determine.

True.

Determine whether the following statement is true or false: During muscle contraction, the myosin and actin filaments do not change in length. This statement is:

True.

Determine whether the following statement is true or false: In the presence of severe DNA damage, the transcription regulator p53 can promote cell death. This statement is:

True.

Determine whether the following statement is true or false: Key components of the cytoskeleton are important in both mitosis and cytokinesis. This statement is:

True.

Determine whether the following statement is true or false: Myosins are motor proteins that use the energy of ATP hydrolysis to move along actin filaments. This statement is:

True.

Determine whether the following statement is true or false: RNAi is a form of reverse genetics. This statement is:

True.

Determine whether the following statement is true or false: The majority of human DNA does not code for genes. This statement is:

True.

Determine whether the following statement is true or false: Transmitter-gated ion channels are insensitive to membrane potential; in the absence of neurotransmitters, they cannot generate an action potential. This statement is:

True.

Determine whether the following statement is true or false: Using just three or four genes, scientists can transform adult cells into cells that behave like embryonic stem cells. This statement is:

True.

Determine whether the following statements are true or false: "At an origin of replication, hydrogen-bonding between both strands of the double helix must denature/separate." This statement is:

True.

Many protein molecules (not just enzymes) are allosteric. - True. - False. - Impossible to determine.

True.

What is the driving force for plant cell growth? - Mitotic division. - Photosynthesis. - Excess sunlight. - Turgor pressure. - Meiotic division.

Turgor pressure.

The function of feedback inhibition of an enzymatic pathway is to: -Accumulate large amounts of important biological molecules. - Increase concentrations of intermediates to drive the reaction forward. - Turn off synthesis of a product when it is in abundance. - Irreversibly shut down a biosynthetic pathway.

Turn off synthesis of a product when it is in abundance.

Certain mutant Ras proteins cannot hydrolyze their bound GTP to GDP, and therefore cannot do which of the following? - Turn themselves off. - Turn themselves on. - Be degraded. - Activate MAP kinase directly. - Promote cell proliferation.

Turn themselves off.

What enables proteins destined for nuclear import to pass through the nuclear pore? - They are recognized by receptors that interact with repeated amino acid sequences in proteins lining the nuclear pore. - They promote the hydrolysis of GTP, which provides the energy needed for their transport. - Their nuclear localization signal interacts with the unstructured meshwork of proteins lining the pore. - They are unfolded to allow them to snake through the meshlike network of proteins lining the nuclear pore. - Their nuclear localization signal interacts with the cytosolic fibrils that extend from the pore.

They are recognized by receptors that interact with repeated amino acid sequences in proteins lining the nuclear pore.

Most extracellular signal molecules act on cell-surface receptors rather than intracellular receptors. Which statements are true about these extracellular molecules? (choose one or more). - They are too hydrophobic to pass directly across the plasma membrane. - They are too small to pass directly across the plasma membrane. - They are too hydrophilic to pass directly across the plasma membrane. - They are too large to pass directly across the plasma membrane.

They are too hydrophilic to pass directly across the plasma membrane. They are too large to pass directly across the plasma membrane.

How can methods such as RNA interference or CRISPR be used to assess the role of a protein in cell signaling? - They can be used to produce intracellular second messenger molecules. - They can be used to isolate the proteins involved in a signaling pathway. - They can be used to shut down genes encoding components of a signaling pathway. - They can be used to introduce into cells the genes encoding components of a signaling pathway. - They can be used to generate constitutively active versions of signaling proteins.

They can be used to shut down genes encoding components of a signaling pathway.

Which statement is true about stem cells? - They are terminally differentiated. - They are usually present in large numbers, especially in tissues that undergo rapid renewal. - They can divide for the lifetime of the organism. - They always divide asymmetrically, giving rise to two different daughter cells.

They can divide for the lifetime of the organism.

How are proteins destined to function in the ER retained there? - They are anchored to dolichol in the ER membrane. - They are embedded in the ER membrane by a transmembrane α helix. - They retain their N-terminal ER signal sequence. - They bind to chaperones within the ER. - They contain a C-terminal ER retention signal.

They contain a C-terminal ER retention signal.

How do specialized brown fat cells take advantage of oxidative phosphorylation to generate heat? - They contain a carrier protein that dissipates the proton gradient across the inner mitochondrial membrane. - They have the ability to carry out photosynthesis. - They contain a carrier protein that produces a larger proton gradient across the inner mitochondrial membrane. - They produce and consume larger than normal amounts of ATP. - They oxidize only sugars, and store all their fat.

They contain a carrier protein that dissipates the proton gradient across the inner mitochondrial membrane.

Which of the following statements is not true about the differences between liver cells and kidney cells in the same organism? - They express different genes. - They contain different genes. - They contain different proteins. - They contain the entire set of instructions needed to form the whole organism.

They contain different genes.

Ras is a GTP-binding protein involved in cell proliferation (division). In its active form, with GTP bound, Ras activates cell signaling pathways that promote cell division. Mutations in the gene that encodes Ras can lead to cancer. How might mutations in the gene encoding Ras lead to the uncontrolled proliferation characteristic of cancer cells? - They increase the rate at which Ras hydrolyzes GTP. - They increase the protein's affinity for GDP. - They prevent Ras from being made. - They decrease the protein's affinity for GTP. - They decrease the rate at which Ras hydrolyzes GTP.

They decrease the rate at which Ras hydrolyzes GTP.

What is the role of K+-gated ion channels in an action potential? - They provide the energy for the sodium-potassium pump to reestablish resting potential. - They help reverse the action potential by repolarizing the cell. - They do not have a role in action potentials. - They lead to the action potential reaching its highest state of cell depolarization.

They help reverse the action potential by repolarizing the cell.

Which is true of cadherin proteins? - They allow ions and small molecules to pass from one plant cell to another. - They anchor epithelial cells to the basal lamina. - They link epithelial cells together by binding to similar cadherins in adjacent epithelial cells. - They create an electrical and metabolic coupling between epithelial cells.

They link epithelial cells together by binding to similar cadherins in adjacent epithelial cells.

The Encyclopedia of DNA Elements (ENCODE) project aims to discover functional elements in DNA. One technique employed is DNase-seq. This technique employs a DNase enzyme that digests accessible regions of chromatin, while inaccessible regions remain undigested. The undigested DNA is then sequenced. Which of the following is likely true of the DNase-seq results? (choose one or more). - The sequenced DNA will be the same from cell type to cell type. - Unsequenced DNA is likely part of euchromatin. - Unsequenced DNA regions were likely bound to nucleosomes. - Centromeric DNA is likely to be sequenced in all samples.

Unsequenced DNA is likely part of euchromatin. Centromeric DNA is likely to be sequenced in all samples.

In eukaryotes, where do transcription regulators bind? - Only the transcription start site. - Only downstream from genes they control. - Only the TATA box. - Only upstream from genes they control. - Upstream, downstream, or within the genes they control.

Upstream, downstream, or within the genes they control.

Solve the riddle by determining which base could be speaking. "I'm a nitrogen-containing base, just like the other nitrogenous bases. However, they don't let me into their exclusive 'DNA base club.' Who am I?" - Adenine. - Cytosine. - Guanine. - Uracil. - Thymine.

Uracil.

Which is a mechanism for restricting the movement of proteins in the plasma membrane? (choose one or more). - Coating proteins with carbohydrates. - Using barriers such as tight junctions. - Tethering proteins to the cell cortex. - Forming a covalent linkage with membrane lipids. - Tethering proteins to the surface of another cell. - Tethering proteins to the extracellular matrix.

Using barriers such as tight junctions. Tethering proteins to the cell cortex. Tethering proteins to the surface of another cell. Tethering proteins to the extracellular matrix.

Scientists have modified a clathrin molecule so that it still assembles but forms an open-ended lattice instead of a closed spherical cage. How would this clathrin molecule affect endocytosis in cells? - Endocytosis would be unaffected, since adaptors and receptors can still interact. - All movement of molecules into and out of the cell would cease. - Vesicles would be larger, increasing the cargo endocytosed. - Vesicles cannot form properly without a clathrin cage, thus inhibiting endocytosis.

Vesicles cannot form properly without a clathrin cage, thus inhibiting endocytosis.

How are newly made lipids supplied to the plasma membrane? - Via vesicles that bud from the ER and fuse with the plasma membrane. - Via lysosomesvia secretory vesicles produced by the regulated exocytosis pathway. - Via the constitutive pathway of exocytosis. - Via enzymes that synthesize phospholipids, which are attached to the plasma membrane.

Via the constitutive pathway of exocytosis.

Following an action potential, a nerve cell goes through a brief refractory period during which it cannot be stimulated. What is true during this refractory period? - Voltage-gated K+ channels in the nerve cell membrane are inactivated. - Voltage-gated Na+ channels in the nerve cell membrane are open. - Voltage-gated Ca2+ channels are open. - Voltage-gated Na+ channels in the nerve cell membrane are inactivated. - The membrane potential remains unchanged.

Voltage-gated Na+ channels in the nerve cell membrane are inactivated.

Which type of ion channel plays the major role in propagating electrical signals in nerve cells? - Voltage-gated. - Mechanically-gated. - Ligand-gated.

Voltage-gated.

Which molecule is displaced when a vesicle and its target membrane fuse? - Clathrin. - Tethering proteins. - SNAREs. - Phospholipids. - Water.

Water.

How are voltage-gated ion channels opened by voltage sensors? - When membrane potential changes sufficiently, the electrical force causes voltage sensor domains to change conformation. - Ion binding to voltage sensors causes the channel pore to widen and open. - Voltage sensors on the channel change their amino acid side chains from positively to negatively charged. - Changes in membrane potential lead to increased gene expression of voltage-gated ion channel proteins.

When membrane potential changes sufficiently, the electrical force causes voltage sensor domains to change conformation.

The transcription initiation site of a eukaryotic gene is found at which location? - Where RNA polymerase first binds. - Where transcription regulators bind. - Where general transcription factors bind. - Where RNA synthesis begins.

Where RNA synthesis begins.

How do calcium ions stimulate contraction in nonmuscle cells and smooth muscle cells? - Calcium ions bind to troponin and lead to a conformational change in tropomyosin that exposes the myosin binding sites on the actin filament. - Calcium ions released into the cell promote the release of secretory vesicles that send neurotransmitters to neighboring cells to stimulate an action potential. - Calcium ions lead to activation of a kinase that phosphorylates nonmuscle myosin to alter its conformation and enable actin binding. - Calcium ions trigger an action potential in the cell, leading to gene expression changes that influence contraction.

Calcium ions bind to troponin and lead to a conformational change in tropomyosin that exposes the myosin binding sites on the actin filament.

Which of the following statements is true about the concentration of calcium ions in cells? - Calcium levels are kept the same in the cytosol compared to outside the cell. - Calcium levels are kept low in the cytosol compared to outside the cell. - Calcium levels are kept high in the cytosol compared to outside the cell.

Calcium levels are kept low in the cytosol compared to outside the cell.

Single nucleotide changes from unrepaired DNA damage that lead to uncontrolled cell division can lead to which of the following conditions? - Aging. - Sickle cell anemia. - Cancer. - Cell Death.

Cancer.

Which of the following characteristics of K+ channels are important for the selectivity for K+ rather than other ions? (choose one or more). - Acidic side chains line the wall of the pore. - Carbonyl groups line the wall of the pore. - Basic side chains line the wall of the pore. - Four rigid protein loops line the narrowest part of the pore.

Carbonyl groups line the wall of the pore. Four rigid protein loops line the narrowest part of the pore.

Which of the following correctly describe steps required for protein transport into the rough ER? (choose one or more). - Cleavage of the signal sequence from the protein by signal peptidase. - Recognition and binding of the protein signal sequence by SRP. - Passing of the protein to a protein translocation channel in the ER membrane. - Initial transfer of the signal sequence to the inside of the rough ER.

Cleavage of the signal sequence from the protein by signal peptidase. Recognition and binding of the protein signal sequence by SRP. Passing of the protein to a protein translocation channel in the ER membrane.

In animal connective tissues, tensile strength is chiefly provided by which one of the following? - Cellulose microfibrils. - Actin filaments. - Collagen fibrils and fibers. - The cells themselves. - Polysaccharides.

Collagen fibrils and fibers.

For both clinical and cosmetic reasons, plastic surgeons inject substances into connective tissue underlying the skin epidermis. This plumps up areas deficient in soft tissue, and is used to reduce surgical scars or wrinkles, for example. Which normal connective tissue components are good candidates to be injected as fillers in this type of procedure? (choose one or more). - Hemidesmosome. - Collagen. - Glycosaminoglycan. - Cadherin.

Collagen. Glycosaminoglycan.

Many transcriptional regulators function together to decide the expression level of a particular gene. This describes the concept of: - Activation. - Operons. - Combinatorial control. - Repression.

Combinatorial control.

What is true of a newly synthesized DNA strand of a human chromosome? - It is produced by a mixture of continuous and discontinuous DNA synthesis from a single replication origin. - It is produced by a mixture of continuous and discontinuous DNA synthesis from multiple replication origins. - It is produced by either continuous or discontinuous DNA synthesis from multiple replication origins, depending on whether the leading or lagging strand is examined. - It is produced by continuous DNA synthesis from a single replication origin. - It is produced by discontinuous DNA synthesis from multiple replication origins. - It is produced by discontinuous DNA synthesis from a single replication origin. - It is produced by continuous DNA synthesis from multiple replication origins.

It is produced by a mixture of continuous and discontinuous DNA synthesis from multiple replication origins.

Glycolysis occurs in a series of 10 steps using 10 different enzymes. Place the following broad events that occur during glycolysis into their correct order.

Energy is invested by using up some ATP. The six-carbon sugar is split into two similar molecules. Energy is captured as ATP and NADH. Two molecules of pyruvate are generated.

In an alpha-helix, hydrogen bonds form between which of the following? - Every other amino acid. - Non-polar amino acid side chains. - Acidic and basic amino acid side chains. - The peptide bonds and DNA. - Every fourth amino acid.

Every fourth amino acid.

Which of the following does not cause a mutation? - Failure of DNA repair systems. - Metabolic activity. - UV radiation. - Evolution. - Replication errors.

Evolution.

What are protein families? - Sets of proteins with the same functions that are commonly found only in organisms of the same species. - Evolutionarily related proteins that are similar in amino acid sequence and three-dimensional conformation. - Proteins found specifically in one type of cell line that tend to interact with each other. - Proteins that are produced at the same time on the ribosome that, once produced, regulate the activity of each other.

Evolutionarily related proteins that are similar in amino acid sequence and three-dimensional conformation.

Ionizing radiation can also cause damage to the nitrogenous bases of DNA. Repair of the damaged bases takes several steps. Which of the following repair mechanisms can be used to repair the nitrogenous bases of DNA damaged by ionizing radiation? - Nonhomologous end joining. - Direct repair. - Excision repair. - Mismatch repair.

Excision repair.

Which of these is not a type of noncoding RNA? - mRNA. - rRNA. - miRNA. - tRNA.

mRNA.

Which type of RNA is converted into protein for performing its cellular function? - mRNA. - tRNA. - miRNA. - rRNA.

mRNA.

Which of the following statements correctly describe an aspect of converting light to chemical energy in chloroplasts? (choose one or more). - Light excites electrons in photosystem II only. - Excited electrons are passed through an electron transport chain. - The energy in excited electrons is used to pump protons across the thylakoid membrane from the stroma to the thylakoid space. - Photosystem II is used to produce NADPH directly from NADP+ and two electrons.

Excited electrons are passed through an electron transport chain. The energy in excited electrons is used to pump protons across the thylakoid membrane from the stroma to the thylakoid space.

Which of the following is a difference between exocytic and endocytic pathways? - Endocytic pathways bring in soluble proteins only, whereas exocytic pathways move both soluble proteins and membrane-bound proteins. - Exocytic pathways utilize transport vesicles, whereas endocytic pathways use only endosomes. - Endocytic pathways transfer proteins to the Golgi apparatus, whereas exocytic pathways transfer proteins from the Golgi. - Exocytic pathways often start with synthesis of proteins, whereas endocytic pathways involve breaking down macromolecules like proteins.

Exocytic pathways often start with synthesis of proteins, whereas endocytic pathways involve breaking down macromolecules like proteins.

In a globular protein, where would the amino acid arginine most likely be found? - Located at the protein's N-terminus. - Exposed at the protein's surface. - Covalently bonded to the cysteine side chain. - Located at the protein's C-terminus - Buried in the protein's interior.

Exposed at the protein's surface.

Determine whether the following statement is true or false: The general transcription factors that assemble at a eukaryotic promoter are different, depending on the specific gene being transcribed by polymerase II. This statement is:

False

Determine whether the following statement is true or false and why: Homologous recombination occurs only between DNA molecules that are identical in nucleotide sequence without any variation. - True, but only for cells that have more than 10 chromosomes. - False, because only similar sequence is needed. - True, for all eukaryotic diploid cells. - False, because crossing-over does not happen.

False, because only similar sequence is needed.

Determine whether the following statement is true or false: A stem cell can only produce a single type of differentiated cell. This statement is:

False.

Determine whether the following statement is true or false: A symport protein would function as an antiport protein if its orientation in the membrane were reversed. This statement is:

False.

Determine whether the following statement is true or false: All proteins are fully functional upon leaving the ribosome. This statement is:

False.

Determine whether the following statement is true or false: All three major subunits of cytoskeletal filaments can bind to and hydrolyze nucleotide triphosphates. This statement is:

False.

Determine whether the following statement is true or false: An electrical signal can jump across the synaptic cleft between the presynaptic and postsynaptic cells. This statement is:

False.

Determine whether the following statement is true or false: Apoptosis is a form of programmed cell death in which cells kill themselves by migrating to the bloodstream where they are carried to the liver for destruction. This statement is:

False.

Determine whether the following statement is true or false: Different cell types that respond to the same hormone usually turn on the same sets of genes. This statement is:

False.

Determine whether the following statement is true or false: Homologous recombination occurs only in eukaryotes. This statement is:

False.

Determine whether the following statement is true or false: Most mRNAs do not encode protein. This statement is:

False.

Determine whether the following statement is true or false: New cells are generated continuously within the epithelial lining of the intestine, particularly in the intestinal villi. This statement is:

False.

Determine whether the following statement is true or false: Nothing of any genetic importance is contained within an intron. This statement is:

False.

Determine whether the following statement is true or false: Once a cell has become specialized to produce the set of proteins that are responsible for its distinctive properties, its gene expression patterns remain fixed. This statement is:

False.

Determine whether the following statement is true or false: Optogenetics uses transmitter-gated channels that have been introduced by genetic engineering techniques to control neurons. This statement is:

False.

Determine whether the following statement is true or false: The glucose-Na+ symport protein uses the electrochemical Na+ gradient to drive the active transport of glucose into the cell. Once this transporter has bound both Na+ and glucose, it preferentially opens toward the cytosol, where it releases both solutes. This statement is:

False.

Determine whether the following statement is true or false: The oxygen consumed during the oxidation of glucose in animal cells is returned to the atmosphere as CO2. - False. - True.

False.

Determine whether the following statement is true or false: Water passes through the cell membrane only through specialized channels called aquaporins. This statement is:

False.

Determine whether the following statements are true or false: "Eukaryotic chromosomes contain a single DNA replication origin." This statement is:

False.

Like the differentiated cells in an individual plant or animal, all bacteria have the same DNA. - True. - False. - Impossible to determine.

False.

Phosphorylation can only take place in the cell nucleus. - True. - False. - Impossible to determine.

False.

Which statement concerning feedback inhibition is false? - Feedback inhibition is difficult to reverse. - Feedback inhibition can work almost instantaneously. - Feedback inhibition regulates the flow through biosynthetic pathways. - Feedback inhibition is a feedback system for controlling enzyme activity. - In feedback inhibition, an enzyme acting early in a reaction pathway is inhibited by a later product of that pathway.

Feedback inhibition is difficult to reverse.

How do fermentation reactions in oxygen-starved muscle cells and anaerobically grown yeast cells differ? - Fermentation in muscle cells produces lactate and in yeast produces ethanol plus CO2. - Fermentation in muscle cells generates NAD+ and in yeast generates NADH. - Fermentation in yeast cells includes glycolysis, whereas in muscle cells it bypasses glycolysis. - Fermentation in muscle cells regenerates NAD+ and in yeast regenerates glucose. - Fermentation in muscle cells produces ethanol plus CO2 and in yeast produces lactate.

Fermentation in muscle cells produces lactate and in yeast produces ethanol plus CO2.

A fibroblast can attach indirectly to collagen via which type of extracellular matrix protein? - Fibronectin. - Cadherins. - Elastins. - Glycosaminoglycans.

Fibronectin.

Following the binding of an extracellular signal molecule, receptor tyrosine kinases (RTKs) do which of the following? - Become internalized so that they can phosphorylate and activate various intracellular signaling proteins. - Rearrange their multiple membrane-spanning helices in a way that enables self-phosphorylation. - Activate (or inactivate) the α and βγ subunits of G proteins. - Form dimers in which each polypeptide chain cross-phosphorylates the cytoplasmic tail of its partner. - Form dimers in which each polypeptide chain phosphorylates its own cytoplasmic tail

Form dimers in which each polypeptide chain cross-phosphorylates the cytoplasmic tail of its partner.

How many oxidation reactions occur during the citric acid cycle? - Three (one for each molecule of NADH produced). - One (to oxidize the acetyl group that enters the cycle). - Two (one for each molecule of CO2 released). - Eight (all are oxidation reactions, which is why cell respiration consumes so much oxygen). - Four (one for each dehydrogenase that participates in the cycle).

Four (one for each dehydrogenase that participates in the cycle).

At any given time, a typical differentiated human cell will express how many of its approximately 19,000 protein-coding genes? - All 19,000. - About 20. - From 5000 to 15,000. - About 200.

From 5000 to 15,000.

During stage I of photosynthesis, excited electrons move in which direction? - From NADPH to a mobile electron carrier to the chlorophyll special pair. - From the stroma to the thylakoid membrane to the thylakoid space. - From sunlight to the chlorophyll special pair. - From an electron carrier to NADP+ to ATP synthase. - From the chlorophyll special pair to an electron carrier to NADP+.

From the chlorophyll special pair to an electron carrier to NADP+.

It is energetically favorable for protons to flow in which direction? - From the intermembrane space to the mitochondrial matrix. - Toward the compartment with the most positive charge. - Across the outer mitochondrial membrane. - From the mitochondrial matrix to the intermembrane space. - Toward the compartment with the lowest pH.

From the intermembrane space to the mitochondrial matrix.

The electron-transport chain pumps protons in which direction? - From the matrix to the intermembrane space. - From the intermembrane space to the cytosol. - From the matrix to the cytosol. - From the intermembrane space to the matrix. - From the cytosol to the intermembrane space.

From the matrix to the intermembrane space.

Which of these glycolytic reactions is catalyzed by a kinase? (choose one or more). - Dihydroxyacetone phosphate → glyceraldehyde 3-phosphate. - 3-phosphoglycerate → 2-phosphoglycerate. - Fructose 6-phosphate → fructose 1,6-bisphosphate. - 1,3-bisphosphoglycerate → 3-phosphoglycerate - Glucose → glucose 6-phosphate. - Glucose 6-phosphate → fructose 6-phosphate. - Glyceraldehyde 3-phosphate → 1,3-bisphosphoglycerate.

Fructose 6-phosphate → fructose 1,6-bisphosphate. 1,3-bisphosphoglycerate → 3-phosphoglycerate Glucose → glucose 6-phosphate.

Botulism is a potentially fatal foodborne disease caused by the bacterium Clostridium botulinum. C. botulinum produces different toxins, several of which are proteases that cleave neuronal SNARE proteins. What normal process is blocked by cleavage and inhibition of SNARE proteins? - Docking of vesicles to target membranes. - Entry of proteins with ER signal sequences into the ER lumen. - Budding of vesicles from the endoplasmic reticulum. - Fusion of vesicles with target membranes.

Fusion of vesicles with target membranes.

Another step in PCR requires small single-stranded DNA primers to anneal to a target sequence on denatured DNA. Two primers are used. Ideally these primers will have similar melting temperatures. The melting temperature is defined as the temperature at which 50% of the DNA is in the single-stranded form. Which of the following primers will have the highest melting temperature? - G G G G A A A T T T C C C C. - A A A A G G C C T T T T. - A A A A G G G C C C T T T T. - G G G A A A T T T C C C.

G G G G A A A T T T C C C C

Which of the following represents the correct order of signaling events that might be triggered by a GPCR that activates expression of a target gene via the production of cyclic AMP? - Adenylyl cyclase → cyclic AMP → G protein → PKA → transcription regulator. - G protein → cyclic AMP → adenylyl cyclase → PKA → transcription regulator. - G protein → adenylyl cyclase → cyclic AMP → PKC → transcription regulator. - G protein → adenylyl cyclase → cyclic AMP → PKA → transcription regulator. - G protein → PKA → cyclic AMP → adenylyl cyclase → transcription regulator.

G protein → adenylyl cyclase → cyclic AMP → PKA → transcription regulator.

Most of the diversity in the rates of cell division in the adult body lies in variations in the time that each cell spends in which phase(s)? - G1 phase and G2 phase. - S phase and M phase. - G1 phase and G0 phase. - S phaseM phase.

G1 phase and G0 phase.

The end of a growing microtubule is rich in which type of tubulin subunit? - ATP-associated tubulin subunits. - ADP-associated tubulin subunits. - GDP-associated tubulin subunits. - GTP-associated tubulin subunits.

GTP-associated tubulin subunits.

Dynamic instability in microtubules stems from the intrinsic capacity of tubulin molecules to hydrolyze what? - Peptide bonds. - Water. - ATP. - Tubulin dimers. - GTP.

GTP.

Using powerful new sequencing technologies, investigators can now catalog every RNA molecule made by a cell and determine at what quantities these RNAs are present. In an experiment, researchers measured the relative quantities of two different mRNAs—one transcribed from gene A, the other from gene B—in two different cell types. Gene B is expressed in both the liver and the brain whereas gene A is expressed in the brain but not in the liver. Which most likely encodes a housekeeping protein? - Both genes. - Gene A only. - Gene B only. - Neither gene.

Gene B only.

In eukaryotes, what must assemble at a promoter before RNA polymerase can transcribe a gene? - Sigma factor. - General transcription factors. - Nucleotides required in transcription. - RNA primer.

General transcription factors.

What performs the function of bacterial sigma factor in eukaryotes? - Initiation proteins. - General transcription factors. - Nucleosomes. - Promoters.

General transcription factors.

Which of these processes require a membrane? (choose one or more). - Generation of ATP by photosynthesis in bacteria. - Generation of ATP by oxidative phosphorylation. - Generation of ATP by glycolysis. - Generation of energy by mitochondria. - Generation of ATP by photosynthesis in plants.

Generation of ATP by photosynthesis in bacteria. Generation of ATP by oxidative phosphorylation. Generation of energy by mitochondria. Generation of ATP by photosynthesis in plants.

When ATP is abundant, which processes are likely to occur? (choose one or more). - Gluconeogenesis. - Glycogen breakdown. - Glycolysis. - Mobilization of fats. - Glycogen synthesis.

Gluconeogenesis. Glycogen synthesis.

The glucose-Na+ symport transports glucose into the epithelial cells lining the gut. How would import of glucose into the cells be affected by addition of a leaky Na+ channel to their plasma membrane? - Na+ transport would slow, but glucose transport would remain high because glucose could still be transported by the glucose-Na+ symport. - Glucose transport would slow because the Na+gradient is dissipated by the Na+ channel. - Glucose transport would increase because the Na+gradient is strengthened by the Na+ channel. -A leaky Na+ channel would not affect glucose transport because these two transporters are unrelated.

Glucose transport would slow because the Na+gradient is dissipated by the Na+ channel.

When food is plentiful, animals can store glucose as what? - Glycogen. - Glycogen or starch. - Starch. - Glucose 6-phosphate. - Acetyl CoA

Glycogen.

For many anaerobic microorganisms, which metabolic pathway is the principal source of ATP? - Oxidative phosphorylation. - Calvin cycle. - Citric acid cycle. - Glycolysis. - Gluconeogenesis.

Glycolysis.

Which of the following organelles is surrounded by a single membrane? - Nucleus. - Chloroplasts. - Mitochondria. - Golgi apparatus.

Golgi apparatus.

Which organelle cannot receive proteins directly from the cytosol? - Golgi apparatus. - Nucleus. - Chloroplast. - Mitochondrion. - Peroxisome.

Golgi apparatus.

GTP hydrolysis and whether GTP or GDP is bound to tubulin is an important mechanism to control the dynamic instability of microtubules. Certain aspects of dynamic instability can be viewed using GFP-EB1. Which process(es) is it useful for visualizing and why? - Growing microtubules, because EB1 binds to the GTP-tubulin cap on microtubules. - Growing and shrinking microtubules, because EB1 binds to the GDP-tubulin cap on microtubules. - Shrinking microtubules, because EB1 binds to the GTP-tubulin cap on microtubules. - Growing and shrinking microtubules, because EB1 binds to the GTP-tubulin cap on microtubules.

Growing microtubules, because EB1 binds to the GTP-tubulin cap on microtubules.

Solve the riddle by determining which base could be speaking. "I'm a bulky base that clings extra tightly (by three hydrogen bonds, not just two) to my more petite partner on the opposite side of the DNA double helix. Who am I?" - Uracil. - Thymine. - Adenine. - Cytosine. - Guanine.

Guanine.

Which of the following steps are required in the activation of the G-protein signaling pathway? (choose one or more). - Gα exchanges GDP for GTP.. - Ligand binds to the G-protein-coupled receptor. - The activated receptor induces interaction between Gα and Gβγ. - Activated Gα influences target proteins.

Gα exchanges GDP for GTP.. Ligand binds to the G-protein-coupled receptor. Activated Gα influences target proteins.

In fungi, plants, and bacteria, which pump helps to drive the import of solutes? - H+ pumps. - Ca2+ pumps. - Na+ pumps. - K+ pumps. - ATP pumps.

H+ pumps.

What histone protein is thought to act as a linker that pulls nucleosomes together into a regular repeating array, resulting in a 30-nm fiber? - H1. - H2A. - H2B. - H3. - H4.

H1.

In the electron transport chain, the oxygen atoms in O2 become part of which of the following molecules? - ATP. - H2O. - CO2. - NADH. - Glucose (C6H12O6).

H2O

In one experiment, investigators create a liposome—a vesicle made of phospholipids—that contains a solution of 1 mM glucose and 1 mM sodium chloride. If this vesicle were placed in a beaker of distilled water, what would happen the fastest? - Na+ would diffuse out. - H2O would diffuse in. - NaCl would diffuse out. - Glucose would diffuse out. - Cl- would diffuse out.

H2O would diffuse in.

During electron transport, which serves as a ready source for protons that can be pumped across the membrane? - ATP. - Glucose. - H2O. - O2. - NADH.

H2O.

If DNA replication were conservative (although we know it is not), what would Meselson and Stahl have seen following the first round of replication in E. coli that had been switched from a heavy (15N-containing) nutrient medium to a light (14N-containing) nutrient medium? - Half the DNA would be heavy, and the other half would be intermediate in density. - All the DNA would be light. - Half the DNA would be intermediate in density, and one-quarter would be heavy while one-quarter would be light. - All the DNA would be heavy. - Half the DNA would be heavy, and the other half would be light.

Half the DNA would be heavy, and the other half would be light.

Which portion of a membrane phospholipid faces the outside of the membrane? - None, because phospholipids are confined to the interior of the membrane. - Amphipathic portion. - Fatty acids. - Head. - Tail.

Head.

Choose all of the following that correctly describe a characteristic of mismatch repair. (choose one or more). - Helicase unwinds the DNA in the mismatched area. - Exonuclease removes the newly synthesized DNA in the mismatched area. - Regions of improper base-pairing between parent and daughter strand are detected and repaired. - DNA polymerase and ligase fill in the gap.

Helicase unwinds the DNA in the mismatched area. Exonuclease removes the newly synthesized DNA in the mismatched area. Regions of improper base-pairing between parent and daughter strand are detected and repaired. DNA polymerase and ligase fill in the gap.

AMP-PNP is a non-hydrolyzable analog of ATP that can bind to proteins in a similar manner as ATP but is no longer hydrolyzed. Predict what would happen to helicase activity if AMP-PNP were added to a DNA replication reaction. - Helicase would function normally, since AMP-PNP is not the same as ATP and would not bind to helicase. - Helicase would no longer function, since the AMP-PNP is not hydrolyzed. ATP binding and hydrolysis induce the conformational changes that facilitate DNA unwinding by helicase. - Helicase would function normally, since AMP-PNP is an analog of ATP and would function in the place of ATP. - Helicase would function faster than normal, since the AMP-PNP is not hydrolyzed. ATP binding induces conformational changes, but hydrolysis slows DNA unwinding by helicase.

Helicase would no longer function, since the AMP-PNP is not hydrolyzed. ATP binding and hydrolysis induce the conformational changes that facilitate DNA unwinding by helicase.

What attaches epithelial cells to the basal lamina? - Cadherins. - Desmosomes. - Hemidesmosomes. - Tight junctions. - Adherens junctions.

Hemidesmosomes.

What is the general name given to the most highly condensed form of chromatin? - X chromatin. - 30-nm chromatin fiber. - Nucleosome. - Heterochromatin. - Euchromatin.

Heterochromatin.

Which statement is true about the association of histone proteins and DNA? - Each histone protein has a deep groove into which a DNA double helix tightly fits. - Histone proteins form hydrogen bonds with the nucleotide bases of DNA. - Histone proteins have a high proportion of positively charged amino acids, which bind tightly to the negatively charged DNA backbone. - Histone proteins insert themselves into the major groove of DNA. - Histone proteins have a high proportion of negatively charged amino acids, which bind tightly to the positively charged DNA backbone.

Histone proteins have a high proportion of positively charged amino acids, which bind tightly to the negatively charged DNA backbone.

What do microtubules resemble? - Hollow tubes. - Interlocking chains. - Ropelike strands. - Meshlike networks.

Hollow tubes.

Double-strand breaks can be repaired flawlessly by which of the following? - DNA mismatch repair system. - Homologous recombination. - DNA ligase. - Nonhomologous end joining. - DNA maintenance methyltransferase.

Homologous recombination.

Which type of non-covalent interaction can involve either the polypeptide backbone or amino acid side chains? - Hydrophobic forces. - Van der Waals attractions. - Hydrogen bonds. - Electrostatic interactions.

Hydrogen bonds.

What kinds of bonds link the two strands of a double helix to each other? Choose the most specific answer. - Hydrogen. - Ionic. - Covalent. - Hydrophobic.

Hydrogen.

What type of bond connects base pairs? - Hydrogen. - Covalent. - Ionic. - Van der Waals.

Hydrogen.

Which reaction releases the most energy? - Hydrolysis of ATP to ADP. - Transfer of phosphate group from ATP to glucose. - Hydrolysis of glucose 6-phosphate to glucose. - Hydrolysis of 1,3-bisphosphoglycerate to 3-phosphoglycerate. - Hydrolysis of ADP to AMP.

Hydrolysis of 1,3-bisphosphoglycerate to 3-phosphoglycerate.

Phospholipids assemble into in a membrane using: - Covalent bonds. - Assembly proteins. - Enzymes. - Hydrophobic forces.

Hydrophobic forces.

Which important intramolecular force for polypeptide folding is demonstrated by the clustering of green side chains? - Hydrogen bonding. - Electrostatic attraction. - Hydrophobic interaction. - All of the answer choices are correct.

Hydrophobic interaction.

Which characteristic describes the tails of phospholipids? - Amphipathic. - Coated with sugars. - Stiff. - Hydrophilic. - Hydrophobic.

Hydrophobic.

The electrochemical Na+ gradient established by the Na+ pumps in the plasma membrane allows animal cells to do what? (choose one or more). - Import nutrients, such as sugars and amino acids. - Stimulate muscle cell contraction. - Propagate electrical signals. - Control their pH.

Import nutrients, such as sugars and amino acids. Stimulate muscle cell contraction. Propagate electrical signals. Control their pH.

The electrochemical proton gradient across the mitochondrial membrane can be used to drive which processes? (choose one or more). - Export of pyruvate from the mitochondrial matrix. - Import of ADP into the mitochondrial matrix. - Export of ATP from the mitochondrial matrix. - Import of ATP into the mitochondrial matrix. - Import of pyruvate into the mitochondrial matrix.

Import of ADP into the mitochondrial matrix. Export of ATP from the mitochondrial matrix. Import of pyruvate into the mitochondrial matrix.

Which of the following statements is false? - The lifetime of a eukaryotic mRNA is controlled in part by nucleotide sequences in the mRNA itself. - In a eukaryotic mRNA, the poly-A tail promotes degradation of the molecule. - The longer the lifetime of an mRNA, the more protein it can produce. - Many eukaryotic mRNAs have a longer lifetime than bacterial mRNAs.

In a eukaryotic mRNA, the poly-A tail promotes degradation of the molecule.

Rb is an important protein for controlling cell proliferation by blocking entry into S phase. How does it exert its effect? - In its phosphorylated state, Rb is active and blocks G1-Cdk activity. - Signaling through mitogen-activated pathways activates Rb kinase activity. - When phosphorylated by G1-Cdk, Rb binds to DNA and activates transcription. - In its unphosphorylated state, Rb is active and blocks transcriptional regulators.

In its unphosphorylated state, Rb is active and blocks transcriptional regulators.

When compared to each other, the two replication forks that form at an origin of replication move in which direction? - In the 5'-to-3' direction. - In opposite directions. - In the 3'-to-5' direction. - Toward the template strand. - Toward the origin.

In opposite directions.

For every three molecules of CO2 that enter the carbon-fixation cycle, what is produced and what is consumed? - 1 molecule of glyceraldehyde 3-phosphate is produced, and 9 molecules of ATP + 12 molecules of NADPH are consumed. - 1 molecule of glyceraldehyde 3-phosphate is produced, and 6 molecules of ATP + 9 molecules of NADPH are consumed. - 1 molecule of glyceraldehyde 3-phosphate is produced, and 9 molecules of ATP + 6 molecules of NADPH are consumed. - 6 molecules of glyceraldehyde 3-phosphate are produced, and 6 molecules of ATP + 9 molecules of NADPH are consumed. - 3 molecules of glyceraldehyde 3-phosphate are produced, and 9 molecules of ATP + 6 molecules of NADPH are consumed.

1 molecule of glyceraldehyde 3-phosphate is produced, and 9 molecules of ATP + 6 molecules of NADPH are consumed.

Intracellular Ca2+ levels are important in cardiac muscle. Increasing intracellular Ca2+ levels in heart muscle cells leads to an increase in muscle contraction. Lowering the intracellular Ca2+ levels decreases the strength of cardiac muscle contraction. Congestive heart failure can occur when the heart's pumping of blood is weaker than normal, which leads to fluid collecting around organs, including the heart. One treatment method is to give the patient drugs that increase the strength of the heart muscle contraction. Which of the following might function as an effective treatment of congestive heart failure by increasing the strength of heart muscle contraction? (choose one or more). - A drug that decreases the activity of the Na+/Ca2+transporter. - A drug that blocks the calcium channel in heart muscle cells. - A drug that inhibits the Na+-K+ pump from establishing a strong Na+ gradient. - A diuretic drug that triggers removal of excess Na+from the body.

A drug that decreases the activity of the Na+/Ca2+transporter. A drug that inhibits the Na+-K+ pump from establishing a strong Na+ gradient. A diuretic drug that triggers removal of excess Na+from the body.

Colon cancer can occur when cells in the crypt become mutated and do not migrate properly out of the crypt. Which of the following drugs could potentially block the development of colon cancer? - A drug that leads to degradation of β-catenin. - A drug that decreases differentiation of transit amplifying cells. - A drug that increases survival of transit amplifying cells. - A drug that leads to degradation of APC.

A drug that leads to degradation of β-catenin.

Which chemical group is at the 3' end of a DNA strand? - A phosphate group. - A nitrogenous base. - A carboxyl group. - A hydroxyl group.

A hydroxyl group.

Which membrane would show a more rapid recovery of fluorescence in a FRAP study? - The saturation of fatty acids in a cell membrane does not affect the speed of fluorescence recovery in a FRAP study. - A membrane containing a larger proportion of unsaturated fatty acids. - A membrane containing a larger proportion of saturated fatty acids. - A membrane containing equal amounts of saturated and unsaturated fatty acids. - A membrane containing a large amount of cholesterol.

A membrane containing a larger proportion of unsaturated fatty acids.

Several genetic diseases are caused by mutations in mitochondrial DNA. In some countries, a technique known as mitochondrial replacement therapy is being used to allow couples to conceive healthy children. What is the basic procedure in this technique? - A nucleus from an egg with faulty mitochondria is placed into an enucleated donor egg. - Healthy mitochondria are placed into a sperm with faulty mitochondria. - A nucleus from a sperm with faulty mitochondria is placed into an enucleated donor sperm. - Healthy mitochondria are placed into an egg with faulty mitochondria.

A nucleus from an egg with faulty mitochondria is placed into an enucleated donor egg.

The genetic code was originally deciphered, in part, by experiments in which synthetic polynucleotides with repeating sequences were used as mRNAs to direct protein synthesis in cell-free extracts. Under these conditions, ribosomes could be made to start translation anywhere within the RNA molecules, with no start codon necessary. What peptide would be made by translation from a synthetic mRNA made of the repeating trinucleotide UCGUCG...? - A polymer of only serine: Ser-Ser-Ser... - A polymer of Ser-Arg-Ser... and a polymer of Arg-Val-Arg... - A peptide containing serines, arginines, and valines: ...Ser-Arg-Val-Ser-Arg-Val... - A polymer of serine (Ser-Ser-Ser...), a polymer of arginine (Arg-Arg-Arg...), and a polymer of valine (Val-Val-Val...).

A polymer of serine (Ser-Ser-Ser...), a polymer of arginine (Arg-Arg-Arg...), and a polymer of valine (Val-Val-Val...).

How does a polyribosome differ from the rough ER? - A polyribosome will synthesize only soluble proteins. - A polyribosome will synthesize only cytosolic proteins. - A polyribosome will not interact with the ER membrane. - A polyribosome will synthesize a single type of protein. - A polyribosome will be found only in prokaryotes.

A polyribosome will synthesize a single type of protein.

An electrochemical gradient has a chemical component and an electrical component. Which of the following will have the largest electrochemical gradient? - A positively charged ion, such as Na+, at high concentrations outside the cell. - A negatively charged ion, such as Cl-, at high concentrations outside the cell. - A positively charged ion, such as K+, at high concentrations inside the cell.

A positively charged ion, such as Na+, at high concentrations outside the cell

Which of the following drives the production of ATP from ADP and Pi by ATP synthase? - Sunlight. - A sodium (Na+) gradient. - Phosphorylation. - A proton (H+) gradient. - Hydrolysis.

A proton (H+) gradient.

In photosynthesis, what drives the generation of ATP by ATP synthase? - A proton gradient across the thylakoid membrane. - The absorption of light by a photosynthetic reaction center. - The phosphorylation of ATP synthase. - The generation of a charge separation in the photosynthetic reaction center. - The transfer of high-energy electrons to ATP synthase.

A proton gradient across the thylakoid membrane.

What is an operon? - A set of genes that is constitutively active. - A set of genes transcribed as a single mRNA from a single promoter. - A sequence of DNA that produces a variety of mRNAs. - A short sequence of DNA to which a transcription regulator binds. - A set of genes controlled by the binding of two or more transcription regulators.

A set of genes transcribed as a single mRNA from a single promoter.

Which of the following describes a primer used in DNA replication? - A short segment of RNA, about 10 nucleotides in length. - A short segment of DNA, about 100 nucleotides in length. - A short segment of RNA that serves as a template for synthesizing an Okazaki fragment. - A short segment of DNA, about 10 nucleotides in length. - A short segment of RNA, about 100 nucleotides in length.

A short segment of RNA, about 10 nucleotides in length.

Which of the following signaling pathways would be likely to trigger the most rapid cell response? - Platelet-derived growth factor binds to a receptor tyrosine kinase to activate a signaling pathway that stimulates cell proliferation at the site of a wound. - Acetylcholine binds to anion-channel-coupled receptor that allows Na+ to flow down its electrochemical gradient, triggering contraction of a skeletal muscle cell. - Adrenaline binds to a GPCR to activate a cyclic AMP signaling pathway that triggers glycogen breakdown. - Adrenaline binds to a GPCR to activate a cyclic AMP signaling pathway that triggers the synthesis of hormones in endocrine cells. - Nerve growth factor binds to a receptor tyrosine kinase to activate a signaling pathway that enhances the transcription of Bcl2, a protein that suppresses cell death.

Acetylcholine binds to anion-channel-coupled receptor that allows Na+ to flow down its electrochemical gradient, triggering contraction of a skeletal muscle cell.

Which of the following processes does not help move the duplicated chromosomes to opposite poles of the spindle in anaphase? - Dynein proteins attached to the cell cortex pull the poles apart. - Actin and myosin filaments in the spindle contract.. - Kinetochore microtubules shorten by depolymerization. - Kinesins push the spindle poles apart.

Actin and myosin filaments in the spindle contract.

What is the contractile ring made of? - Condensins. - Actin and myosin. - Integrins. - Microtubules. - Cohesins.

Actin and myosin.

What does cell crawling depend on? - Hydrolysis of ADP by motor proteins. - Actin filament polymerization. - Intermediate filaments. - Microtubule assembly.

Actin filament polymerization.

In eukaryotic cells, the cell cortex is made of a network of what type of protein filament? - Microtubules. - Intermediate filaments. - Actin filaments.

Actin filaments.

Which actin-binding protein would be most involved in the assembly and extension of lamellipodia? - Actin-related protein. - Myosin. - Profilin. - Formin. - Thymosin.

Actin-related protein.

Cholera and whooping cough are caused by bacterial toxins that do which of the following? - Disable a G protein. - Alter the activity of ion-gated channels. - Promote GTP hydrolysis. - Activate a G protein. - Activate adenylyl cyclase.

Activate adenylyl cyclase.

Which of the following statements is false? - Activated M-Cdk triggers the onset of cytokinesis. - Activated G1-Cdks and G1/S-Cdks help drive cells through G1 into S phase. - Activated S-Cdk initiates DNA replication.

Activated M-Cdk triggers the onset of cytokinesis.

One approach to killing cancer cells is to induce apoptosis. Which of the following are reasonable approaches that researchers can take to induce apoptosis in cancer cells? (choose one or more). - Activating the Bax protein. - Inhibiting the G1-cyclin-dependent kinase protein. - Inhibiting the p53 protein. - Inhibiting the Bcl2 protein.

Activating the Bax protein. Inhibiting the Bcl2 protein.

Cdk activity is regulated in several ways. Sort each of the following forms of Cdk into the active or inactive Cdk category.

Active Cdk: Cdk bound to cyclin. Threonine in T loop phosphorylated. Inactive Cdk: T loop in active site. Cdk alone. Cdk/cyclin bound to p27.

The movement of an ion against its concentration gradient is called what? - Passive transport. - Osmosis. - Facilitated diffusion. - Active transport.

Active transport.

Which of the following would be most likely to disrupt lipid bilayer formation? - Addition of a hydroxyl group to the head group of the lipid. - Addition of cholesterol to the membrane. - Addition of a methyl group to the end of the lipid tail. - Addition of a phosphate to the end of the lipid tail.

Addition of a phosphate to the end of the lipid tail.

IP3 signaling helps regulate sweating, which is important for regulating body temperature. Anhidrosis, the inability to sweat normally, can be caused by genetic and environmental factors. A rare mutation has been identified in a family with several children suffering from anhidrosis. The mutation inactivates the protein that IP3 binds on the ER membrane. Suppose cells were isolated from affected family members and exposed to different treatments. Which of the following treatments would be able to repair the signaling defect in cells isolated from these patients? - Addition of high amounts of Ca2+ in the cytosol. - Addition of high amounts of IP3 in the cell. - Addition of PKC to the cell. - Activation of phospholipase Cβ in the cell.

Addition of high amounts of Ca2+ in the cytosol.

Which type of epithelial cell junction plays a major role in an epithelial sheet's ability to develop tension and change its shape? - Gap junction. - Desmosome. - Tight junction. - Adherens junction.

Adherens junction.

Rigor mortis is a muscle stiffening that sets in a few hours after death. It is due to muscles being in a contracted state. It dissipates after several days as muscle proteins degenerate and cells break down. Rigor mortis is one sign that coroners take into account when estimating a time of death. At a cellular level, what is the mechanism behind rigor mortis? - After death, membranes become leaky, and sodium flowing into motor neurons trigger action potentials that contract muscles. - After death, ATP production ceases and ATP is needed for myosin release from actin. - After death, calcium ions are no longer available to bind troponin. - In the absence of ATP, kinesin and dynein no longer move along microtubules and remain locked in place.

After death, ATP production ceases and ATP is needed for myosin release from actin.

Which of these strategies do prokaryotic cells use to isolate and organize their chemical reactions? - Confining the proteins required for different metabolic processes within the plasma membrane. - Aggregating proteins into multicomponent complexes that form biochemical subcompartments with distinct functions. - None; these strategies are used only by eukaryotic cells. - None; prokaryotes do not regulate their metabolic processes. - Confining proteins required for different metabolic processes within different membrane-enclosed compartments

Aggregating proteins into multicomponent complexes that form biochemical subcompartments with distinct functions.

Which of the following accurately describes a step in GTP-driven nuclear transport? - Ran-GDP escorts the nuclear receptor back to the cytosol. - GTP hydrolysis powers a membrane-bound transporter protein. - GTP-bound cargo interacts specifically with the protein fibrils of the pore. - Binding of Ran-GTP to the receptor releases the cargo protein.

Binding of Ran-GTP to the receptor releases the cargo protein.

Three separate pathways make up the unfolded protein response in the ER. Sort the following characteristics of the unfolded protein responses into the correct pathway. - Activates transcription of specific genes. - Increases the ability of the ER to fold more proteins. - Leads to removal of an intron from a specific RNA and translation of the mRNA. - Contains both kinase and RNAse domains. - Phosphorylates a translation initiation factor, leading to a global reduction in translation. - Requires cleavage of the protein. - Travels to the Golgi apparatus and the nucleus before activating transcription.

All Three Pathways: Activates transcription of specific genes. Increases the ability of the ER to fold more proteins. IRE1 Pathway Only: Leads to removal of an intron from a specific RNA and translation of the mRNA. Contains both kinase and RNAse domains. PERK Pathway Only: Phosphorylates a translation initiation factor, leading to a global reduction in translation. ATF6 Pathway Only: Requires cleavage of the protein. Travels to the Golgi apparatus and the nucleus before activating transcription.

What is the best description of connective tissue? - All of its cells associate via cadherins in their plasma membrane. - All of its cells are electrically connected via gap junctions. - All of its cells are sparsely distributed in the extracellular matrix. - All of its cells rest on a basal lamina.

All of its cells are sparsely distributed in the extracellular matrix.

If the cell is deprived of mitogens for a prolonged period of time, it will enter a non-proliferating state in which it may remain for how long? - Hours. - Days. - Weeks. - The remaining lifetime of the cell. - All of the above are possible.

All of the above are possible.

Which of the following is true for most genes? - A gene is a segment of DNA that contains the instructions for making a particular protein. - A gene is a segment of DNA that contains the instructions for making a particular RNA. - A gene is a unit of heredity that contains instructions that dictate the characteristics of an organism. - All of the above are true regarding genes. - None of the above are true regarding genes.

All of the above are true regarding genes.

The tails of the core histone proteins can be chemically modified by the covalent addition of what type of chemical group? - Acetyl. - Methyl. - Phosphate. - All of these. - None of these.

All of these.

Which parts of amino acids are involved in a peptide bond? - Carboxyl groups of both amino acids. - Amino groups of both amino acids. - Side chains of both amino acids. - Amino group of one amino acid and carboxyl group of the other. - Amino group of one amino acid and side chain of the other. - Carboxyl group of one amino acid and side chain of the other.

Amino group of one amino acid and carboxyl group of the other.

Which of the following is true of lysosomes? - An ATP-driven H+ pump in the lysosomal membrane maintains the organelle's pH. - The products of digestion in lysosomes leave the lysosome by transport vesicles. - Lysosomes have a pH that is higher than that of the cytosol. - Lysosomes contain around 40 types of hydrolytic enzymes, which are optimally active at pH 7.2. - Most of the lysosomal membrane proteins have glycosylated regions on the cytosolic side of the membrane.

An ATP-driven H+ pump in the lysosomal membrane maintains the organelle's pH.

Within the ribosome, the formation of peptide bonds is catalyzed by what component? - tRNA. - mRNA. - An RNA molecule in the large ribosomal subunit. - A peptidase in the small ribosomal subunit. - Aminoacyl-tRNA synthetase.

An RNA molecule in the large ribosomal subunit.

What do the segments of a transmembrane protein that cross the lipid bilayer usually consist of? - A beta-sheet with alternating polar and non-polar side chains. - A beta-sheet with mostly polar side chains. - An alpha-helix with mostly polar side chains. - An alpha-helix with mostly non-polar side chains.

An alpha-helix with mostly non-polar side chains.

Translation takes place in a series of four steps. Which of these best describes this four-step cycle during elongation? - An aminoacyl-tRNA binds to the vacant A site on the ribosome; the large subunit of the ribosome translocates, moving the bound tRNA to the P site; a peptide bond forms; the small subunit of the ribosome translocates, ejecting the tRNA from the E site. - An aminoacyl-tRNA binds to the vacant A site on the ribosome; a peptide bond forms; the large subunit of the ribosome translocates, moving the bound tRNAs to the E and P sites; the small subunit of the ribosome translocates, ejecting the tRNA from the E site. - The large subunit of the ribosome translocates, moving the bound tRNAs to the E and P sites; an aminoacyl-tRNA binds to the vacant A site on the ribosome; a peptide bond forms; the small subunit of the ribosome translocates, ejecting the tRNA from the E site. - An aminoacyl-tRNA binds to the vacant E site on the ribosome; a peptide bond forms; the large subunit of the ribosome translocates, moving the bound tRNAs to the A and P sites; the small subunit of the ribosome translocates, ejecting the tRNA from the A site. - An aminoacyl-tRNA binds to the vacant P site on the ribosome; a peptide bond forms; the large subunit of the ribosome translocates, moving the bound tRNAs to the E and P sites; the small subunit of the ribosome translocates, ejecting the tRNA from the E site.

An aminoacyl-tRNA binds to the vacant A site on the ribosome; a peptide bond forms; the large subunit of the ribosome translocates, moving the bound tRNAs to the E and P sites; the small subunit of the ribosome translocates, ejecting the tRNA from the E site.

Place the following steps of RNA splicing in the correct order from first step to last.

BBP and U2AF bind to the A in the branch-point site in the intron, while the U1 snRNP binds the 5' splice site. The U2 snRNP binds the brand-point site and the U4/U5 and U5 snRNPs bind to the 5' splice site. A conserved adenine in the branch-point site attacks the 5' splice site, cutting the RNA at the 5' splice site. The 5' end of the intron binds the branch-point adenine, forming a lariat. The 3' end of the first exon interacts with the 3' splice site, forming a new bond in the sugar-phosphate backbone. The lariat is released and degraded.

Hydrogen bonds between which types of atoms have been found to stabilize a polypeptide's folded shape? (choose one or more). - Multiple carbon atoms. - Backbone and side-chain atoms. - Backbone atoms. - Side-chain atoms.

Backbone and side-chain atoms. Backbone atoms. Side-chain atoms.

If the backbone of a polypeptide is hydrophilic, how can a transmembrane alpha helix span the hydrophobic portion of the lipid bilayer? - Because many transmembrane alpha helices must come together in a way that neutralizes the hydrophilic backbone. - Because amino acid side chains in a transmembrane helix are hydrophobic and interact with the hydrophobic interior of the bilayer. - Because the hydrophilic backbone makes a hole in the membrane. - Because the membrane bends in such a way that the polar heads of the lipids contact the transmembrane helix.

Because amino acid side chains in a transmembrane helix are hydrophobic and interact with the hydrophobic interior of the bilayer.

Which of the following is a common transmembrane protein structure that can traverse the membrane to form a pore or channel by alternating hydrophobic and hydrophilic amino acids? - Beta barrel. - Beta sheet. - Multipass alpha helix. - Single-pass alpha helix.

Beta barrel.

Animals exploit the phospholipid asymmetry of their plasma membrane to distinguish between live cells and dead ones. When animal cells undergo a form of programmed cell death called apoptosis, phosphatidylserine—a phospholipid that is normally confined to the cytosolic monolayer of the plasma membrane—rapidly translocates to the extracellular, outer monolayer. The presence of phosphatidylserine on the cell surface serves as a signal that helps direct the rapid removal of the dead cell. How might a cell actively engineer this phospholipid redistribution? - By boosting the activity of a flippase in the plasma membrane. - By activating a scramblase and inactivating a flippase in the plasma membrane. - By inverting the existing plasma membrane. - By inactivating a scramblase in the plasma membrane. - By inactivating both a flippase and a scramblase in the plasma membrane.

By activating a scramblase and inactivating a flippase in the plasma membrane.

Mitochondria are essentially the same in all eukaryotes, including plants, animals, and fungi. Based on this observation, how were mitochondria most likely acquired? - By an ancestral eukaryotic cell and then replaced by chloroplasts in the line that led to plant cells. - By an ancestral eukaryotic cell before the lines that led to animal cells, plant cells, and fungi diverged. - By an ancestral prokaryote and then lost in the line that led to archaea. - By a prokaryotic cell approximately 1000 years ago. - From a free-living, photosynthetic bacterium.

By an ancestral eukaryotic cell before the lines that led to animal cells, plant cells, and fungi diverged.

Prozac, a common antidepressant medication, functions by altering neurotransmitter levels in the brain. How does Prozac work? - By blocking the re-uptake of serotonin after it has been released, increasing the amount available in the synapses that use it. - By making serotonin-gated channels easier to open. - By increasing the amount of serotonin released from the presynaptic neuron, increasing the amount available in the synapses that use it. - By decreasing the amount of serotonin released from the presynaptic neuron, decreasing the amount available in the synapses that use it.

By blocking the re-uptake of serotonin after it has been released, increasing the amount available in the synapses that use it.

How do proteases function? - By hydrolyzing peptide bonds between amino acids in a protein. - By forming peptide bonds between amino acids in a protein. - By folding polypeptides into correct three-dimensional structures. - By folding polypeptides into incorrect, abnormal three-dimensional structures. - By blocking the activity of ribosomes.

By hydrolyzing peptide bonds between amino acids in a protein.

In which organism is silencing genes by RNA interference (RNAi) particularly easy since the animals can be fed E. coli that has been genetically engineered to produce the double-stranded RNAs that trigger RNAi? - C. elegans. - Mouse. - Human. - D. melanogaster.

C. elegans.

Which of the following describes the Lac operon in E. coli when lactose, but not glucose, is present in the culture medium? - Neither CAP nor the Lac repressor is bound to the Lac operon's regulatory DNA, and the Lac operon is not expressed. - The Lac repressor, but not CAP, is bound to the Lacoperon's regulatory DNA, and the Lac operon is not expressed. - Neither CAP nor the Lac repressor is bound to the Lac operon's regulatory DNA, and the Lac operon is expressed. - CAP, but not the Lac repressor, is bound to the Lacoperon's regulatory DNA, and the Lac operon is expressed. - CAP and the Lac repressor are both bound to the Lac operon's regulatory DNA, and the Lac operon is not expressed.

CAP, but not the Lac repressor, is bound to the Lacoperon's regulatory DNA, and the Lac operon is expressed.

Given the type of transporter as determined in Part 1, choose all of the correct statements below that relate to the function of the Na+/Ca2+transporter. (choose one or more). - Ca2+ is transported against its electrochemical gradient. - The transporter directly uses ATP as an energy source for transporting ions. - The transporter uses the Na+ electrochemical gradient as an energy source for transporting ions. - Na+ is transported against its electrochemical gradient.

Ca2+ is transported against its electrochemical gradient. The transporter uses the Na+ electrochemical gradient as an energy source for transporting ions.

During vertebrate development, a sheet of epithelial tissue invaginates to form the neural tube, a structure that eventually forms the spinal cord and brain. Mutations that interfere with the function of which proteins would be most likely to disrupt the epithelial sheet movement that drives this developmental process? - Cadherins. - Connexons. - Claudins. - Occludins.

Cadherins.

Ions in solution are found in a hydration shell of water. This shell must be removed for an ion to pass through the channel. How does the K+channel accomplish removal of the water from the shell around the ion? - Carbonyl groups lining the wall of the pore can interact with the unsolvated K+ ion, balancing the energy needed to remove the hydration shell. - The K+ channel has four subunits; one subunit removes the hydration shell as the ion passes through the pore formed by the three other subunits. - Rigid protein loops strip the hydration shell from the potassium so that the ion is the right diameter to pass through the pore. - The K+ channel uses the energy in ATP hydrolysis to remove the hydration shell from the K+ ion.

Carbonyl groups lining the wall of the pore can interact with the unsolvated K+ ion, balancing the energy needed to remove the hydration shell.

Which statement regarding the ratio of ATP and ADP concentrations in the cell cytosol is accurate? - Cells keep the concentration of ADP in the cytosol about 100 times higher than that of ATP. - Cells keep the concentration of ATP in the cytosol about 10 times higher than that of ADP. - The poison cyanide depletes ADP by halting electron transport. - Cells keep the concentration of ATP in the cytosol about 100 times higher than that of ADP. - Cells keep the concentration of ADP in the cytosol about 10 times higher than that of ATP. - The poison cyanide depletes ATP by halting the citric acid cycle.

Cells keep the concentration of ATP in the cytosol about 10 times higher than that of ADP.

Experimental evidence supporting the semiconservative nature of DNA replication came from which of the following? - Incubation of purified DNA with radioactive nucleotides and DNA polymerase. - The structure of DNA determined by Watson and Crick. - Centrifugation of DNA purified from E. coli grown in a radioactive medium. - Observation of replication forks with an electron microscope. - Visual observation of cell division in E. coli.

Centrifugation of DNA purified from E. coli grown in a radioactive medium.

Which of the following represents the specialized DNA sequence that attaches to microtubules and allows duplicated eukaryotic chromosomes to be separated during M phase? - Centrosome. - Histone. - Telomere. - Mitotic spindle. - Centromere. - Nucleosome.

Centromere.

Which of the following events occur in the hypothalamus of a mouse brain expressing channelrhodopsin when the blue light is switched on? (choose one or more). - Channelrhodopsin is activated and opened. - The neuron membrane is depolarized. - Na+ floods into the neuron. - Cl- floods out of the neuron.

Channelrhodopsin is activated and opened. The neuron membrane is depolarized. Na+ floods into the neuron.

Channelrhodopsin can be used to manipulate the activity of neurons for what reason? - Channelrhodopsin, when activated by light, depolarizes the membrane and stimulates neurons. - Channelrhodopsin and neurons are both sensitive to light. - Channelrhodopsin opens in response to neurotransmitters. - Channelrhodopsin, when activated by light, polarizes the membrane and stimulates neurons. - Channelrhodopsin is a natural component of excitatory neurons.

Channelrhodopsin, when activated by light, depolarizes the membrane and stimulates neurons.

How do transporters and channels select which solutes they help move across the membrane? - Channels allow the passage of solutes that are electrically charged; transporters facilitate the passage of molecules that are uncharged. - Channels discriminate between solutes mainly on the basis of size and electric charge; transporters bind their solutes with great specificity in the same way an enzyme binds its substrate. - Channels will allow the passage of any solute as long as it has an electrical charge; transporters bind their solutes with great specificity in the same way an enzyme binds its substrate. - Both channels and transporters discriminate between solutes mainly on the basis of size and electric charge. - Transporters discriminate between solutes mainly on the basis of size and electric charge; channels bind their solutes with great specificity in the same way an enzyme binds its substrate.

Channels discriminate between solutes mainly on the basis of size and electric charge; transporters bind their solutes with great specificity in the same way an enzyme binds its substrate.

How are misfolded proteins and incompletely assembled proteins retained in the ER? - Incompletely assembled proteins aggregate into large complexes that are degraded in the ER. - Misfolded proteins can exit the ER but are immediately returned by a KDEL sequence. - Chaperone proteins bind them and prevent their entry into vesicles. - The misfolded proteins are immediately sent to lysosomes.

Chaperone proteins bind them and prevent their entry into vesicles.

Which of the following occurs during metaphase? - Nuclear envelope reassembly. - Nuclear envelope breakdown. - Chromosomes being aligned under tension at the spindle equator. - Chromosomes being captured by spindle microtubules. - Chromosomes being condensed.

Chromosomes being aligned under tension at the spindle equator.

Touching a mimosa plant leaf triggers an action potential that propagates along the leaflet cells. When the action potential reaches the cells in the hinge region, the action potential triggers the opening of ion channels. Which of the following correctly describe the direction of movement of substances across the plasma membrane in response to the action potential? (choose one or more). - H2O enters the cell through aquaporin channels. - Cl- channels open and Cl- exits the cell. - K+ channels open and K+ exits the cell. - Ca2+ channels open and Ca2+ enters the cell.

Cl- channels open and Cl- exits the cell. K+ channels open and K+ exits the cell. Ca2+ channels open and Ca2+ enters the cell.

Stem cells hold great promise both for studying cell fate determination and organ development and for clinical use, such as blood stem cell transplants to treat patients whose own immune systems have been severely impaired. Clinicians also hope that stem cells can be coaxed into specific cell types to treat diseases in which that cell type is damaged, such as replacing neurons in Parkinson's disease. In 2008, researchers assessed the ability of neurons derived from reprogrammed fibroblasts to ameliorate the symptoms of Parkinson's disease in a rat model. Based on your knowledge of stem cells and cell differentiation, what intermediate step was necessary to convert fibroblasts into neurons? - Converting fibroblasts into embryonic stem cells via introduction of tumor suppressor genes. - Converting fibroblasts into organoids in culture. - Converting fibroblasts into embryonic stem cells via introduction of transcription factors. - Converting fibroblasts into induced pluripotent stem cells.

Converting fibroblasts into induced pluripotent stem cells.

How does phosphorylation of a protein affects its activity? - Always increases activity. - Could increase or decrease activity. - Never affects activity. - Always decreases activity.

Could increase or decrease activity.

A disulfide bond is a(n) ___________ interaction within the protein.

Covalent.

Which of the following statements is false? - Cyclins help activate Cdks by phosphorylating them. - Cyclins have no enzymatic activity on their own. - Cyclins are degraded at specific times in the cell cycle. - Cyclins vary in concentration at different stages of the cell cycle.

Cyclins help activate Cdks by phosphorylating them.

Which amino acid is involved in the formation of disulfide bonds? - Cysteine. - Proline. - Tryptophan. - Methionine. - None of these.

Cysteine.

To reinforce cell identity, vertebrate cells can methylate which nucleotide? - Any cytosine or guanine. - Any cytosine. - Any guanine. - Cytosine that falls next to guanine in the sequence CG. - Guanine that falls next to cytosine in the sequence CG.

Cytosine that falls next to guanine in the sequence CG.

Which best describes a pathway that a protein might follow from synthesis to secretion? - ER → Golgi apparatus → transport vesicle → endosome → secretory vesicle → plasma membrane. - Cytosol → ER → Golgi apparatus → transport vesicle → endosome → secretory vesicle → plasma membrane. - Cytosol → ER → transport vesicle → Golgi apparatus → transport vesicle → plasma membrane. - Cytosol → ER → secretory vesicle → plasma membrane. - ER → Golgi apparatus → secretory vesicle → plasma membrane.

Cytosol → ER → transport vesicle → Golgi apparatus → transport vesicle → plasma membrane.

What is the largest single compartment in most cells? - Cytosol. - Mitochondrial matrix. - Nucleus. - Gogli apparatus. - Endoplasmic reticulum.

Cytosol.

What is the largest single compartment in most cells? - Golgi apparatus. - Mitochondrial matrix. - Nucleus. - Cytosol. - Endoplasmic reticulum.

Cytosol.

Researchers studying yeast discovered that, for some mutants, when the temperature at which the cells are grown is elevated from 25ºC to 37ºC, their secretory pathway no longer functions and the cells grow dense with unsecreted protein. When these cells are examined microscopically, they can be divided into groups that vary in terms of where the unsecreted proteins accumulate. In some of the mutants, proteins accumulate in the ER; in others, the Golgi; in others, they accumulate in vesicles near the plasma membrane. What is the likely explanation for this difference in appearance? - Different temperature-sensitive mutations promote an increase in protein synthesis. - Different temperature-sensitive mutations disrupt protein synthesis. - Different temperature-sensitive mutations affect different stages of the transport process. - Different temperature-sensitive mutations disrupt the integrity of cell membranes. - The temperature-sensitive mutant proteins accumulate in different compartments.

Different temperature-sensitive mutations affect different stages of the transport process.

In eukaryotes, which parts of a gene are transcribed into RNA? - Introns and exons. - The protein-coding region only. - Exons only. - Introns only.

Introns and exons.

Which of the following is a difference between the mechanisms of DNA polymerase and RNA polymerase? - DNA polymerase uses DNA as a template for making DNA copies; RNA polymerase uses RNA as a template for making RNAs. - DNA polymerase needs a base-paired 3'-OH for a polymerization reaction to occur; RNA polymerase can polymerize two nucleotides without a base-paired 3'-OH. - The direction of polymerization for DNA polymerase is 5' to 3', but RNA polymerase is 3' to 5'. - DNA polymerase uses the energy from the hydrolysis of the nucleotide triphosphates to drive the reaction; RNA polymerase uses reduced electron carriers.

DNA polymerase needs a base-paired 3'-OH for a polymerization reaction to occur; RNA polymerase can polymerize two nucleotides without a base-paired 3'-OH.

External signals can stimulate apoptosis by activating a set of cell-surface receptors known as what? - Bax receptors. - Death receptors. - Suicide receptors. - Caspase receptors.

Death receptors.

The reassembly of the nuclear envelope at telophase depends on which of the following? - Phosphorylation of M cyclin. - Dephosphorylation of nuclear lamins and nuclear pore proteins. - Dephosphorylation of M cyclin. - Phosphorylation of nuclear lamins and nuclear pore proteins.

Dephosphorylation of nuclear lamins and nuclear pore proteins.

In their 1953 paper on the double-helical structure of DNA, Watson and Crick famously wrote: "It has not escaped our notice that the specific pairing we have postulated immediately suggests a possible copying mechanism for the genetic material." What did they mean? - The sugar-phosphate backbone of DNA holds the helix together in a way that allows the genetic information to be copied. - When a cell divides, each DNA helix is split between the daughter cells. - Each strand in a DNA double helix contains all the information needed to produce a complementary partner strand. - Sexually reproducing organisms swap their DNA.

Each strand in a DNA double helix contains all the information needed to produce a complementary partner strand.

How does the tRNA synthetase enzyme charge a tRNA with the correct amino acid? - Different tRNA synthetases recognize specific sequences in the D and T loops of the tRNA and attach the matching amino acid. - Different tRNA synthetases recognize specific sequences in the final three nucleotides on the 3' end of the tRNA and attach the matching amino acid. - Each different tRNA synthetase has one region that recognizes the tRNA anticodon and a second region that attaches the matching amino acid to the CCA at the 3' end of the tRNA. - A single tRNA synthetase in the cell is responsible for recognizing specific anticodon sequences and adding the correct amino acid to each tRNA.

Each different tRNA synthetase has one region that recognizes the tRNA anticodon and a second region that attaches the matching amino acid to the CCA at the 3' end of the tRNA.

Which statement about cell signaling is correct? - Each receptor is generally activated by only one type of signal molecule. - All extracellular signal molecules act by binding to receptors on the cell surface. - Each type of extracellular signal molecule induces the same response in all target cells. - Each receptor triggers one particular type of cell behavior, for example, activating gene expression. - All cell types are able to respond to the same set of signal molecules.

Each receptor is generally activated by only one type of signal molecule.

Which of the following organelles is an irregular maze of interconnected spaces enclosed by a membrane and functions as the site where most cell membrane components, as well as materials destined for export from the cell, are made? - Golgi apparatus. - Ribosome. - Endoplasmic reticulum. - Peroxisome. - Lysosome.

Endoplasmic reticulum.

Which organelle is important for controlling the concentration of calcium ions in the cytosol? - Golgi apparatus. - Lysosome. - Endoplasmic reticulum. - Nucleus.

Endoplasmic reticulum.

When scientists were first studying the fluidity of membranes, they did an experiment using hybrid cells. Certain membrane proteins in a human cell and a mouse cell were labeled using antibodies coupled with differently colored fluorescent tags. The two cells were then coaxed into fusing, resulting in the formation of a single, double-sized hybrid cell. Using fluorescence microscopy, the scientists then tracked the distribution of the labeled proteins in the hybrid cell. Which best describes the results they saw and what they ultimately concluded? - The mouse and human proteins remained confined to the portion of the plasma membrane that derived from their original cell type. This suggests that cells can restrict the movement of their membrane proteins to establish cell-specific functional domains. - Initially, the mouse and human proteins were confined to their own halves of the newly formed hybrid cell, but over time, the two sets of proteins recombined such that they all fluoresced with a single, intermediate color. - Initially, the mouse and human proteins were confined to their own halves of the newly formed hybrid cell, but over time, the two sets of proteins became evenly intermixed over the entire cell surface. This suggests that proteins, like lipids, can move freely within the plane of the bilayer. - The mouse and human proteins began to intermix and spread across the surface of the hybrid cell, but over time, one set of proteins became dominant and the other set was lost. This suggests that cells can ingest and destroy foreign proteins. - Initially, the mouse and human proteins intermixed, but over time, they were able to re-segregate into distinct membrane domains. This suggests that cells can restrict the movement of membrane proteins. - At first, the mouse and human proteins were confined to their own halves of the newly formed hybrid cell, but over time, the two sets of proteins became divided such that half faced the cytosol and half faced the hybrid cell exterior. This suggests that flippases are activated by cell fusion.

Initially, the mouse and human proteins were confined to their own halves of the newly formed hybrid cell, but over time, the two sets of proteins became evenly intermixed over the entire cell surface. This suggests that proteins, like lipids, can move freely within the plane of the bilayer.

When activated phospholipase C cleaves an inositol phospholipid, what happens to the small signaling molecules the enzyme produces? - Both inositol 1,4,5-trisphosphate (IP3) and diacylglycerol are released into the cytosol. - Both inositol 1,4,5-trisphosphate (IP3) and diacylglycerol are retained in the membrane. - Both inositol 1,4,5-trisphosphate (IP3) and Ca2+ are released into the cytosol. - Inositol 1,4,5-trisphosphate (IP3) is released into the cytosol, while diacylglycerol is retained in the membrane. - Inositol 1,4,5-trisphosphate (IP3) remains in the membrane, while diacylglycerol is released into the cytosol.

Inositol 1,4,5-trisphosphate (IP3) is released into the cytosol, while diacylglycerol is retained in the membrane.

Your friend is attempting to study the function of the Na+-K+ pump and has created spherical liposomes that contain only the Na+-K+ pump. She has inserted the pumps so that the extracellular side of the pump is also outside the liposome. She has added different ions and energy sources to the beaker with the liposomes, but no pumping of ions occurs. You explain that components must be in the proper location inside or outside the liposome for the pump to work. Help her by adding the proper components to the inside or outside of the liposome so that proper pumping occurs. Place unneeded components in the unneeded box. - ATP - Na+ - K+ - GTP - Cl−

Inside the Liposome: ATP Na+ Outside the Liposome: K+ Unneeded: GTP Cl−

Which of the following is not true of integrin? - When integrin binds to the extracellular matrix, it stretches into an extended, activated state to attach to molecules on the inside of the cell. - Integrin molecules serve as stable, permanent anchors that anchor a cell to the extracellular matrix. - Integrins undergo extensive conformational changes on binding to molecules on either side of the plasma membrane. - Certain intracellular chemical signals can activate integrins from inside the cell, causing them to reach out and grab hold of extracellular structures.

Integrin molecules serve as stable, permanent anchors that anchor a cell to the extracellular matrix.

Which type of protein in a fibroblast's plasma membrane attaches to the extracellular matrix on the outside of the cell and (through adapter molecules) to actin inside the cell? - Integrin. - Cadherin. - Fibronectin. - Proteoglycan - Collagen.

Integrin.

Which of the following statements are consistent with the structure and function of intermediate filaments? (choose one or more). - Intermediate filaments protect cells from mechanical stress because they have high tensile strength and resist stretching. - Each filament is made of eight strands, and each strand is made from staggered tetramers linked end to end. - Intermediate filaments can connect cells at cell-cell junctions called desmosomes. - Intermediate filaments are constructed of identical subunits found in all eukaryotic cells.

Intermediate filaments protect cells from mechanical stress because they have high tensile strength and resist stretching. Each filament is made of eight strands, and each strand is made from staggered tetramers linked end to end. Intermediate filaments can connect cells at cell-cell junctions called desmosomes.

Which of these tissues has the fastest turnover rate? - Intestinal epithelium. - Nerves. - Bone. - Blood.

Intestinal epithelium.

Which of these statements is true? - Attachment to a polyribosome prevents a growing polypeptide chain from being translocated into the ER. - Only those proteins that are destined to remain inserted in the membrane are made on the rough ER; soluble proteins are made in the cytosol. - Membrane-bound ribosomes and free ribosomes are structurally and functionally identical; they differ only in the proteins they are making at a particular time. - The N-terminus of a protein translocated into the ER will always remain in the ER lumen. - The signal sequence that directs a growing polypeptide chain to enter the ER membrane is always removed by a transmembrane signal peptidase.

Membrane-bound ribosomes and free ribosomes are structurally and functionally identical; they differ only in the proteins they are making at a particular time.

Mutations in the nucleic acid sequence of a gene can sometimes direct the substitution of one amino acid for another in the encoded protein. Which amino acid substitution would be most likely to severely disrupt the normal structure of a protein? - Leucine to Isoleucine. - Asparagine to Threonine. - Methionine to Arginine. - Alanine to Glycine. - Tryptophan to Phenylalanine.

Methionine to Arginine.

c-Met is an oncogene that contributes to the development of certain cancers by triggering cell division and tumor growth. In a 2009 article, Yan and colleagues found regions in the 3' untranslated region of c-Met mRNA complementary to microRNA-1/206. In addition, higher levels of microRNA-1/206 were associated with slower cell proliferation. What is a likely explanation for the inverse correlation between microRNA-1/206 and cell proliferation? - MicroRNA-1/206 stabilizes c-Met mRNA, leading to enhanced translation. - MicroRNA-1/206 targets c-Met mRNA for destruction via RISC. - MicroRNA-1/206 codes for a protein that directly binds to and inhibits c-Met protein. - MicroRNA-1/206 codes for a tumor suppressor protein that directly inhibits cell proliferation.

MicroRNA-1/206 targets c-Met mRNA for destruction via RISC.

Microtubules participate in the spatial polarization of nerve cells because: - Microtubules are only located in the nerve cell body. - Microtubules have no polarity. - Microtubules extend through the axon with (−) ends toward the terminal and bind vesicles there. - Microtubule (−) ends originate near the cell body allowing (+) end-directed transport along the axon.

Microtubule (−) ends originate near the cell body allowing (+) end-directed transport along the axon.

How would the animation of microtubule dynamics change after adding a non-hydrolyzable analog of GTP to the cells expressing GFP tubulin? - Microtubule dynamics would not change. - Microtubules would shrink. - Dynamic instability would increase as microtubules rapidly switch between growing and shrinking. - Microtubules would grow longer.

Microtubules would grow longer.

Which of the following cytoskeletal structures are the most common for providing tracks for guiding intracellular transport? - Kinesins. - Actin filaments. - Microtubules. - Dyneins. - Intermediate filaments.

Microtubules.

Which of the following cytoskeletal structures grows out from a centrosome toward the cell periphery? - Microtubules. - Actin filaments. - Intermediate filaments.

Microtubules.

Which of the following cytoskeletal structures makes up the mitotic spindle? - Microtubules. - Actin filaments. - Intermediate filaments.

Microtubules.

Which of the following accurately matches the types of filaments in the cytoskeleton with their cellular function? - Microtubules: cell movement, Intermediate Filaments: chromosome segregation, Actin Filaments: strength/support. - Microtubules: chromosome segregation, Intermediate Filaments: strength/support, Actin Filaments: cell movement. - Microtubules: cell movement, Intermediate Filaments: strength/support, Actin Filaments: chromosome segregation. - Microtubules: strength/support, Intermediate Filaments: cell movement, Actin Filaments: chromosome segregation.

Microtubules: chromosome segregation, Intermediate Filaments: strength/support, Actin Filaments: cell movement.

What is the name of the type of damage caused by an improperly paired base in the DNA? - Thymine dimer. - Double-strand break. - Mismatch. - Depurination.

Mismatch.

Which is not evidence for the endosymbiotic origins of mitochondria and chloroplasts? - Mitochondria and chloroplasts are each surrounded by a double membrane. - Mitochondria and chloroplasts have similar DNA. - Mitochondria and chloroplasts resemble bacteria. - Mitochondria and chloroplasts contain their own DNA. - Mitochondria and chloroplasts reproduce by dividing in two.

Mitochondria and chloroplasts have similar DNA.

Which statement is not true of mitochondria? - Mitochondria are involved in the chemical energy cycle of the cell. - Mitochondria are thought to have originated from bacteria. - Mitochondria have an inner and outer membrane. - Mitochondria contain their own DNA.

Mitochondria have an inner and outer membrane.

In plant cells, where does the citric acid cycle take place? - Cytosol. - Chloroplasts and mitochondria. - Chloroplasts. - Mitochondria. - The citric acid cycle does not occur in plant cells.

Mitochondria.

Where does the oxidative (oxygen-dependent) stage of the breakdown of food molecules occur in a eukaryotic cell? - Golgi apparatus. - Endoplasmic reticulum. - Cytosol. - Mitochondrion.

Mitochondrion.

Which two processes together constitute the M phase of the cell cycle? - Mitosis and G0. - Interphase and metaphase. - S phase and metaphase. - Interphase and mitosis. - Mitosis and cytokinesis.

Mitosis and cytokinesis.

Mutation of the muscle-specific intermediate filament desmin leads to the rare disease desmin-related myopathy. This disorder starts with weakness of the lower limbs when patients are in their 20s or 30s. As symptoms worsen, weakness in respiratory and cardiac muscles occurs, which can lead to serious problems including sudden cardiac arrest. Which of the following mutations would disrupt desmin intermediate filament structure or function and could explain the symptoms of desmin-related myopathy? Choose all of the possible mutations. (choose one or more). - Mutation such that formation of dimers is blocked. - Alteration in head groups, so tetramers are unable to link end to end. - Alteration in the twist or coiling of the dimers, blocking formation of staggered tetramers. - Disruption of the polarity of the final desmin strands.

Mutation such that formation of dimers is blocked. Alteration in head groups, so tetramers are unable to link end to end. Alteration in the twist or coiling of the dimers, blocking formation of staggered tetramers.

Heart conditions known as cardiomyopathies include two main subtypes. In hypertrophic cardiomyopathies, the heart muscle becomes stiff. Such disorders can be treated by drugs like verapamil, which block calcium channels. In a second class of cardiomyopathies, the heart muscle becomes dilated. These disorders are sometimes treated with the drug digoxin, which blocks Na+ pumps and elevates intracellular Ca2+in cardiac muscle cells. Both types of disorder can be caused by mutations in the genes encoding troponin. Some of these mutations increase troponin's sensitivity to Ca2+; other mutations reduce it. Based on the treatment protocols outlined, what type of mutation is responsible for hypertrophic cardiomyopathy? - Mutations that decrease troponin's sensitivity to verapamil. - Mutations that decrease troponin's sensitivity to digoxin. - Mutations that decrease troponin's sensitivity to Ca2+. - Mutations that increase troponin's sensitivity to Ca2+. - Mutations that either increase or decrease troponin's sensitivity to Ca2+.

Mutations that increase troponin's sensitivity to Ca2+.

When a muscle is stimulated to contract, myosin heads walk along actin filaments in repeated cycles of attachment and detachment. Which of the following represents a correct description of the events in this cycle? - ATP hydrolysis enhances the affinity of myosin for actin; myosin is "cocked" as its head is displaced along the actin filament; myosin attaches to actin; the power stroke puts myosin in a "rigor" configuration. - ATP binding enhances the affinity of myosin for actin; myosin attaches to actin; the power stroke puts myosin in a "rigor" configuration; myosin is "cocked" as its head is displaced along the actin filament. - The power stroke puts myosin in a "rigor" configuration; myosin attaches to actin; myosin is "cocked" as its head is displaced along the actin filament; ATP hydrolysis enhances the affinity of myosin for actin. - ATP binding enhances the affinity of myosin for actin; myosin attaches to actin; myosin is "cocked" as its head is displaced along the actin filament; the power stroke puts myosin in a "rigor" configuration. - Myosin attaches to actin; ATP binding reduces the affinity of myosin for actin; myosin is "cocked" as its head is displaced along the actin filament; the power stroke puts myosin in a "rigor" configuration.

Myosin attaches to actin; ATP binding reduces the affinity of myosin for actin; myosin is "cocked" as its head is displaced along the actin filament; the power stroke puts myosin in a "rigor" configuration.

Soon after nerve growth factor (NGF) was discovered, investigators injected newborn mice with antibodies that inactivated it. Compared to control mice, those that received the antibody showed massive amounts of nerve cell death. After a week of daily antibody injections, up to 99% of the neurons in certain parts of the nervous system of these mice were gone. What do these results suggest? - NGF signaling is sufficient for survival of developing neurons. - NGF is a survival factor for developing neurons. - NGF cannot protect developing neurons from death by apoptosis. - NGF is a mitogen that promotes the proliferation of developing neurons. - NGF triggers apoptosis in developing neurons.

NGF is a survival factor for developing neurons.

When the glucose-Na+ symport protein is in its outward-open state, which is more likely to occur? - The transporter releases glucose into the cell. - Glucose binds to its binding site. - Both solutes bind simultaneously. - Na+ binds to its binding site. - The transporter releases Na+ into the cell.

Na+ binds to its binding site.

Nuclear import is driven by the hydrolysis of GTP, which is triggered by an accessory protein called Ran-GAP (GTPase-activating protein). Which is true of this process? - Nuclear receptors carry Ran-GTP from the nucleus to the cytosol. - Ran-GTP is present in high concentrations in the cytosol. - Ran-GDP displaces proteins from nuclear import receptors inside the nucleus. - Nuclear import receptors have the ability to catalyze hydrolysis of GTP. - Ran-GAP is present exclusively in the nucleus.

Nuclear receptors carry Ran-GTP from the nucleus to the cytosol.

The Na+ pump in the plasma membrane of animal cells uses energy from ATP hydrolysis to pump sodium and potassium ions against their electrochemical gradients. In which direction are the ions pumped across the membrane? - Na+ in and K+ out. - Na+ and K+ both in. - Na+ out and K+ in. - Na+ and K+ both out.

Na+ out and K+ in.

Place the following steps of muscle contraction in the order in which they occur.

Neuron stimulates a muscle cell. Action potential triggers opening of Ca2+-release channel. Ca2+ is released from the sarcoplasmic reticulum Troponin moves tropomyosin protein. Myosin interacts with actin.

The genome of the bacterium E. coli contains 4.6 million (4.6 x 10^6) nucleotide pairs, whereas the human genome contains some 3200 x 10^6 nucleotide pairs. What can be concluded based on these numbers? - E. coli are unicellular, whereas humans are multicellular. - Human cells are larger than E. coli cells. - Human cells have 700 times more genes than E. coli. - Humans are a more complex life-form than E. coli. - All of the above conclusions are drawn correctly. - None of the above conclusions are drawn correctly.

None of the above conclusions are drawn correctly.

Everyone is exposed regularly to ionizing radiation found in the soil, water, and air and from cosmic rays. In fact, 80% of the ionizing radiation people are exposed to comes from naturally occurring sources. Ionizing radiation can cause double-strand breaks in the DNA. Often, the DNA breaks have missing nucleotides at the broken ends. What type of repair would likely be used, and what would be the result of repairing this type of damage? - Nonhomologous end joining would be used to join the DNA, but errors would still remain. - Direct repair would be used to join the DNA, but errors would still remain. - Nonhomologous end joining would be used and would lead to properly repaired DNA. - Direct repair would be used and would lead to properly repaired DNA.

Nonhomologous end joining would be used to join the DNA, but errors would still remain.

Which of these actions occur(s) commonly at a nuclear pore complex? (choose one or more). - Nuclear import receptors enter from the cytosol. - Proteins with a nuclear localization signal enter at the same time that proteins with a nuclear export signal exit. - Small water-soluble molecules are selectively transported into the nucleus. - Protein complexes diffuse into the nucleus. - Nuclear import receptors are exported to the cytosol. - mRNA molecules are exported to the cytosol.

Nuclear import receptors enter from the cytosol. Proteins with a nuclear localization signal enter at the same time that proteins with a nuclear export signal exit. Nuclear import receptors are exported to the cytosol. mRNA molecules are exported to the cytosol.

How does the nuclear pore restrict the passage of large molecules that do not bear the correct nuclear localization signal? - Inbound proteins are captured by the nuclear basket and released by GTP hydrolysis. - The cytosolic fibrils obstruct access to the pore and can only be parted by nuclear import receptors. - The pores remain closed until they are stimulated by the binding of proteins with the proper localization signal. - Nuclear pore proteins contain disordered segments that form a gel-like meshwork inside the pore. - The hydrophobic interior of the pore repels proteins that lack the correct nuclear localization signal.

Nuclear pore proteins contain disordered segments that form a gel-like meshwork inside the pore.

Optogenetics is a powerful tool that uses light to control the activity of specific neurons. These neurons contain artificially introduced light-gated ion channels. A number of different light-gated channels with different ion specificities have been either found in nature (such as the sodium-specific channelrhodopsin, originally found in green algae) or produced via genetic engineering (the production of a chloride ion-specific form of channelrhodopsin). There are also light-gated ion channels specific for potassium or calcium. Sort each light-activated channel type based on whether activation of this channel will tend to depolarize cells or not. - Sodium channel. - Calcium channel. - Chloride channel. - Potassium channel.

Promote depolarization: Sodium channel. Calcium channel. Inhibit depolarization: Chloride channel. Potassium channel.

To begin transcription, eukaryotic RNA polymerase recognizes nucleotide sequences in what region of the DNA? - G-C-rich region. - Terminator region. - Initiator region. - Replication origin. - Promoter region.

Promoter region.

MicroRNAs regulate the activity of protein-coding genes by doing which of the following? (choose one or more). - Inhibiting the transcription of mRNAs. - Promoting the degradation of mRNAs. - Inhibiting the translation of mRNAs.

Promoting the degradation of mRNAs. Inhibiting the translation of mRNAs.

The long noncoding RNA Xist silences genes on the X chromosome by doing what? - Encoding a repressor protein. - Acting as an antisense transcript that binds to mRNAs. - Producing miRNAs. - Promoting the formation of heterochromatin. - Producing siRNAs.

Promoting the formation of heterochromatin.

What is the name of the complex that degrades proteins that have reached the end of their lifespan, are damaged, or are misfolded? - Proteasome. - Nuclear pore complex. - Ribosome. - Nuclease.

Proteasome.

In a fluorescence recovery after photobleaching (FRAP) experiment, a fluorescently tagged membrane protein, A, shows very little recovery of fluorescence ten minutes after photobleaching, while membrane protein B shows a rapid increase in fluorescence after bleaching, recovering nearly 80% of its original fluorescent signal by ten minutes. Based on this information, which of the following statements can be made? - Protein B is diffusing in a more fluid membrane. - Protein A has faster mobility in the membrane. - Proteins A and B interact with each other in a complex. - Protein A is easily denatured.

Protein B is diffusing in a more fluid membrane.

Which of the following statements are true of mitochondrial matrix proteins that are synthesized in the cytosol? (choose one or more). - Protein transport into mitochondria is powered in part by ATP hydrolysis. - Proteins usually have a signal sequence at their C-terminus. - Chaperone proteins help draw the proteins inside the mitochondrion. - Proteins are unfolded as they are transported into the organelle. - Proteins are transported across the mitochondrial membranes while being synthesized. - Proteins cross both the inner and outer mitochondrial membranes as they are imported.

Protein transport into mitochondria is powered in part by ATP hydrolysis. Chaperone proteins help draw the proteins inside the mitochondrion. Proteins are unfolded as they are transported into the organelle. Proteins cross both the inner and outer mitochondrial membranes as they are imported.

Why does the nuclear envelope break down at the start of prometaphase? - The sister chromatids separate. - The replicated chromosomes decondense. - The growing mitotic spindle pushes through the nuclear envelope. - Nuclear lamins are unstable filaments. - Proteins that form the nuclear pores and nuclear lamina become phosphorylated.

Proteins that form the nuclear pores and nuclear lamina become phosphorylated.

Some types of bacteria can survive under both aerobic and anaerobic conditions. Regardless of whether oxygen is present, these cells maintain a proton gradient across the plasma membrane to drive ATP synthesis and the import of nutrients. Under aerobic conditions, an H+gradient across the plasma membrane is produced by the transfer of electrons along the respiratory chain. When oxygen is present, what would be expected to occur in the plasma membrane of these bacteria? - ATP synthase hydrolyzes ATP, pumping protons out of the cell to help maintain the H+ gradient. - ATP synthase hydrolyzes ATP, pumping protons into the cell to help maintain the H+ gradient. - Electrons flow into the cell through ATP synthase, generating ATP. - Protons flow out of the cell through ATP synthase, generating ATP. - Protons flow into the bacterial cell through ATP synthase, generating ATP.

Protons flow into the bacterial cell through ATP synthase, generating ATP.

In the electron-transport chain, as electrons move along a series of carriers, they release energy that is used to do what? - Phosphorylate ADP to form ATP. - Split water into protons and oxygen. - Oxidize food molecules. - Pump protons across a membrane. - Hydrolyze ATP.

Pump protons across a membrane.

Which protein complex mediates the RNAi silencing process by inhibiting RNA polymerase via histone methylation and heterochromatin formation? - RISC - RITS. - Mediator. - RNase.

RITS.

Which of these is an evolutionarily ancient defense mechanism that protects a variety of organisms from viral infection? - Transcription. - Epigenetic inheritance. - DNA methylation. - Reverse transposition. - RNA interference (RNAi).

RNA interference (RNAi).

In the late 1920s, bacteriologist Fred Griffith was studying Streptococcus pneumoniae. This bacterium comes in two forms: one that is highly infectious (called the "S strain" because it forms colonies that appear smooth when grown on a nutrient plate in the lab) and one that is relatively harmless (called the "R strain" because its colonies appear rough). When injected into mice, the S strain is lethal, whereas the R strain causes no ill effect. Griffith confirmed that when the S strain is killed by heating, it is no longer infectious. But he then discovered that if he injected mice with both the heat-killed S strain pneumococci and the live, harmless R strain bacteria, the animals died of pneumonia. Furthermore, their blood was swarming with live, S strain bacteria that, when grown in culture, remained infectious and lethal. Based on these results, what could Griffith conclude? - The infectious S strain of bacteria cannot be killed by heating. - The R strain of bacteria is more deadly than previously thought. - DNA is conclusively confirmed as the genetic material of cells. - S. pneumoniae is a poor choice for investigating the molecular basis of heredity, as most bacteria behave unpredictably in the laboratory. - Some substance in the infectious S strain can change the harmless R strain into the more lethal form.

Some substance in the infectious S strain can change the harmless R strain into the more lethal form.

Microtubules are inherently unstable unless they are: - Bound to kinesin. - Assembled in the nucleus. - Stabilized by a (+) end attachment. - Able to bind GTP.

Stabilized by a (+) end attachment.

Dynamic instability is the ability of a microtubule to do which of the following? - Rapidly switch its plus and minus ends. - Grow rapidly during mitosis. - Switch back and forth between polymerization and depolymerization. - Shrink rapidly during mitosis.

Switch back and forth between polymerization and depolymerization.

What is the name of the specialized junction between a neuron and a target cell? - Synapse. - Dendrite. - Axon. - Synaptic vesicle. - Nerve terminal.

Synapse.

What is the source of energy for protein translocation into the ER? - Hydrolysis of ATP by the translocator. - Hydrolysis of GTP by the translocator. - The action of chaperones. - Hydrolysis of ATP by the SRP receptor. - Synthesis of the protein.

Synthesis of the protein.

Researchers sought to grow blood vessels in a lab for implantation into patients with clogged coronary arteries. To avoid immune rejection, they wanted to grow vessels by starting with patients' own vascular smooth muscle cells (SMCs). However, most patients requiring coronary artery replacement are elderly, and their vascular SMCs do not divide often enough to grow a new vessel. To get around this problem, the researchers introduced into the cells the gene coding for a particular protein in a format through which the gene would be transcribed and the protein produced. What gene was introduced into the elderly patients' SMCs to allow continued cell divisions? - DNA polymerase. - DNA ligase. - Primase. - Telomerase.

Telomerase.

What is the enzyme that replicates the ends of linear chromosomes called? - Telomerase. - DNA ligase. - Helicase. - DNase. - RNA polymerase.

Telomerase.

Prokaryotes have chromosomes that are circular in structure. Which of the following would such chromosomes lack? - Complementary base pairs. - Sugar-phosphate backbone. - DNA double helix. - Telomeres. - Replication origin

Telomeres.

The low redox potential of NADH means that it has a: - Very stable bond. - Low free energy. - Tendency to give up electrons. - High electron affinity.

Tendency to give up electrons.

Which of the following are activated by many receptor tyrosine kinases (RTKs)? (choose one or more). - The MAP-kinase signaling module. - Phospholipase C. - The monomeric GTP-binding protein Ras. - Phosphoinositide 3-kinase. - Trimeric GTP-binding proteins.

The MAP-kinase signaling module. Phospholipase C. The monomeric GTP-binding protein Ras. Phosphoinositide 3-kinase.

Some applications in biology, such as polymerase chain reaction (PCR), require melting the DNA double helix into single strands of DNA. This can be accomplished by heating the DNA. As DNA is heated, why does the double helix structure denature into single strands of DNA but not into individual nucleotides? In other words, why do the single strands remain intact even though the double helix does not? - The double helix is held together with hydrogen bonds, while the single strands are linked by phosphodiester bonds. - The double helix structure is important for DNA replication, while the single strands carry the information. - The single strands are wrapped around histones, which protects them from denaturing. - The double helix is held together with phosphodiester bonds, while the single strands are linked by hydrogen bonds.

The double helix is held together with hydrogen bonds, while the single strands are linked by phosphodiester bonds.

What is one reason that plants do not generally produce sugars after dark? - O2 is unavailable after dark. - The enzymes involved in stage 2 of photosynthesis are inactivated in the dark. - CO2 is unavailable after dark.The enzymes involved in stage 2 of photosynthesis are inactivated in the light. - The enzymes involved in stage 1 of photosynthesis are inactivated in the dark.

The enzymes involved in stage 2 of photosynthesis are inactivated in the dark.

Which of the following statements most accurately describes the expression of the repressor protein of the tryptophan operon? - The gene for the tryptophan repressor is turned on in response to high levels of tryptophan in the cell. - The gene for the tryptophan repressor is turned off in response to high levels of tryptophan in the cell. - The gene for the tryptophan repressor is turned on in response to low levels of tryptophan in the cell. - The gene for the tryptophan repressor is turned off in response to low levels of tryptophan in the cell. - The gene for the tryptophan repressor is expressed constitutively.

The gene for the tryptophan repressor is expressed constitutively.

In the 1920s, bacteriologist Fred Griffith demonstrated that a heat-killed, infectious pneumococcus produced a substance that could convert a harmless form of the bacterium into a lethal one. Fifteen years later, researchers prepared an extract from the disease-causing S strain of pneumococci and showed that this material could transform the harmless R-strain pneumococci cells into the infectious S-strain form. This change to the bacteria was both permanent and heritable, suggesting that this "transforming principle" represents the elusive genetic material of the cells. The researchers subjected their extract to a variety of tests to determine the chemical identity of the "transforming principle." In one experiment, they treated the material with enzymes that destroy all proteins. This treatment did not affect the ability of the extract to transform harmless bacteria into an infectious form. From this result, what could the researchers conclude? - The transforming principle is not DNA. - DNA acts as the genetic material. - Proteins act as the genetic material. - The transforming principle is not genetic material. - The genetic material is not protein.

The genetic material is not protein.

Translation proceeds in a series of steps in the active site of the ribosome. Which of the following are important steps in polypeptide formation? (choose one or more). - The hydrogen bonds formed between ribosomal RNA and tRNAs position the aminoacyl-tRNAs to catalyze peptide bond formation. - The incoming aminoacyl-tRNA forms a covalent bond with the RNA in the A site. - The tRNA holding the growing polypeptide moves from the P site to the A site after peptide bond formation. - The RNA in the P site makes hydrogen bonds with the 3' end of the aminoacyl-tRNA.

The hydrogen bonds formed between ribosomal RNA and tRNAs position the aminoacyl-tRNAs to catalyze peptide bond formation. The RNA in the P site makes hydrogen bonds with the 3' end of the aminoacyl-tRNA.

In an experiment conducted in 1962, investigators took tRNAs bearing cysteine and chemically converted the charged amino acid to an alanine. They then introduced these "hybrid" alanine-bearing tRNAs into a cell-free translation system from which they removed all of the normal, cysteine-bearing tRNAs. How did this chemical manipulation affect the proteins produced by this altered system? - Proteins were truncated at the position of the first cysteine because the hybrid tRNAs no longer recognized the codon that specifies incorporation of cysteine. - The proteins had a normal amino acid sequence because the ribosome checked for the accuracy of the amino acids it incorporated into the growing polypeptide chain. - The proteins were slightly "shorter" than normal because, with no cysteine-bearing tRNAs present, the ribosome skipped over codons specifying cysteine. - Proteins were truncated at the position of the first cysteine because ribosomes detected an amino acid-tRNA mismatch and halted translation. - The proteins contained alanines where cysteines were supposed to be.

The proteins contained alanines where cysteines were supposed to be.

Scientists were studying yeast cells in which a mutation inactivated a gene required for cell division. Without this gene—and the protein it encoded—these mutant yeast cells were unable to divide normally. But the scientists discovered that introducing a related protein from human cells could "rescue" these mutant yeast cells, allowing the cells to resume normal division. Based on this finding, what is the most likely conclusion you could make? - Yeast cells require human proteins to divide. - Yeast and humans diverged from a common ancestor much more recently than previously thought, perhaps fewer than a million years ago. - The proteins that control cell division in yeast and humans are functionally equivalent and have been conserved, almost unchanged, for more than a billion years. - Modern humans most likely obtained their cell-division proteins from ancient yeast. - Yeast cells most likely obtained their cell-division proteins from the cells of early humans.

The proteins that control cell division in yeast and humans are functionally equivalent and have been conserved, almost unchanged, for more than a billion years.

Your friend now has the pumps successfully pumping ions. She added an equal concentration of both ions to the correct sides of the liposomes along with an excess of the energy source. She is surprised when the pumps stop working after a short time. Which of the following could explain why the transporter stopped pumping ions? - The pump ran out of Na+ to pump because it pumps 3 Na+ out for every 2 K+ pumped in. - The pump ran out of K+ to pump because it pumps 3 K+ out for every 2 Na+ pumped in. - The pump ran out of both Na+ and K+ because an equal number of both ions is pumped in each cycle. - The liposomes ran out of pumps to pump ions.

The pump ran out of Na+ to pump because it pumps 3 Na+ out for every 2 K+ pumped in.

The main regulatory step of glycolysis occurs in step 3. Choose all of the following that correctly describe some aspect of step 3 in glycolysis. (choose one or more). - The reaction is an irreversible reaction. - The enzyme that catalyzes step 3 is phosphofructokinase. - The enzyme uses an ATP. - The reaction generates the product fructose 1,6-bisphosphate.

The reaction is an irreversible reaction. The enzyme that catalyzes step 3 is phosphofructokinase. The enzyme uses an ATP. The reaction generates the product fructose 1,6-bisphosphate.

What happens after the DNA polymerase on the lagging-strand template completes an Okazaki fragment? - The sliding clamp attached to the polymerase dissociates from the DNA, and the polymerase temporarily releases the lagging-strand template. - The sliding clamp pushes the DNA polymerase to the next RNA primer on the lagging strand. - The Okazaki fragment dissociates from the lagging-strand template, attracting a DNA ligase. - The DNA polymerase dissociates from the replication complex and is replaced by a new polymerase. - DNA ligase removes the RNA primer and replaces it with DNA to form a continuous new DNA strand.

The sliding clamp attached to the polymerase dissociates from the DNA, and the polymerase temporarily releases the lagging-strand template.

During photosynthesis, charge separation takes place when which of the following occurs? - The special pair passes an electron to an electron carrier in the antenna complex, leaving behind a negative charge. - The special pair passes an electron to an electron carrier in the reaction center, leaving behind a negative charge. - Energy jumps randomly from one chlorophyll molecule to another in the antenna complex. - The special pair passes an electron to an electron carrier in the reaction center, leaving behind a positive charge. - Light energy is captured by a chlorophyll molecule in the antenna complex. - The special pair passes an electron to an electron carrier in the antenna complex, leaving behind a positive charge.

The special pair passes an electron to an electron carrier in the reaction center, leaving behind a positive charge.

In eukaryotic cells, why must metabolism be tightly regulated? - The substrates involved in metabolic reactions are each recognized by only a single, unique enzyme. - Anabolic and catabolic pathways must compete for oxygen. - Anabolic and catabolic pathways must compete for scarce resources. - The substrates involved in metabolic reactions can be used by a number of different enzymes. - All metabolic reactions require energy.

The substrates involved in metabolic reactions can be used by a number of different enzymes.

What are two types of noncoding regulatory RNAs? - siRNAs and miRNAs. - miRNAs and rRNAs. - mRNAs and siRNAs. - rRNAs and mRNAs.

siRNAs and miRNAs.


Related study sets

Module #1: Database Systems (Review)

View Set

Combo with "The Art of Public Speaking - Chapter 8" and 1 other

View Set

nobles__ch 05__merchandising operations

View Set

Operations Instructional Area Exam (PQ, CS)

View Set

Male Reproductive Pathology (Ch. 14)

View Set

Lesson 2.1 Who am I? Think again

View Set